You are on page 1of 120

ΜΑΘΗΜΑΤΙΚΟ

ΒΗΜΑ
Μαθηματικό Περιοδικό
Δεκέμβριος 2013
για το Γυμνάσιο και το Λύκειο

r 2
= γ
2 +y = 2
x + βy
x a
6ο Ευρωπαϊκό Μαθητικό Συνέδριο
για τα Μαθηματικά
2
+ β
+22 αβ
β
2
) =α
(α +

ΚΥΠΡΙΑΚΗ ΜΑΘΗΜΑΤΙΚΗ ΕΤΑΙΡΕΙΑ


Περιεχόμενα

Μαθηματικό Βήμα
ΚΥΠΡΙΑΚΗ ΜΑΘΗΜΑΤΙΚΗ ΕΤΑΙΡΕΙΑ
Στασίνου 36, Γραφείο 102, Στρόβολος 2003, Λευκωσία, Κύπρος
Τηλ. +35722378101
Φαξ. +35722379122
e-mail: cms@cms.org.cy

Επιμέλεια Έκδοσης

Γρηγόρης Μακρίδης, Ανδρέας Φιλίππου, Θεόκλητος Παραγιού, Δημήτρης Καραντάνος

α/α Θέμα Σελίδα

Le-MATH:
1 1
«Μαθαίνοντας Μαθηματικά μέσω νέων παραγόντων επικοινωνίας»
Γρηγόρης Μακρίδης
Μαθηματικές Ολυμπιάδες
2 «Τεχνικές στην Αλγεβρική Συνδυαστική» 5
Γεώργιος Χαραλάμπους
Κατασκευές τριγώνων με λογισμικό γεωμετρίας
3 ή με χάρακα και διαβήτη (Α΄ & Β΄ Γυμνασίου) 21
Κωνσταντίνος Παπαγιάννης

Παραγοντοποίηση – Εφαρμογές (Γ΄ Γυμνασίου)


4 28
Σάββας Τιμοθέου
Ένα πρόβλημα, πολλές λύσεις (Α΄ Λυκείου)
5 40
Κωνσταντίνος Παπαγιάννης, Σάββας Τιμοθέου

Μαθηματική επαγωγή (Β΄ Λυκείου)


6 43
Ελένη Παπαθωμά

Εύρεση συνόλου τιμών συνάρτησης με εναλλακτικές μεθόδους (Β΄ Λυκείου)


7 48
Παντελής Ζαμπυρίνης

Λίγη εξάσκηση πριν τις Παγκύπριες εξετάσεις (Γ΄ Λυκείου)


8 51
Αναστασία Ηρακλέους
Περιεχόμενα

Μαθηματικό Βήμα
Παγκύπριες εξετάσεις 2013
9 Μαθηματικά 60

Παγκύπριες εξετάσεις 2013


10 Μαθηματικά κ.κ. 73

Ο Αρχιμήδης, η Γραμμική Άλγεβρα και τα Βόδια


11 83
Κ. Κουππάρης
Μεθοδολογία για Λύση Προβλήματος
12 Ανδρέας Σκοτεινός 88

Διασκεδάζοντας με τα Μαθηματικά
13 99
Ανδρέας Σκοτεινός
Πρωτότυπες ασκήσεις από τα θέματα Kangourou με τις λύσεις
14 105
Ανδρέας Σαββίδης

15 Δραστηριότητες της ΚΥ.Μ.Ε. κατά το 2013 108


Παιδεία

Μαθηματικό Βήμα

Le-MATH: ΜΑΘΑΙΝΟΝΤΑΣ ΜΑΘΗΜΑΤΙΚΑ ΜΕΣΩ ΝΕΩΝ ΠΑΡΑΓΟΝΤΩΝ ΕΠΙΚΟΙΝΩΝΙΑΣ

Γρηγόρης Μακρίδης, Κυπριακή Μαθηματική Εταιρεία και συνεργάτες*,


makrides.g@ucy.ac.cy, cms@cms.org.cy, www.le-math.eu

Ευρωπαϊκό Έργο Le-MATH: Learning mathematics through new communication factors,


526315-LLP-2012-CY-COMENIUS-CMP

Περίληψη
Δυστυχώς πολλοί μαθητές και γονείς θεωρούν τα μαθηματικά ως ένα θέμα δύσκολο και ανιαρό. Αντί να
μελετούν μαθηματικά (και άλλα θέματα) πολλοί μαθητές προτιμούν να αφιερώνουν το μεγαλύτερο μέρος
του χρόνου τους παρακολουθώντας διάφορα προγράμματα στην τηλεόραση ή παίζοντας ηλεκτρονικά
παιχνίδια, ανταλλάσσοντας μηνύματα με το κινητό τους τηλέφωνο, ανταλλάσσοντας φωτογραφίες ή
βίντεο κλπ. Ένας τρόπος να φέρουμε τους μαθητές πίσω στο «Παιχνίδι» της εκπαίδευσης είναι να
χρησιμοποιήσουμε παρόμοια (όπλα) εργαλεία, με τους «αντίπαλους», δηλαδή η επικοινωνία στη
μάθηση των μαθηματικών με ένα μη παραδοσιακό τρόπο, όπως ένα θεατρικό παιχνίδι ή μέσω
διαγωνισμών παρόμοιων με το γνωστό X-Factor και άλλα. Το έργο Le-MATH, το οποίο χρηματοδοτείται
από την Ευρωπαϊκή Επιτροπή με συντονιστή την Κυπριακή Μαθηματική Εταιρεία και 12 συνεργαζόμενους
φορείς ανάλαβε τη δημιουργία αυτών των εργαλείων από τον Νοέμβρη του 2012 με ολοκλήρωση τον
Οκτώβρη του 2014. Σε αυτή τη δημοσίευση θα παρουσιαστούν τα αποτελέσματα του πρώτου έτους
λειτουργίας του έργου.

1. Εισαγωγή
Πολλοί μαθητές ισχυρίζονται ότι τα μαθηματικά είναι πολλές φορές αφηρημένα και ως εκ τούτου
απρόσιτα, έτσι το έργο αυτό χρησιμοποιεί μια τελείως διαφορετική και νέα προσέγγιση καλώντας
καθηγητές και μαθητές να εφαρμόσουν νέες μεθόδους επικοινωνίας στην εκμάθηση των μαθηματικών,
που θα μπορούσαν να είναι διασκεδαστικές και απολαυστικές την ίδια στιγμή. Μια προσέγγιση που
φέρνει νέες ιδέες στο πλαίσιο «μαθαίνω παίζοντας».
Αυτό το Ευρωπαϊκό έργο προτίθεται να αναπτύξει νέες μεθοδολογίες στην εκμάθηση και τη διδασκαλία
των μαθηματικών για μαθητές ηλικίας 9-18, και μπορεί να χρησιμοποιηθεί σε οποιοδήποτε σχολικό
περιβάλλον. Επίσης θα κάνει την μάθηση πιο ελκυστική και απολαυστική για όλους τους μαθητές και θα
ενισχύσει τις δεξιότητες των μαθητών για δημιουργική σκέψη. Οι μέθοδοι αυτοί θα μπορούσαν να
χρησιμοποιηθούν και σε άλλα αντικείμενα του αναλυτικού προγράμματος καθώς και σε άλλες ηλικίες.

1
Παιδεία

Μαθηματικό Βήμα

Οι εταίροι του προγράμματος προέρχονται από πανεπιστήμια, σχολεία, Μαθηματικές οργανώσεις,


οργανισμούς, θεατρικά σχολεία, σχολεία καλών τεχνών και επιχειρήσεις.
Οι δραστηριότητες του έργου συμβάλλουν στο Εκπαίδευση και Κατάρτιση 2020 καθώς ενισχύει την
δημιουργικότητα και την καινοτομία μεταξύ των νέων. Συμβάλλει επίσης στο ορόσημο για τα άτομα με
μειωμένες επιδόσεις σε βασικές δεξιότητες (μαθηματικά και την επιστήμη) σε 15%. Προωθεί την
ευρωπαϊκή συνεργασία σε σχολεία στον τομέα των ικανοτήτων, υποστηρίζοντας την βασική ικανότητα για
τα μαθηματικά.

2. Στόχος
Στόχος του έργου αυτού είναι η ανάπτυξη της μεθοδολογίας στη διδασκαλία και την μάθηση των
μαθηματικών με τη δημιουργία δύο κύριων εργαλείων που μπορούν να χρησιμοποιηθούν από τους
εκπαιδευτικούς. Τα εργαλεία θα δημιουργηθούν με τέτοιο τρόπο έτσι ώστε να μπορούν να
χρησιμοποιηθούν για επιμόρφωση και κατάρτιση των εκπαιδευτικών που διδάσκουν μαθηματικά σε
μαθητές ηλικίας 9-18.
Τα δύο εργαλεία είναι:
A. MATHeatre: Διδασκαλία και μάθηση μαθηματικών μέσω μαθηματικού θεάτρου
B. MATHFactor: Διδασκαλία και μάθηση μαθηματικών μέσω της επικοινωνίας των μαθηματικών
Αυτές οι νέες μέθοδοι αναμένεται να ανταγωνίζονται με τα σύγχρονα ενδιαφέροντα και δραστηριότητες
που έχουν οι νέοι ηλικίας 9-18. Το έργο αναπτύσσεται μέσω εννέα πακέτων εργασιών. Παρακάτω
περιγράφονται μερικά από αυτά.

3. Καλές Πρακτικές στον Ευρωπαϊκό Χώρο


Σε αυτό το πακέτο εργασιών πραγματοποιήθηκε μια συλλογή πρακτικών σε σχέση με το αντικείμενο, η
οποία αναπτύχθηκε σε ηλεκτρονικό εγχειρίδιο (e-book) με δομή ευρετηρίου κατά πόσο αφορά την μέθοδο
Α ή Β, αν είναι μελέτη, έκθεση, πρακτική, αξιολόγηση, βίντεο κ.α. Στο εγχειρίδιο αυτό μπορεί να βρει
κανείς ότι έγινε ή γίνεται στον τομέα αυτό.
Το εγχειρίδιο σε τρέχουσα έκδοση βρίσκεται στην ιστοσελίδα του έργου www.le-math.eu

4. MATHeatre
Το Μαθηματικό Θέατρο βασίζεται σε όλους του κανόνες θεατρικής παράστασης αλλά με αντικείμενο που
να σχετίζεται με μαθηματικά και με πρωταγωνιστές μαθητές ηλικίας 9-18 ετών. Αυτό μπορεί να έχει όλες
τις μορφές θεάτρου όπως δράμα, κωμωδία, μουσικοχορευτικό κ.α.
Το αντικείμενο των μαθηματικών μπορεί να είναι οτιδήποτε μέρος αναλυτικού προγράμματος ή ιστορίας
των μαθηματικών. Η δυσκολία βρίσκεται στη δημιουργία των διαλόγων των μαθητών «ηθοποιών» ώστε
το αποτέλεσμα να δίνει ολοκληρωμένη μαθηματική γνώση.

2
Παιδεία

Μαθηματικό Βήμα

Η αρχική έκδοση του εγχειριδίου οδηγιών «MATHeatre Guidebook» που δημοσιεύθηκε το Σεπτέμβρη του
2013 περιέχει περιγραφή διαδικασίας και δείγματα θεατρικών έργων έτοιμα για χρήση στο σχολικό
περιβάλλον.
Το έργο προκήρυξε διαγωνισμό συγγραφής τέτοιων έργων μέσω του οποίου θα δημοσιευθούν δείγματα
στην ιστοσελίδα του έργου. Το έργο ανάπτυξε επίσης έκδοση με τίτλο «Μαθηματικές Ιστορίες για
Θέατρο» οι οποίες μπορούν να χρησιμοποιηθούν για αυτό το σκοπό.
Κατά το δεύτερο έτος του έργου προκηρύχθηκε Ευρωπαϊκός διαγωνισμός με διεθνή συμμετοχή
ΜΑΤΗeatre EUROPE 2014 στον οποίο καλούνται σχολεία ή οργανισμοί ή ομάδες μαθητών να
συμμετάσχουν με θεατρική παράσταση διάρκειας από 5-12 λεπτών και με ελάχιστο αριθμό ηθοποιών 2
και μέγιστο 10. Στην πρώτη φάση , από Σεπτέμβρη 2013 μέχρι 7 Φεβρουαρίου 2014 οι συμμετέχοντες θα
αναρτούν το Μαθηματικό Θέατρο σε u-tube μορφή στο διαδίκτυο, μέσω ειδικής πλατφόρμας. Μετά από
πρώτη αξιολόγηση θα προσκληθούν οι καλύτεροι πέντε τουλάχιστο σε δύο διαφορετικές ομάδες ηλικιών ,
9-13 και 14-18 για τον τελικό που θα γίνει στο πλαίσιο του μαθητικού συνεδρίου EUROMATH 2014, από
24-28 Απριλίου 2014.
Τα κριτήρια αξιολόγησης των έργων μαθηματικού θεάτρου είναι μέρος του εγχειριδίου οδηγού σε δύο
μορφές, η μία για χρήση εντός σχολικής δραστηριότητας και η άλλη σε περίπτωση ανοικτού δημόσιου
διαγωνισμού όπως το MATHeatre Europe 2014.

5. MATHFactor
To MATHFactor είναι δραστηριότητα ατομικής επικοινωνίας των μαθηματικών, δηλαδή ο μαθητής
καλείται να παρουσιάσει και να εξηγήσει μέσα σε μέγιστο χρόνο 3 λεπτών μαθηματικές έννοιες, θεώρημα,
εφαρμογή, ιστορία των μαθηματικών, ιδιότητες κ.α. με εκλαϊκευμένη μορφή ώστε να μπορεί να το
καταλάβει και ο μη ειδικός ή ο πρωτομαθείς μαθητής. Η παρουσίαση δεν χρησιμοποιεί εργαλεία
προβολής και πίνακα αλλά αν χρειάζεται μικρά αντικείμενα που μπορεί να μεταφέρει ο μαθητής με το ένα
χέρι.
Μια καλή παρουσίαση αξιολογείται για το μαθηματικό περιεχόμενο της, για τη σαφήνεια σε ότι αφορά
την παρουσίαση των εννοιών και την πρωτοτυπία στην προσέγγιση ώστε να γίνουν οι μαθηματικές έννοιες
κατανοητές και ενδιαφέρουσες και το ταλέντο που θα δείξει ο μαθητής στην προφορική του επικοινωνία
και γλώσσα.
Η όλη προσέγγιση μοιάζει με το γνωστό τηλεοπτικό διαγωνισμό X-Factor αλλά γίνεται με αντικείμενο τα
μαθηματικά. Η μέθοδος μπορεί να εφαρμοστεί ως εκπαιδευτική δραστηριότητα εντός της σχολικής τάξης
ή σε δημόσιο διαγωνισμό.
Κατά το δεύτερο έτος του έργου προκηρύχθηκε Ευρωπαϊκός διαγωνισμός με διεθνή συμμετοχή
ΜΑΤΗFactore EUROPE 2014 στον οποίο καλούνται μαθητές να συμμετάσχουν με παρουσίαση διάρκειας 3
λεπτών μέγιστο. Στην πρώτη φάση, από Σεπτέμβρη 2013 μέχρι 7 Φεβρουαρίου 2014 οι συμμετέχοντες θα
αναρτούν την ΜΑΤΗFactor παρουσίαση του/της σε u-tube μορφή στο διαδίκτυο, μέσω ειδικής
πλατφόρμας. Μετά από πρώτη αξιολόγηση θα προσκληθούν οι καλύτεροι πέντε τουλάχιστο σε δύο

3
Παιδεία

Μαθηματικό Βήμα

διαφορετικές ομάδες ηλικιών , 9-13 και 14-18 για τον τελικό που θα γίνει στο πλαίσιο του μαθητικού
συνεδρίου EUROMATH 2014, από 24-28 Απριλίου 2014.
Τα κριτήρια αξιολόγησης για την επικοινωνία των μαθηματικών τύπου MATHFactor είναι μέρος του
εγχειριδίου οδηγού σε δύο μορφές, η μία για χρήση εντός σχολικής δραστηριότητας και η άλλη σε
περίπτωση ανοικτού δημόσιου διαγωνισμού.

6. Πειραματισμός και Αξιολόγηση


Ο πειραματισμός και αξιολόγηση θα γίνει σε διάφορες φάσεις και επίπεδα.
MATHeatre EUROPE 2014
MATHFactor EUROPE 2014
Η όλη προσπάθεια θα πειραματιστεί σε επίπεδο διεθνούς διαγωνισμού σε δύο ηλικιακά επίπεδα (9-13 και
14-18) , ώστε να εξυπηρετείται ο στόχος αλλά και τα αναγκαία κίνητρα για να κεντρίσουν το ενδιαφέρον
σε μαθητές και εκπαιδευτικούς. Ο διαγωνισμός προκηρύσσεται το Σεπτέμβρη του 2013 με την πρώτη
φάση σε “on-line” συμμετοχή μέχρι 7 Φεβρουαρίου 2014 και μετά την αξιολόγηση θα δημιουργηθεί ο
κατάλογος των φιναλίστ οι οποίοι θα προσκληθούν στο ζωντανό τελικό στο πλαίσιο του EUROMATH 2014.
Στη διαδικασία θα αξιολογηθεί η εμπλοκή και δραστηριότητα των μαθητών στην προετοιμασία της
συμμετοχής τους καθώς και ο ρόλος και εντυπώσεις με σχόλια για τα εγχειρίδια οδηγούς από τους
εκπαιδευτικούς που υποστήριξαν τους μαθητές.
Αξιολόγηση θα πραγματοποιηθεί και για το ζωντανό τελικό και όλα τα αποτελέσματα θα χρησιμοποιηθούν
για να βελτιωθούν τα εγχειρίδια οδηγοί για τις δύο μεθόδους και για την προετοιμασία της επόμενης
προκήρυξης των διαγωνισμών ώστε να εξασφαλιστεί η αειφόρα ανάπτυξη των παραδοτέων του έργου
καθώς και η αποτελεσματική εκμετάλλευση τους.
Μέρος της αειφορίας του έργου αποτελεί επίσης η δημιουργία πενθήμερου επιμορφωτικού
προγράμματος για εκπαιδευτικούς το οποίο θα προσφέρεται ως σεμινάριο τύπου Comenius ή Grundtvig
και θα εγγραφεί στην σχετική πλατφόρμα του “Europa Course Base”.
* Συντονιστικό ίδρυμα του έργου είναι Κυπριακή Μαθηματική Εταιρεία-Cyprus Mathematical Society(CY-
Gr. Makrides, A. Philippou, C. Papayiannis) μαζί με 12 συνεργάτες από την Κύπρο, Ελλάδα, Βουλγαρία,
Ρουμανία, Αυστρία, Σουηδία, Γαλλία, Ισπανία, Τσεχία, Βέλγιο και Ουγγαρία. Οι συνεργαζόμενοι φορείς
είναι
Thales Foundation of Cyprus(CY-A. Skotinos, P. kenderov, E. Christou), Charles University in Prague-Faculty
of Education(CZ-J. Novotna, A. Jancarik, K. Jancarikova ), Loidl-Art (AT-H. Loidl), VUZF University(BG-S.
Grozdev), “CALISTRAT HOGAS” National College Piatra-Neamt (RO-N. Circu, L-M Filimon), Lyckeskolan (SE-
M. Manfjard Lydell), LEOLAB (ES-M. Munoz), Junior Mathematical Society Miskolc(HU-P. Kortesi), European
Office of Cyprus(BE-CY-R. Strevinioti), Collège Saint Charles(FR-K. Treguer, E. Gueguen), National Technical
University of Athens, Institute of Communication and Computer Systems(GR-K. Karpouzis), Com2go Ltd(CY-
G. Economides, N. Nirou).

4
Μαθηματικές Ολυμπιάδες

Τεχνικές στην Αλγεβρική Συνδιαστική :


Λήμμα Fubini και Συνδιαστικό Nullstellensatz
Γεώργιος Χαραλάμπους
Department of Pure Mathematics and Mathematical Statistics
University of Cambridge

17 Μαΐου 2013

1 Εισαγωγή
Τα προβλήματα Συνδιαστικής και ιδιαιτέρως τα προβλήματα εκείνα που τίθενται σε δια-
γωνισμούς Μαθηματικών υψηλού επιπέδου, δεν φημίζονται για την ευκολία επίλυσης τους.
Ο λόγος είναι κυρίως ότι τα προβλήματα αυτά επιλύονται απο προσεγγίσεις ή μαθηματικές
τακτικές οι οποίες δεν εντοπίζονται εύκολα. Ο μόνος τρόπος για να αποκτήσει κάποιος οι-
κειότητα με τα προβλήματα αυτά είναι να ασχοληθεί με πολλές και διαφορετικές τακτικές
επίλυσης. Ο σκοπός μας στο παρών κείμενο είναι να περιγράψουμε δυο απο τις τακτικές
αυτές, οι οποίες είναι λίγο διαφορετικές. Η διαφορετικότητά τους εντοπίζεται στο οτι, ενώ
τα πιο πολλά προβλήματα Συνδιαστικής, άσχετα με το βαθμό δυσκολίας και περιπλοκότη-
τας, επιλύονται χρησιμοποιώντας μεθόδους άμεσου υπολογισμού (π.χ Αρχή Εγκλεισμού -
Αποκλεισμού, Αρχή του Dirichlet, Αμφιέσεις (= ένα προς ένα και επί συναρτήσεις) μεταξύ
συνόλων κ.λ.π ), οι τεχνικές αυτές μας επιτρέπουν να λύσουμε ένα πρόβλημα χρησιμο-
ποιώντας αλγεβρικές τακτικές, αντί αριθμητικές. Αντί, δηλαδή, να προσπαθούμε να βρούμε
το x, που αντιστοιχεί στον αριθμό αντικειμένων ιδιότητας Α, βρίσκουμε τον αριθμό y που
αντιστοιχεί στον αριθμό αντικειμένων μιας ιδιότητας B με σκοπό να δημιουργήσουμε εξι-
σώσεις που αφορούν τον αριθμό x, των οποίων η επίλυση μας οδηγεί (έμμεσα) στην λύση
του προβλήματος.

2 Λήμμα Fubini
Η μέθοδος αυτή βασίζεται στην απλή παρατήρηση οτι όταν πρέπει να μετρήσουμε τον
αριθμό κάποιων αντικειμένων που μοιράζονται μια συγκεκριμένη ιδιότητα, η απάντηση
πρέπει να είναι η ίδια άσχετα με το ποιο τρόπο μέτρησης έχουμε επιλέξει. Μετρώντας
δηλαδή με διαφορετικούς τρόπους την ίδια ποσότητα, μπορούμε πολλές φορές να διευκο-
λυνθούμε. Ας ξεκινήσουμε με ένα απλό παράδειγμα.
Παράδειγμα 1. Ένα τετράγωνο 15 × 15 αποτελείται απο μοναδιαία τετράγωνα. Χρωματί-
ζουμε κάθε κορυφή των τετραγώνων κόκκινη ή μπλέ. Υπάρχουν 133 κόκκινα σημεία. Δύο
απο αυτά τα κόκκινα σημεία αποτελούν κορυφές του αρχικού μεγάλου τετραγώνου, και 32
άλλα κόκκινα σημεία βρίσκονται στις πλευρές του μεγάλου τετραγώνου. Οι πλευρές των
μικρών τετραγώνων χρωματίζονται σύμφωνα με τον ακόλουθο κανόνα : Άν και οι δύο άκρες
της πλευράς είναι κόκκινες, τότε χρωματίζουμε την πλευρά κόκκινη. Άν οι δύο άκρες της
πλευράς είναι μπλέ, τότε χρωματίζουμε την πλευρά μπλέ. Άν η μια άκρη είναι μπλέ και η
άλλη κόκκινη, τότε χρωματίζουμε την πλευρά κίτρινη. Άν υποθέσουμε πως υπάρχουν 196
κίτρινες πλευρές, πόσες μπλέ πλευρές υπάρχουν;

5
Λύση. Υπάρχουν 15 πλευρές μοναδιαίων τετραγώνων σε κάθε γραμμή, και υπάρχουν συ-
νολικά 16 γραμμές. Άρα υπάρχουν 15 × 16 οριζόντιες πλευρές μοναδιαίων τετραγώνων.
Παρομοίως, υπάρχουν 15 × 16 κάθετες πλευρές μοναδιαίων τετραγώνων. Κατα συνέπεια,
υπάρχουν συνολικά 30×16 = 480 πλευρές. ( Ας μετρήσουμε τώρα τον αριθμό αυτό διαφορε-
τικά. Υπάρχουν 152 = 225 μοναδιαία τετράγωνα. Κάθε τετράγωνο έχει τέσσερεις πλευρές.
Υπάρχουν 60 μοναδιαίες πλευρές στις πλευρές του αρχικού τετραγώνου. Όλες οι πλευρές
που βρίσκονται στο εσωτερικό του αρχικού τετραγώνου έχουν μετρηθεί δυο φορές, επειδή
κάθε μια απο αυτές αποτελεί κοινή πλευρά δυο μοναδιαίων τετραγώνων. Άρα, υπάρχουν
(225 × 4 + 60)/2 = 480 πλευρές).
Υπάρχουν 480 − 196 = 284 πλευρές χρωματισμένες μπλέ ή κόκκινες. Έστω ότι υπάρχουν
r κόκκινες πλευρές. Υπάρχουν τότε 284 − r μπλέ πλευρές. Στη συνέχεια μετρούμε πόσες
φορές εμφανίζονται κόκκινα σημεία ως κορυφές ακρών μοναδιαίων τετραγώνων με δυο
διαφορετικούς τρόπους. Έστω |S| ο αριθμός αυτός.
Υπάρχουν δυο τέτοιες εμφανίσεις σε κάθε κόκκινη πλευρα, μια τέτοια εμφάνιση σε κάθε
κίτρινη πλευρά και καθόλου εμφανίσεις στις μπλε πλευρές. Άρα |S| = 2r + 196.
Όμως, κάθε κόκκινη κορυφή στις γωνίες του αρχικού τετραγώνου εμφανίζεται δύο φορές
στο |S|, κάθε κόκκινη κορυφή που βρίσκεται στις πλευρές του αρχικού τετραγώνου εμ-
φανίζεται τρείς φορές και κάθε κόκκινη κορυφή στο εσωτερικό του αρχικού τετραγώνου
εμφανίζεται τέσσερεις φορές. Άρα,

|S| = 2 × 2 + 32 × 3 + (133 − 2 − 32) × 4 = 496.

Επομένως 2r + 196 = 496 και r = 150. Άρα υπάρχουν 284 − 150 = 134 μπλέ πλευρές.
Ορισμός. Έστω m και n δυο θετικοί ακέραιοι αριθμοί, και έστω A = {a1 , a2 , · · · , am } και
B = {b1 , b2 , · · · , bn } δυο πεπερασμένα σύνολα. Το καρτεσιανό γινόμενο των A και B ορίζεται
ως

A × B = {(a, b) | a ∈ A, b ∈ B}.

Έστω S ένα υποσύνολο του A × B. Αν ai ∈ A, ορίζουμε

S(ai , ⋆) = {(ai , b) ∈ S},

και αν bi ∈ B ορίζουμε

S(⋆, bi ) = {(a, bi ) ∈ S}.

Είμαστε έτοιμοι να παρουσιάσουμε το βασικό Θεώρημα της ενότητας αυτής, το οποίο ονο-
μάζουμε Λήμμα για ιστορικούς λόγους.
Θεώρημα 1 (Λήμμα Fubini). Έστω m και n δυο θετικοί ακέραιοι αριθμοί, και έστω
A = {a1 , a2 , · · · , am } και B = {b1 , b2 , · · · , bn } δυο πεπερασμένα σύνολα. Αν S είναι ένα
υποσύνολο του A × B, τότε

n ∑
m
|S| = |S(⋆, bj )| = |S(ai , ⋆)|.
j=1 i=1

Απόδειξη. Ο πιο εύκολος τρόπος να κατανοήσουμε το Λήμμα αυτό είναι να θημηθούμε τη


διαδικασία πρόσθεσης των στοιχείων ενός πίνακα. Βλέπουμε εύκολα πως το άθροισμα αυτό
ισούται με το άθροισμα των αθροισμάτων των στοιχείων των γραμμών όπως επίσης και με
το άθροισμα των αθροισμάτων των στοιχείων των στηλών.
Έστω ένας m × n πίνακας M = (xi,j ) έτσι ώστε
{
1 αν (ai , bj ) ∈ S,
xi,j =
0 διαφορετικά .
6
Τότε |S(⋆, bj )| είναι το άθροισμα των στοιχείων∑της στήλης j, ενώ∑|S(ai , ⋆)| το άθροισμα της
γραμμής i. Επομένως, και τα δυο αθροίσματα j=1 |S(⋆, bj )| και m
n
i=1 |S(ai , ⋆)| υπολογίζουν
το άθροισμα όλων των στοιχείων του πίνακα M, και το αποτέλεσμα συνεπάγεται αμέσως.

Μπορεί η γλώσσα της Θεωρίας Συνόλων που χρησιμοποιούμε εδώ να φαίνεται λίγο δυσνό-
ητη, αλλά για να καταλάβουμε τι γίνεται ας δούμε πως χρησιμοποιείται το Λήμμα Fubini
στο Παράδειγμα 1. Ποιά σύνολα αντιπροσωπεύουν τα A, B και S; Πολύ απλά μπορούμε να
ορίσουμε ως A το σύνολο των κορυφών των μοναδιαίων τετραγώνων, και ως B το σύνολο
των πλευρών. Τότε το υποσύνολο S αποτελεί το σύνολο των ζευγών (v, s) όπου v είναι μια
κόκκινη κορυφή που βρίσκεται στην πλευρά s.

Παράδειγμα 2. Έστω n ένας θετικός ακέραιος και έστω (a1 , a2 , · · · , an ) μια αναδιάταξη
των αριθμών (1, 2, · · · , n). Για 1 ≤ k ≤ n, ορίζουμε

Fk = {ai | ai < ak , i > k} και


Gk = {ai | ai > ak , i < k}.
∑ ∑
Να αποδειχθεί οτι nk=1 |Fk | = nk=1 |Gk |.

Απόδειξη. Ορίζουμε A = B = {a1 , a2 , · · · , an }, και

S = {(ai , aj ) | ai < aj , i > j}.

Τότε έχουμε S(⋆, aj ) = Fj και S(ai , ⋆) = Gi , και το ζητούμενο ακολουθεί άμεσα απο το
Θεώρημα 1.
Τις πιο πολλές φορες είναι αρκετά δύσκολο να αντιληφθούμε ποιά αντικείμενα πρέπει να
υπολογιστούν με διαφορετικούς τρόπους ούτως ώστε να μπορέσουμε να χρησιμοποιήσουμε
το Λήμμα Fubini. Δεν υπάρχουν σταθεροί κανόνες, όπως είπαμε στην εισαγωγή, αλλά πολλές
φορές τα προβλήματα τα ίδια μας δίνουν πολύτιμες πληροφορίες.

Παράδειγμα 3. Σε ένα τμήμα Συνδιαστικής υπάρχουν 12 μαθητές . Στην αρχή κάθε εβδο-
μάδας, ο καθηγητής ορίζει μια εργασία στους μαθητές, οι οποίοι χωρίζονται σε 6 ομάδες,
χωρίς κανένας μαθητής να δουλεύει μόνος του. Κάθε ομάδα δουλεύει αυτόνομα και πα-
ραδίδει την εργασία στο τέλος της εβδομάδας. Κάθε εβδομάδα οι μαθητές μπορούν να
δημιουργήσουν τις ομάδες όπως θέλουν. Να αποδειχθεί οτι, άσχετα με τον τρόπο που επι-
λέγουν οι μαθητές τις ομάδες τους, μπορούμε πάντα να επιλέξουμε δύο μαθητές έτσι ώστε
να υπάρχουν πάντα τουλάχιστον πέντε άλλοι μαθητές που έχουν δουλέψει και με τους δύο
ή δεν έχουν δουλέψει με κανένα απο τους δύο.

Απόδειξη. Ορίζουμε A = {s1 , s2 , · · · , sn }, το σύνολο όλων των μαθητών και B = {(si(, sj)) | 1 ≤
i < j ≤ 12} ,το σύνολο όλων των ζευγαριών των μαθητών. Έχουμε αμέσως |B| = 12 2
= 66.
Θα λέμε ότι ο μαθητής si και το ζεύγος (sj , sk ) είναι ενωμένοι εάν οι μαθητές si , sj , sk είναι
διαφορετικοί και ο μαθητής si έχει δουλέψει με ένα ακριβώς απο τους μαθητές sj και sk .
Έστω

S = {[si , (sj , sk )] | si και (sj , sk ) είναι ενωμένοι}.

Θα αποδείξουμε το ζητούμενο έμμεσα, χρησιμοποιώντας την εις άτοπο απαγωγή. Υποθέ-


τουμε λοιπόν πως το ζητούμενο του προβλήματος δεν ισχύει και άρα κάποια στιγμή το
ζευγάρι (sj , sk ) είναι ενωμένο με τουλάχιστον έξι μαθητές, ή διαφορετικά |S(⋆, (sj , sk ))| ≥ 6.
Συνεπώς έχουμε

|S| = |S(⋆, (sj , sk ))| ≥ 6|B| = 396.
(sj ,sk )∈B

7
Όμως, εάν ο μαθητής si έχει δουλέψει με d άτομα, τότε ο si είναι ενωμένος με d(11 − d)
ζευγάρια μαθητών (αφού έχουμε d τρόπους να επιλέξουμε ένα απο τους προηγούμενους
συνεργάτες και 11 − d τρόπους να επιλέξουμε ένα μαθητή που δεν έχει δουλέψει μαζί του).
Για τους ακέραιους αριθμούς 0 ≤ d ≤ 11, η μέγιστη τιμή της παράστασης d(11 − d) είναι
30, όταν δηλαδή d = 5 ή 6. Άρα έχουμε |S(si , ⋆)| ≤ 30 και συνεπώς

|S| = |S(si , ⋆)| ≤ 30|A| = 360.
si ∈A

Έχουμε λοιπόν 396 ≤ |S| ≤ 360, που είναι άτοπο. Άρα η υπόθεση μας ήταν λανθασμένη,
γεγονός που αποδυκνείει το ζητούμενο.
Το πρόβλημα αυτό αποτελεί ειδική περίπτωση του πάρακάτω.
Παράδειγμα 4. Έστω X ένα πεπερασμένο σύνολο με |X| = n. Έστω A1 , A2 , · · · , Am υποσύ-
√ έτσι ώστε |Ai ∩ Aj | ≤ 1 για i ̸= j. Υπάρχει τότε
νολα του X με τρία στοιχεία το καθένα και
ένα υποσύνολο A του X με τουλάχιστον ⌊ 2n⌋ στοιχεία και το οποίο δεν συμπεριλαμβάνει
κανένα απο τα Ai .
Απόδειξη. Έστω A ένα υποσύνολο του X το οποίο δεν συμπεριλαμβάνει κανένα απο τα
Ai , και το οποίο έχει το μέγιστο δυνατό αριθμό στοιχείων. Έστω k = |A|. Η δυσκολία στην
άσκηση αυτή είναι ο τρόπος που θα χρησιμοποιήσουμε την μεγιστότητα του A. Θα μετρή-
σουμε τον αριθμό των στοιχείων του συνόλου X \ A με διαφορετικούς τρόπους. Προφανώς
ο αριθμός αυτός ισούται με n − k.
Έστω x ένα στοιχείο του X που δεν ανήκει στο A. Απο τη μεγιστότητα του A προκύ-
πτει οτι το σύνολο A ∪ {x} δεν ικανοποιεί τις συνθήκες του προβλήματος. Υπάρχει, άρα,
i(x) ∈ {1, 2, · · · , m} έτσι ώστε Ai(x) ⊆ A ∪ {x}. Επομένως x ∈ Ai(x) και το σύνολο Ai(x) \ {x}
αποτελεί υποσύνολο του συνόλου A. Άρα, το σύνολο Lx = A ∩ Ai(x) αποτελείται απο 2
στοιχεία. Τώρα, επειδή |Ai ∩ Aj | ≤ 1 για i ̸= j, τα σύνολα Lx είναι όλα διαφορετικά.
Επομένως έχουμε ορίσει μια ένα-προς-ένα συνάρτηση f (x) = Lx απο το σύνολο X \ A στο
σύνολο των υποσυνόλων του A με δύο στοιχεία. Συμπεραίνουμε έτσι πως
( )
k k2 − k
n−k ≤ = ,
2 2

( k + k ≥ 2n και επομένως k 2≥ ⌊2n⌋.


2
ή √ √ )
Παρατηρούμε πως (⌊2n⌋ − 1) + (⌊2n⌋ − 1) ≤ 2n( 2n − 1) < 2n .
Για να παρουσιάσουμε στον αναγνώστη την πραγματική δύναμη του Λήμματος Fubini,
παραθέτουμε παρακάτω την απόδειξη των Lubell, Meshalkin και Yamamoto σε ένα θεώρημα
του E. Sperner.
Θεώρημα 2 (Sperner). Έστω S ένα σύνολο με |S| = n. Υποθέτουμε οτι τα S1 , S2 , · · · , Sm
είναι υποσύνολα του S με Si ⊈ Sj για i ̸= j. Τότε
( )
n
m≤ .
⌊ n2 ⌋

Απόδειξη. Μια διατεταγμένη n-αδα συνόλων (T1 , T2 , · · · , Tn ) ονομάζεται μέγιστη αλυσίδα


του S εάν

∅ ̸= T1 ⊂ T2 ⊂ T3 ⊂ · · · ⊂ Tn = S.

Στην περίπτωση αυτή, ονομάζουμε κάθε σύνολο Ti κρίκο της αλυσίδας. Επειδή το σύνολο
Ti αποτελεί γνήσιο υποσύνολο του συνόλου Ti+1 , το σύνολο Ti αποτελείται απο i στοιχεία.
Υπάρχουν n τρόποι ορισμού του συνόλου T1 . Αφού ορίσουμε το T1 , υπάρχουν n − 1 τρόποι
για να ορίσουμε το σύνολο T2 και ούτω καθεξής. Υπάρχουν επομένως n! μέγιστες αλυσίδες

8
του S, αν |S| = n.
Για ένα σύνολο Si , θα μελετήσουμε τις μέγιστες αλυσίδες στο σύνολο S οι οποίες συμπε-
ριλαμβάνουν το σύνολο Si ως κρίκο. Έστω (T1 , T2 , · · · , Tn ) μια τέτοια αλυσίδα. Αν |Si | = ki ,
τότε Tki = Si . Παρατηρούμε επίσης οτι η ki -αδα (T1 , T2 , · · · , Tki ) είναι μια μέγιστη αλυσίδα
του Si και άρα υπάρχουν ki ! τέτοιες αλυσίδες. Επειδή υπάρχουν n − ki στοιχεία στο σύ-
νολο S \ Si , υπάρχουν n − ki τρόποι για να ορίσουμε το Tki +1 και ούτω καθεξής. Υπάρχουν,
άρα, (n − ki )! τρόποι για να επιλέξουμε τα σύνολα Tki +1 , · · · , Tn . Κατα συνέπεια, υπάρχουν
ki !(n − ki )! μέγιστες αλυσίδες του S οι οποίες συμπεριλαμβάνουν το Si ως κρίκο.
Έστω A το σύνολο όλων των μέγιστων αλυσίδων και B = {S1 , S2 , · · · , Sm }. Θέτουμε
S = {(Ci , Sj ) | Ci ∈ A , Sj ∈ B , Sj κρίκος του Ci }.
Όπως και στην απόδειξη του Θεωρήματος 1, ορίζουμε ένα πίνακα M = (xi,j ) με n! γραμμές
και m στήλες, έτσι ώστε
{
1 αν (Ci , Sj ) ∈ S,
xi,j =
0 διαφορετικά .
Επομένως υπάρχουν kj !(n − kj )! μονάδες στη στήλη j, και

m
|S| = kj !(n − kj )!.
j=1

Όμως, επειδή το σύνολο Si δεν είναι υποσύνολο του Sj όταν i ̸= j, τα σύνολα Si και Sj δεν
μπορούν να είναι κρίκοι της ίδιας μέγιστης αλυσίδας του S. Επομένως το άθροισμα των
στοιχείων κάθε γραμμής του M είναι το πολύ 1. Άρα το άθροισμα όλων των αθροισμάτων
των γραμμών είναι το πολύ n!, ή |S| ≤ n!. Συνεπάγεται οτι

m
kj !(n − kj )! ≤ n!,
j=1

ή διαφορετικά

m
1
( n ) ≤ 1.
j=1 kj

Θα χρειαστούμε την παρακάτω εύκολη ανισότητα που αφορά τους διωνυμικούς συντελε-
στές.
Λήμμα. Έστω n θετικός ακέραιος αριθμός. Ισχύει η ανισότητα
( ) ( ) ( ) ( ) ( )
n n n n n
< < < ··· < = .
0 1 2 ⌈ n−1
2
⌉ ⌊ n
2

Απόδειξη Λήμματος. Αν ο αριθμός n είναι ζυγός, τότε ⌈ n−1 2
⌉ = n2 . Αν ο αριθμός n είναι
περιττός, τότε ⌈ 2 ⌉ = 2 . Για 0 ≤ k ≤ ⌈ 2 ⌉ − 1, έχουμε 2k ≤ n − 2, ή k + 1 ≤ n − k − 1.
n−1 n−1 n−1

Επομένως
(n)
(k + 1)!(n − k − 1)! k+1
( nk ) = = < 1.
k+1
k!(n − k)! n − k
( ) ( n )
Χρησιμοποιώντας τη γνωστή σχέση nk = n−k για n ≥ k, έχουμε
( ) ( ) ( ) ( ) ( ) ( ) ( )
n n n n n n n
< < < ··· < = > > ··· >
0 1 2 m m+1 m+2 n
εαν ο αριθμός n είναι περιττός με n = 2m + 1, και
( ) ( ) ( ) ( ) ( ) ( ) ( )
n n n n n n n
< < < ··· < > > > ··· >
0 1 2 m m+1 m+2 n
εαν ο αριθμός n είναι ζυγός με n = 2m, που αποδυκνείει τη ζητούμενη ανισότητα.

9
(m) ( )
Χρησιμοποιώντας την ανισότητα που μόλις αποδείξαμε, έχουμε ότι kj
≤ n
⌊n ⌋
, και κατα
2
συνέπεια,

1 ∑
m
1
m( n
) ≤ ( n ) ≤ 1.
⌊n
2
⌋ j=1 kj
( n
)
όπως θέλαμε. (Παρατήρηση: Έχουμε m = ⌊n ⌋
όταν το σύνολο B αποτελείται απο όλα τα
2
υποσύνολα του S με ⌊ n2 ⌋ στοιχεία).
Στη συνέχεια παραθέτουμε κάποια παραδείγματα γεωμετρικής φύσεως.
Παράδειγμα 5 (IMO 2000, Shortlist). Έστω n ≥ 4 θετικός ακέραιος. Έστω S = {P1 , P2 , · · · , Pn }
ένα σύνολο n σημείων στο επίπεδο τα οποία ανα τρία δεν είναι συνευθειακά και ανα τέσ-
σερα δεν είναι συγκυκλικά. Θα καλούμε κύκλο Pi Pj Pk τον περιγεγραμμένο κύκλο του τρι-
γώνου Pi Pj Pk . Έστω at , 1 ≤ t ≤ n, ο αριθμός των κύκλων Pi Pj Pk που περιέχουν το σημείο
Pt στο εσωτερικό τους, και έστω

m(S) = a1 + a2 + · · · + an .

Να αποδειχθεί οτι υπάρχει θετικός αριθμός f (n), εξαρτώμενος μόνο απο το n, έτσι ώστε τα
σημεία του S είναι κορυφές κυρτού πολυγώνου αν και μόνο αν m(S) = f (n).
Απόδειξη. Αν το ABCD είναι ένα κυρτό, μη εγγράψιμο τετράπλευρο, τότε Â + Ĉ ̸= B̂ + Ĉ.
Επειδή το άθροισμα των τεσσάρων πλευρών ισούται με 360◦ , μπορούμε να υποθέσουμε,
χωρίς βλάβη της γενικότητας, οτι Â + Ĉ > 180◦ > B̂ + D̂. Τότε τα σημεία B και D βρίσκονται
εκτός των κύκλων ACD και ABC, αντίστοιχα, και τα σημεία A και C βρίσκονται εντός
των κύκλων BCD και BAD αντίστοιχα. Αν το ABCD δεν είναι κυρτό, υποθέτουμε, χωρίς
βλάβη της γενικότητας, οτι Â > 180◦ . Τότε το σημείο A βρίσκεται εντός του κύκλου BCD
και τα B, C και D βρίσκονται εκτός των κύκλων ACD, ABD και ABC αντίστοιχα. Για κάθε
σύνολο {Pi , Pj , Pk , Pl }, υπάρχουν τέσσερα πιθανά ζεύγη τα οποία αυξάνουν την ποσότητα
m(S) κατά μία μονάδα και τα οποία είναι τα (Pi , Pj Pk Pl ), (Pj , PK PL PI ), (Pk , Pl Pi Pj ) και
(Pl , Pi Pj Pk ). Αν τα τέσσερα σημεία αποτελούν κυρτό πολύγωνο, τότε ακριβώς δύο απο τα
πιο πάνω ζεύγη αυξάνουν την ποσότητα m(S) κατά μία μονάδα. Αν όχι, μόνο ένα απο τα
ζεύγη αυξάνει το m(S) κατά μία μονάδα.
Έστω a(S) και b(S) οι αριθμοί των κυρτών και μη κυρτών τετραπλεύρων που ορίζονται απο
τα σημεία του S, αντίστοιχα. Έχουμε
( )
n
a(S) + b(S) = και 2a(S) + b(S) = m(S).
4
( ) ( )
Συνεπώς, m(S) = n4 +a(S). Εικάζουμε οτι ο αριθμός f (n) = 2 n4 έχει τη ζητούμενη ιδιότητα.
Όντως, αν τα σημεία του S ορίζουν ένα κυρτό πολύγωνο, (n)τότε κάθε πολύγωνο που ορίζεται
απο αυτά τα σημεία είναι (επίσης ) κυρτό και άρα a(S) = 4
και m(S) = f (n). Αντίστροφα, αν
n
m(S) = f (n), τότε a(S) = 4 και άρα κάθε τετράπλευρο που ορίζεται απο τέσσερα σημεία
του S είναι κυρτό. Ακολουθεί ευκολα οτι τα σημεία του S ορίζουν ένα κυρτό πολύγωνο.
Παράδειγμα 6. Έστω n και k θετικοί ακέραιοι που ικανοποιούν την ακόλουθη ιδιότητα:
Υπάρχει σύνολο T που αποτελείται απο n σημεία στο επίπεδο έτσι ώστε
(i) τα σημεία αυτά ανα τρία δεν είναι συνευθειακά και
(ii) για κάθε σημείο P του T , υπάρχουν τουλάχιστον k σημεία του T τα οποία έχουν την
ίδια απόσταση απο το σημείο P .
Να αποδειχθεί οτι
1 √
k < + 2n.
2
10
Απόδειξη. Έστω A = T = {P1 , P2 , ·(· ·) , Pn }, και έστω B = {ℓi,j | 1 ≤ i < j ≤ n}, όπου ℓi,j η
μεσοκάθετος του Pi Pj . Τότε |B| = n2 . Έστω

S = {(Pi , ℓj,k ) | Pi ανήκει στο ℓj,k }.

Αφού ανα τρία τα σημεία δεν είναι συνευθειακά, έχουμε |S(⋆, ℓj,k )| ≤ 2. Επομένως,

S = |S(⋆, ℓj,k )| ≤ 2|B| = n2 − n.
ℓj,k ∈B

Όμως, αφού το σημείο Pi βρίσκεται σε ίση απόσταση απο τουλάχιστον k άλλα σημεία, το
σημειο Pi πρέπει
(k) να ανήκει στην μεσοκάθετο οποιονδήποτε δύο απο αυτά τα k σημεία. Άρα
|S(Pi , ⋆)| ≥ 2 και επομένως
∑ ( )
k n(k 2 − k)
S = |S(Pi , ⋆)| ≥ n = .
P ∈A
2 2
i

Συνδιάζοντας τα πιο πάνω έχουμε

k 2 − k − 2(n − 1) ≤ 0.

Λύνοντας την πιο πάνω ανισότητα παίρνουμε


√ √
1 7 1 7
− 2n − ≤ k ≤ + 2n − ,
2 4 2 4
απο όπου το ζητούμενο ακολουθεί άμεσα.

11
3 Συνδιαστικό Nullstellensatz
Το 2007, στη Διεθνή Μαθηματική Ολυμπιάδα στο Βιετνάμ, τέθηκε σαν τελευταία άσκηση
ένα πολύ δύσκολο αλλά ταυτόχρονα πολύ όμορφο πρόβλημα Συνδιαστικής, το οποίο λύθηκε
μόνο απο πέντε διαγωνιζόμενους. Μου πήρε προσωπικά αρκετό καιρό για να καταλάβω πως
λυνόταν. Επειδή η τακτική επίλυσης του προβλήματος αυτού, το Συνδιαστικό Nullstellensatz,
που αποτελεί μία πολύ χρήσιμη τεχνική της Αλγεβρικής Γεωμετρίας, προσαρμόζεται αρ-
κετά εύκολα σε Συνδιαστική γλώσσα, θεώρησα πως αξίζει τον κόπο να προσπαθήσω να
την εξηγήσω λεπτομερώς. Το θεώρημα αυτό έχει πολλές εφαρμογές σε τομείς όπως η Θε-
ωρία Αριθμών, η Θεωρία Γραφημάτων και η Συναρτησιακή Ανάλυση και προβλήματα που
μπορούν να λυθούν χρησιμοποιώντας το έχουν παρουσιαστεί αρκετές φορές σε Ολυμπιάδες.

3.1 Ομάδες, Δακτύλιοι και Σώματα


ΣΗΜΕΙΩΣΗ. Η ενότητα αυτή συμπεριλαμβάνεται για λόγους πληρότητας του κειμέ-
νου και περιγράφει τις μαθηματικές δομές που εμφανίζονται στη γενική διατύπωση του
Nullstellensatz. Αν και το υλικό αυτό αποτελεί τα θεμέλια των Μαθηματικών και άρα θα
ήταν χρήσιμο, έμμεσα, για τον καθένα, μπορεί να παραλειφθεί. Στην περίπτωση αυτή
ο αναγνώστης μπορεί να συνεχίσει στην επόμενη ενότητα, αντικαθιστώντας το σύμβολο
F με το σύμβολο R των πραγματικών αριθμών.
Ξεκινούμε με μια πολύ θεμελιώδη έννοια.
Ορισμός. Έστω S ένα μη κενό σύνολο. Μια δυαδική πράξη στο S είναι ένας κανόνας με
τον οποίο συνδιάζουμε δύο στοιχεία του S για να δημιουργήσουμε ένα άλλο στοιχείο του
S: Αν a, b ∈ S, γράφουμε a ⋆ b για το στοιχείο που δημιουργείται απο το συνδιασμό των a
και b.
Παραδείγματα.
(i) Αν S = N τότε ο κανόνας a ⋆ b = a + b ορίζει μία δυαδική πράξη στο S.

(ii) Αν S = N τότε ο κανόνας a ⋆ b = ab ορίζει μία δυαδική πράξη στο S.

(iii) Αν S = N τότε ο κανόνας a ⋆ b = a − b δεν ορίζει δυαδική πράξη στο S, αφού εάν a ≤ b
τότε a − b ∈
/ S. Όμως, αν αντικαταστήσουμε το σύνολο N με το σύνολο Z τότε ο πιο
πάνω κανόνας ορίζει δυαδική πράξη.

(iv) Αν S = N ή Z τότε ο κανόνας a ⋆ b = ab δεν ορίζει δυαδική πράξη στο S, αφού εάν ο
αριθμός b δεν διαιρεί τον αριθμό a τότε ab ∈
/ S. Αν θέσουμε S = Q τότε πάλι δεν έχουμε
δυαδική πράξη, αφού εάν b = 0 τότε το κλάσμα ab δεν ορίζεται. Εαν όμως θέσουμε
S = Q∗ := Q \ {0}, τότε ο κανόνας a ⋆ b = ab ορίζει δυαδική πράξη στο Q∗ .
Τα σημαντικά θέματα που πρέπει να προσέξουμε εδώ είναι ότι: (i) το στοιχείο a ⋆ b πρέπει
να ορίζεται για κάθε a, b ∈ S και (ii) το στοιχείο a ⋆ b πρέπει να είναι και εκείνο στοιχείο
του S.
Ορισμός. Η δυαδική πράξη ⋆ καλείται αντιμεταθετική εάν (∀a, b ∈ S)(a ⋆ b = b ⋆ a).
Παραδείγματα.
(i) Η πρόσθεση στο σύνολο Z είναι αντιμεταθετική, αφού a + b = b + a για κάθε a, b ∈ Z.

(ii) Ο πολλαπλασιασμός στο σύνολο Z είναι αντιμεταθετικός, αφού ab = ba για κάθε


a, b ∈ Z.

(iii) Η αφαίρεση στο σύνολο Z δεν είναι αντιμεταθετική, αφού, για παράδειγμα, 3 − 1 = 2
ενώ 1 − 3 = −2.

12
Ορισμός. Η δυαδική πράξη ⋆ καλείται προσεταιριστική εάν (∀a, b, c ∈ S)((a⋆b)⋆c = a⋆(b⋆c)).

Παραδείγματα.

(i) Η πρόσθεση στο σύνολο Z είναι προσεταιριστική, αφού (a + b) + c = a + (b + c) για


κάθε a, b, c ∈ Z.

(ii) Ο πολλαπλασιασμός στο σύνολο Z είναι αντιμεταθετικός, αφού (ab)c = a(bc) για κάθε
a, b, c ∈ Z.

(iii) Η αφαίρεση στο σύνολο Z δεν είναι αντιμεταθετική, αφού, για παράδειγμα, (3−2)−1 =
0 ενώ 3 − (2 − 1) = 2.

Η επόμενη ιδιότητα αφορά ένα στοιχείο που διαθέτει ένα συγκεκριμένο χαρακτηριστικό.

Ορισμός. Ένα στοιχείο e του συνόλου S καλείται ουδέτερο στοιχείο για τη δυαδική πράξη
⋆ εάν (∀a ∈ S)(e ⋆ a = a = a ⋆ e). Λέμε ότι η δυαδική πράξη ⋆ έχει ουδέτερο στοιχείο, εάν
ένα τέτοιο στοιχείο e υπάρχει.

Παραδείγματα.

(i) Η πρόσθεση στο σύνολο Z εχει σαν ουδέτερο στοιχείο τον αριθμό 0, αφού 0 + a = a =
a + 0 για κάθε a ∈ Z.

(ii) Ο πολλαπλασιασμός στο σύνολο Z έχει σαν ουδέτερο στοιχείο τον αριθμό 1, αφού
1.a = a = a.1 για κάθε a ∈ Z.

(iii) Η αφαίρεση στο σύνολο Z δεν έχει ουδέτερο στοιχείο, αφού απο την εξίσωση e − a = a
θα παίρναμε e = 2a και κανένα στοιχείο e δεν επαληθεύει την ισότητα αυτή για όλα
τα στοιχεία a ταυτόχρονα.

Ο πιο πάνω ορισμός δεν απορρίπτει την πιθανότητα να υπάρχουν περισσότερα απο ένα
ουδέτερα στοιχεία για μια δυαδική πράξη. Μπορούμε εύκολα να αποδείξουμε όμως ότι
κάτι τέτοιο δεν μπορεί να ισχύει.

Πρόταση. Εάν μια δυαδική πράξη έχει ουδέτερο στοιχείο, τότε αυτό είναι μοναδικό.

Απόδειξη. Εάν ⋆ είναι μια δυαδική πράξη και e1 , e2 ουδέτερα στοιχεία της, τότε e1 ⋆ e2 = e1 ,
αφού το e2 είναι ουδέτερο στοιχείο, καθώς επίσης e1 ⋆ e2 = e2 , αφού και το e1 είναι ουδέτερο
στοιχείο. Άρα e1 = e2 .

Ορισμός. Έστω ότι μια δυαδική πράξη ⋆ στο σύνολο S έχει ουδέτερο στοιχείο e, και έστω
a ∈ S. Το στοιχείο b καλείται αντίστροφος του a εάν a ⋆ b = e = b ⋆ a. Αν ένα τέτοιο στοιχείο
υπάρχει, λέμε οτι το στοιχείο a έχει αντίστροφο.

Παραδείγματα.

(i) Για την πρόσθεση στο σύνολο Z, ο αντίστροφος ενός στοιχείου a είναι το −a, αφού
a + (−a) = 0 = (−a) + a.

(ii) Για τον πολλαπλασιασμό στο σύνολο Z, τα μόνα στοιχεία που έχουν αντίστροφο είναι
οι αριθμοί 1 και −1 (και σε κάθε περίπτωση ο αντίστροφος είναι το ίδιο το στοιχείο) .

(iii) Για τον πολλαπλασιασμό στο σύνολο Q, κάθε μη μηδενικό στοιχείο a έχει σαν αντί-
στροφο το a1 . Ο αριθμός 0 όμως δεν έχει αντίστροφο.

13
Παρακάτω θα δούμε κάποια πιο “παράξενα” παραδείγματα δυαδικών πράξεων.
Παραδείγματα.
(i) Αν S = Mn (R), το σύνολο των n × n πινάκων των οποίων οι γραμμές και στήλες
αποτελούνται απο στοιχεία του R, τότε ο κανόνας A ⋆ B = AB, με AB τον πίνακα
που προκύπτει απο τον πολλαπλασιασμό των πινάκων A και B, ορίζει μια δυαδική
πράξη στο S. Η πράξη αυτή δεν είναι αντιμεταθετική, είναι όμως προσαιτεριστική.
Ο μοναδιαίος πίνακας In×n αποτελεί ουδέτερο στοιχείο της πράξης αυτής, όμως δεν
υπάρχει αντίστροφο στοιχείο για κάθε A ∈ Mn (R), αφού δεν είναι όλοι οι πίνακες
αντιστρέψιμοι.

(ii) Η δυαδική πράξη που ορίζεται στο Z ως a⋆b = a+b+1 είναι προφανώς αντιμεταθετική.
Είναι επίσης προσεταιριστική, αφού (a ⋆ b) ⋆ c = (a + b + 1) ⋆ c = a + b + 1 + c + 1 =
a ⋆ (b + c + 1) = a ⋆ (b ⋆ c) για κάθε a, b, c ∈ Z. Ο αριθμός −1 είναι ουδέτερο στοιχείο,
αφού (−1) ⋆ a = −1 + a + 1 = a για κάθε a ∈ Z (Παρατηρούμε εδώ οτι δε χρειάζεται
να ελέγξουμε τη δεύτερη ισότητα για το ουδέτερο στοιχείο, αφού η δυαδική πράξη
είναι αντιμεταθετική). Επίσης, κάθε στοιχείο a ∈ Z έχει σάν αντίστροφο το −a − 2,
αφού a ⋆ (−a − 2) = a − a − 2 + 1 = −1.
Είμαστε έτοιμοι τώρα να ορίσουμε την ομάδα, μια πολύ σημαντική μαθηματική δομή, η
οποία αναγνωρίζεται ως μία από τις πλέον θεμελιώδεις έννοιες στην επιστήμη των μαθη-
ματικών και τις εφαρμογές τους.
Ορισμός. Καλούμε ομάδα ένα σύνολο G, εφοδιασμένο με μια δυαδική πράξη ⋆, για την
οποία ισχύουν οι παρακάτω ιδιότητες:
(i) η δυαδική πράξη ⋆ είναι προσεταιριστική,

(ii) η δυαδική πράξη ⋆ έχει ουδέτερο στοιχείο, και

(iii) κάθε στοιχείο του συνόλου G έχει αντίστροφο.


Σε αυτή τη περίπτωση, λέμε οτι το σύνολο G είναι ομάδα ως προς τη δυαδική πράξη ⋆, και
τη συμβολίζουμε με το ζεύγος (G, ⋆).
ΣΗΜΕΙΩΣΗ. Όταν η δυαδική πράξη συμβολίζεται με + λέμε οτι είναι πρόσθεση και
η ομάδα (G, +) ονομάζεται προσθετική ομάδα. Σε αυτή την περίπτωση, το ουδέτερο
στοιχείο συμβολίζεται με 0 και ο αντίστροφος ενός στοιχείου a ∈ G με −a. Όταν η δυαδική
πράξη συμβολίζεται με × (ή .) λέμε οτι είναι πολλαπλασιασμός και η ομάδα (G, ×)
ονομάζεται πολλαπλασιαστική ομάδα. Σε αυτή την περίπτωση, το ουδέτερο στοιχείο
συμβολίζεται με 1 και ο αντίστροφος ενός στοιχείου a ∈ G με a−1 .
Έχουμε ήδη δεί αρκετά παραδείγματα ομάδων στα προηγούμενα παραδείγματα, αλλά
παραθέτουμε μια λίστα με τις πιο σημαντικές παρακάτω.
Παραδείγματα.
(i) Τα σύνολα Z, Q, R, C αποτελούν ομάδες ως προς την πρόσθεση.

(ii) Τα σύνολα Q∗ , R∗ , C∗ αποτελούν ομάδες ως προς τον πολλαπλασιασμό, όπως επίσης


και το σύνολο {z ∈ C : |z| = 1}.

(iii) Το σύνολο των αμφιέσεων (= ένα-προς-ένα και επί συναρτήσεις) A → A, όπου A ένα
μη κενό σύνολο, αποτελεί ομάδα ως προς τη σύνθεση συναρτήσεων.

(iv) Το σύνολο των αντιστρέψιμων n × n πινάκων των οποίων οι γραμμές και στήλες
αποτελούνται απο στοιχεία του R αποτελεί ομάδα ως προς τον πολλαπλασιασμό
πινάκων.

14
Παρατηρείστε οτι στον ορισμό της ομάδας, η δυαδική πράξη δεν είναι αναγκαστικά αντι-
μεταθετική.
Ορισμός. Μια ομάδα της οποίας η δυαδική σχέση είναι αντιμεταθετική καλείται αβελιανή.
Ορισμός. Ένα σύνολο R, εφοδιασμένο με δύο δυαδικές πράξεις + και × θα λέγεται δα-
κτύλιος, όταν ισχύουν τα ακόλουθα αξιώματα:
(i) Το (R, +) αποτελεί αβελιανή ομάδα (δηλαδή το σύνολο R αποτελεί αβελιανή ομάδα
ως προς την δυαδική πράξη +),

(ii) Για κάθε a, b, c ∈ R ισχύει a × (b × c) = (a × b) × c (προσεταιρισμός),

(iii) Για κάθε a, b, c ∈ R ισχύουν τα πιο κάτω (επιμερισμός):

(a) a × (b + c) = a × b + a × c και
(b) (a + b) × c = a × c + b × c.

Σε αυτή την περίπτωση συμβολίζουμε τον δακτύλιο με την τριάδα (R, +, ×).
Παρατηρήσεις.
(i) Αν υπάρχει στοιχείο 1 ∈ R τέτοιο ώστε 1 × a = a = a × 1 για κάθε a ∈ R, τότε αυτό
θα λέγεται μοναδιαίο στοιχείο του δακτύλιου και ο δακτύλιος λέγεται δακτύλιος με
μοναδιαίο.

(ii) Αν ισχύει a × b = b × a για κάθε a, b ∈ R τότε ο δακτύλιος λέγεται αντιμεταθετικός.

(iii) Αν, τέλος, ένας δακτύλιος R έχει μοναδιαίο και ισχύει η αντιμεταθετικότητα του πολ-
λαπλασιασμού, θα λέγεται αντιμεταθετικός δακτύλιος με μοναδιαίο.
Παραδείγματα.
(i) Τα σύνολα Z, Q, R, C εφοδιασμένα με τις συνήθεις δυαδικές πράξεις της πρόσθεσης
και του πολλαπλασιασμού αποτελούν δακτύλιους.

(ii) Έστω R το σύνολο των ακεραίων Z, εφοδιασμένο με τις δυαδικές πράξεις +R και ×R
οι οποίες ορίζονται ως εξής:

a +R b = a + b + 1 και a ×R b = ab + a + b για κάθε a, b ∈ R.

Τότε η τριάδα (R, +R , ×R ) αποτελεί δακτύλιο.

(iii) Έστω n ∈ {2, 3, 4, · · · }. Υπενθυμίζουμε οτι δύο ακέραιοι αριθμοί είναι ισότιμοι modulo
n εαν ο αριθμός n διαιρεί τον k − m. Σε αυτή τη περίπτωση, γράφουμε k ≡ m mod n.
Γράφουμε m̂ για την κλάση ισοτιμίας του m modulo n :

m̂ = {k | k ≡ m mod n},

και γράφουμε Zn για το σύνολο των κλάσεων ισοτιμίας modulo n :

Zn = {0̂, 1̂, · · · , n[
− 1},

όπου οι κλάσεις 0̂, 1̂, · · · , n[


− 1 δεν έχουν κοινά στοιχεία μεταξύ τους και η ένωση τους
ισούται με ολόκληρο το Z (δεν είναι δύσκολο να αποδειχθεί). Μπορούμε να ορίσουμε
δυαδικές πράξεις πρόσθεσης και πολλαπλασιασμού στο Zn , όπως παρακάτω:

m̂ + n̂ = k\
+m και c
k̂ × m̂ = km.

Τότε η τριάδα (Zn , +, ×) αποτελεί δακτύλιο.

15
(iii) Έστω R ένας δακτύλιος. Ο δακτύλιος πολυωνύμων επί του R, μιας μεταβλητής X,
ορίζεται σαν το σύνολο R[X] των εκφράσεων

a0 + a1 X + a2 X 2 + · · · + am X m ,

όπου m ∈ N0 και ak ∈ R για κάθε k ∈ N0 . Οι δυαδικές πράξεις της πρόσθεσης και του
πολλαπλασιασμού ορίζονται όπως παρακάτω, όπου f = f (X) = a0 + a1 X + a2 X 2 +
· · · + am X m και g = g(X) = b0 + b1 X + b2 X 2 + · · · + bn X n :

f + g = (a0 + b0 ) + (a1 + b1 )X + · · · + (ak + bk )X k + · · ·


· · · + (amax{m,n} + bmax{m,n} )X max{m,n}

και

f × g = (a0 b0 ) + (a0 b1 + a1 b0 )X + (a0 b2 + a1 b1 + a2 b0 )X 2 + · · ·


· · · + (a0 bk + a1 bk−1 + · · · + aj bk−j + · · · + ak b0 )X k + · · ·
· · · + am bn X m+n .

(Μπορούμε να ορίσουμε, παρόμοια, το δακτύλιο πολυωνύμων R[X1 , X2, · · · , Xn ] επι ενός


δακτυλίου R σε n μεταβλητές. Τα στοιχεία του δακτυλίου σε αυτή την περίπτωση έχουν
την μορφή

f (X1 , X2 , · · · , Xn ) = c α X α1 X α2 · · · X αn
α

όπου α = (α1 , α2 , · · · , αn ) με αi ∈ N0 και cα ∈ R.)

Ορισμός. Ένα σύνολο F, εφοδιασμένο με δύο δυαδικές πράξεις + και ×, ονομάζεται σώμα,
εαν ισχύουν τα παρακάτω.

(i) Η τριάδα (F, +, ×) είναι αντιμεταθετικός δακτύλιος με μοναδιαίο στοιχείο.

(ii) Το ζεύγος (F × , ×), όπου F × = F \ {0} και 0 το ουδέτερο στοιχείο της πρόσθεσης, είναι
μια ομάδα.

Συνοπτικά μπορούμε να πούμε οτι το σώμα είναι ένας αντιμεταθετικός δακτύλιος με μο-
ναδιαίο στοιχείο του οποίου όλα τα μη μηδενικά στοιχεία είναι μονάδες, δηλαδή έχουν
πολλαπλασιαστικό αντίστροφο.

Παραδείγματα.

(i) Τα σύνολα Q, R, C με τις συνήθεις δυαδικές πράξεις της πρόσθεσης και του πολλα-
πλασιασμού είναι σώματα.

(ii) Η τριάδα (Z, +, ×) δεν αποτελεί σώμα γιατί τα μόνα μη μηδενικά στοιχεία που έχουν
αντίστροφο είναι οι αριθμοί 1 και −1.

Κλείνουμε την ενότητα αυτή με ένα πολύ σημαντικό θεώρημα, το οποίο όμως δεν αποδυ-
κνείουμε γιατί η απόδειξή του ξεφεύγει απο τους σκοπούς του παρόντος.

Θεώρημα 3. Ο δακτύλιος Zn αποτελεί σώμα αν και μόνο αν ο ακέραιος n είναι πρώτος


αριθμός.

16
3.2 Το Θεώρημα και Εφαρμογές
Θα παρουσιάσουμε αρχικά μια απλή γενίκευση ενός πολύ γνωστού θεωρήματος που λέει
ότι ένα πολυώνυμο (μιας μεταβλητής) βαθμού k δεν μπορεί να έχει περισσότερες απο μια
ρίζες.

Λήμμα. Έστω F ένα τυχαίο σώμα και έστω f = f (x1 , x2 , · · · , xn ) ∈ F[x1 , x2 , · · · , xn ], έτσι
ώστε ο βαθμός του f ως προς τη μεταβλητή xi να είναι το πολύ ti , 1 ≤ i ≤ n. Έστω
S1 , S2 , · · · , Sn υποσύνολα του F με |Si | ≥ t1 +1. Αν f (s1 , s2 , · · · , sn ) = 0 για κάθε (s1 , s2 , · · · , sn ) ∈
S1 × S2 × · · · Sn , τότε f ≡ 0.

Απόδειξη. Χρησιμοποιούμε επαγωγή στο n, τον αριθμό των μεταβλητών. Για n = 1, το


λήμμα λέει απλά οτι ένα πολυώνυμο βαθμού t μιας μεταβλητής το οποίο μηδενίζεται σε
t + 1 σημεία πρέπει να είναι το μηδενικό πολυώνυμο. Έστω οτι το λήμμα ισχύει για n − 1
μεταβλητές, με n ≥ 2. Χωρίς βλάβη της γενικότητας, θα χειριστούμε το f σαν πολυώνυμο
μεταβλητής xn :


tn
fn (xn ) = f (x1 , x2 , · · · , xn ) = gi (x1 , x2 , · · · , xn−1 )xin .
i=0

Το πολυώνυμο fn μηδενίζεται στα tn + 1 σημεία του Sn . Αφού deg fn = tn , συμπεραίνουμε


οτι fn ≡ 0, ή gi (x1 , x2 , · · · , xn−1 ) = 0 για κάθε i. Τώρα, αφού αυτό συμβαίνει για όλες τις
(n − 1)-αδες (x1 , x2 , · · · , xn−1 ) ∈ S1 × S2 × · · · × Sn−1 , απο την επαγωγική υπόθεση έχουμε οτι
gi ≡ 0 για κάθε i = 0, 1, · · · , tn . Επομένως f ≡ 0.

Θεώρημα 4. Έστω F ένα τυχαίο σώμα και έστω f = f (x1 , x2 , · · · , xn ) ∈ F[x1 , x2 , · · · , xn ].


Έστω S1 , S2 , · · · , Sn μη κενά υποσύνολα του F. Ορίζουμε τα πολυώνυμα

gi (xi ) = (xi − s).
s∈Si

Εαν f (s1 , s2 , · · · , sn ) = 0 για κάθε (s1 , s2 , · · · , sn ) ∈ S1 × S2 × · · · × Sn , τότε υπάρχουν


πολυώνυμα h1 , h2 , · · · hn ∈ F [x1 , x2 , · · · , xn ] με deg hi ≤ deg f − deg gi έτσι ώστε

f = h1 g1 + h2 g2 + · · · + hn gn .

Απόδειξη. Ορίζουμε ti = |Si | − 1. Θα χρησιμοποιήσουμε τον ακόλουθο αλγόριθμο:


Εαν υπάρχει μονόνυμο cxm 1 x2 · · · xn
1 m2 mn
στο f έτσι ώστε mi > ti για κάποιο i, τότε αντι-
m −(t +1)
mi
καθιστούμε τον παράγοντα xi του μονόνυμου με τον όρο xm i
i
− xi i i .gi (xi ). Αφού
deg gi = ti + 1 και ο συντελεστής του μεγιστοβάθμιου όρου του gi είναι η μονάδα, το μονό-
νυμο xm1 x2 · · · xn
1 m2 mn
αντικαθίσταται ουσιαστικά απο αρκετά μονόνυμα συνολικού βαθμού
μικρότερου του m1 + m2 + · · · mn . Κατα συνέπεια, σε κάθε βήμα, το άθροισμα των βαθμών
όλων των μονονύμων στο f μειώνεται αυστηρά. Αφού το άθροισμα είναι πεπερασμένο, ο
αλγόριθμος τερματίζεται σε πεπερασμένο αριθμό βημάτων.
Έστω f¯ το πολυώνυμο που έχουμε στο τέλος. Ο μετασχηματισμός
mi −(ti +1)
1 x2 · · · xn 7→ cx1 x2 · · · (xi − xi gi (xi )) · · · xm
mi
cxm1 m2 mn m1 m2
n
n

αντιστοιχεί στην αφαίρεση ενός όρου της μορφής hi ′ gi απο το f , όπου το μονόνυμο
mi −(ti +1)
h′i = cxm
1 x2 · · · xi
1 m2
· · · xm
n
n

ικανοποιεί την ανισότητα deg h′i = m1 + m2 + · · · + mn − (ti + 1) ≤ deg f − deg gi . Επομένως


f¯ = f − h1 g1 − h2 g2 − · · · − hn gn , για κάποια πολυώνυμα h1 , h2 , · · · hn ∈ F [x1 , x2 , · · · , xn ] που
ικανοποιούν την deg hi ≤ deg f − deg gi . Αφού το πολυώνυμο f¯ ήταν οτι απέμεινε στο τέλος

17
του αλγόριθμου, κάθε xi εμφανίζεται στο f¯ με εκθέτη το πολύ ti . Αφού επίσης gi (xi ) = 0
αν xi ∈ Si , συνεπάγεται οτι

f¯(s1 , s2 , · · · , sn ) = f (s1 , s2 , · · · , sn ) = 0

για κάθε (s1 , s2 , · · · , sn ) ∈ S1 × S2 × · · · Sn . Τότε οι προϋποθέσεις του πιο πάνω λήμματος


ικανοποιούνται και έτσι εφαρμόζοντάς το παίρνουμε f¯ ≡ 0 ή f = h1 g1 + h2 g2 + · · · + hn gn
όπως θέλαμε.
Έχουμε σε αυτό το σημείο μαζέψει όλα τα εργαλεία που χρειάζονται για να παρουσιάσουμε
το πιο σημαντικό θεώρημα της ενότητας.

Θεώρημα 5 (Συνδιαστικό Nullstellensatz). Έστω F ένα τυχαίο σώμα και έστω

f = f (x1 , x2 , · · · , xn ) ∈ F [x1 , x2 , · · · , xn ].

Υποθέτουμε οτι το πολυώνυμο f έχει βαθμό t1 + t2 + · · · + tn , όπου τα ti είναι μη αρνητικοί


ακέραιοι και ο συντελεστής του όρου xt11 xt22 · · · xtnn είναι μη μηδενικός. Εαν S1 , S2 , · · · , Sn
είναι υποσύνολα του F για τα οποία ισχύει |Si | > ti , τότε υπάρχει n-αδα (s1 , s2 , · · · , sn ) ∈
S1 × S2 × · · · Sn έτσι ώστε

f (s1 , s2 , · · · , sn ) ̸= 0.

Απόδειξη. Χωρίς βλάβη της γενικότητας, μπορούμε να υποθέσουμε οτι |Si | = ti + 1. Έστω
οτι δεν υπάρχει (s1 , s2 , · · · , sn ) ∈ S1 × S2 × · · · × Sn έτσι ώστε f (s1 , s2 , · · · , sn ) ̸= 0. Τότε απο
το Θεώρημα 4 έχουμε (χρησιμοποιώντας τους ίδιους συμβολισμούς) οτι


n
f = hi gi , με deg hi ≤ deg f − deg gi .
i=1

Απο την υπόθεση προκύπτει οτι ο συντελεστής του μονόνυμου xt11 xt22 · · · xtnn στο δεξί μέλος
της πιο πάνω ισότητας είναι μη μηδενικός. Όμως, deg hi gi ≤ deg f και ένα μονόνυμο του
hi gi είναι πλήρους βαθμού t1 + t2 + · · · + tn μόνο αν προκύπτει απο τον πολλαπλασιασμό
των μονονύμων μεγίστου βαθμου των hi και gi , που στην περίπτωση του gi συμβαίνει όταν
επιλέξουμε το μονόνυμο xtii +1 στην ανάλυση

gi (xi ) = (xi − s) = xtii +1 + (όροι μικρότερου βαθμού).
s∈Si

Επομένως, κάθε μονόνυμο στο άθροισμα h1 g1 + h2 g2 + · · · + hn gn βαθμού t1 + t2 + · · · + tn διαι-


ρείται απο το μονόνυμο xtii +1 για κάποιο i. Επομένως, ο συντελεστής του όρου xt11 xt22 · · · xtnn
στο δεξί μέλος της ισότητας είναι μηδενικός, που είναι άτοπο.
Ας δούμε τώρα πως εφαρμόζεται το Θεώρημα αυτό στη λύση του Προβλήματος 6 απο την
IM O του 2007.

Παράδειγμα 7 (IMO 2007, Πρόβλημα 6). Έστω n θετικός ακέραιος αριθμός. Έστω

S = {(x, y, z) | x, y, z ∈ {0, 1, · · · , n}, x + y + z > 0}

ένα σύνολο (n + 1)3 − 1 σημείων στον χώρο των τριών διαστάσεων. Να βρεθεί ο μικρότερος
αριθμός επιπέδων, των οποίων η ένωση συμπεριλαμβάνει το σύνολο S αλλά δεν συμπερι-
λαμβάνει το σημείο (0, 0, 0).

18
Λύση. Βλέπουμε εύκολα οτι τα 3n επίπεδα που δίνονται απο τις εξισώσεις x + y + z = i,
για i = 1, 2, · · · , 3n ικανοποιούν το ζητούμενο. Θα αποδείξουμε οτι ο αριθμός 3n είναι ο
μικρότερος αριθμός επιπέδων που χρειάζονται.
Θα χρησιμοποιήσουμε την εις άτοπο απαγωγή. Έστω οτι υπάρχουν επίπεδα P1 , P2 , · · · , Pk ,
με k ≤ 3n − 1, τα οποία καλύπτουν το S αλλά δεν παιρνούν απο το σημείο (0, 0, 0). Κάθε
επίπεδο Pi ορίζεται απο μια ∪ εξίσωση της μορφής ai x + bi y + ci z + di = 0, όπου di ̸= 0, αφού
(0, 0, 0) ∈
/ Pi . Τότε η ένωση Pi καλύπτει το S αν και μόνο αν το πολυώνυμο

k
g(x, y, z) = (ai x + bi y + ci z + di )
i=1

μηδενίζεται σε κάθε σημείο του S. Αφού η ένωση των επιπέδων δεν συμπεριλαμβάνει το
(0, 0, 0), έχουμε g(0, 0, 0) ̸= 0.
Για να εφαρμόσουμε το Συνδιαστικό Nullstellensatz, θα χρησιμοποιήσουμε το σώμα F = R.
Παρατηρούμε όμως οτι το σύνολο S που μας ενδιαφέρει δεν είναι της μορφής S1 × S2 × S3
για κάποια S1 , S2 , S3 ⊂ R.
Θεωρούμε το πολυώνυμο

n
f (x, y, z) = g(x, y, z) − c (x − i)(y − i)(z − i),
i=1

όπου
g(0, 0, 0)
c = .
(−1)3n (n!)3
Είναι προφανές οτι το f μηδενίζεται σε όλα τα σημεία του συνόλου S ∪ {(0, 0, 0)} =
S1 × S2 × S3 , όπου S1 = S2 = S3 = {0, 1, · · · , n}. Αφού k < 3n, έχουμε deg f = 3n. Ο
συντελεστής του όρου xn y n z n στο f ισούται με c ̸= 0. Τότε το Συνδιαστικό Nullstellensatz
εφαρμόζεται και άρα υπάρχει σημείο (x0 , y0 , z0 ) ∈ S1 × S2 × S3 τέτοιο ώστε f (x0 , y0 , z0 ) ̸= 0,
που είναι προφανώς άτοπο.
Στη συνέχεια, θα αποδείξουμε, χρησιμοποιώντας το Συνδιαστικό Nullstellensatz, το Θε-
ώρημα Cauchy-Davenport, που αποτελεί σημαντικό εργαλείο της Συνδιαστικής Θεωρίας
Αριθμών.
Ορισμός. Έστω (G, +) μια αβελιανή ομάδα και έστω A, B πεπερασμένα της υποσύνολα.
Τότε
A + B := { a + b | a ∈ A, b ∈ B } και
A − B := { a − b := a + (−b) | a ∈ A, b ∈ B }.
Θεώρημα 6 (Cauchy-Davenport). Αν A, B υποσύνολα του Zp , με p πρώτο αριθμό, τότε
|A + B| ≥ min{ p, |A| + |B| − 1 }.
Απόδειξη. Υποθέτουμε αρχικά οτι |A|+|B|−1 ≥ p. Τότε πρέπει να αποδείξουμε οτι |A+B| ≥
p και αφού τα A και B είναι υποσύνολα του Zp , πρέπει ουσιαστικά να αποδείξουμε οτι
A + B = Zp . Παρατηρούμε οτι αν έχουμε |A| + |B| > p, τότε, για κάθε x ∈ Zp , τα σύνολα A
και {x} − B τέμνονται. Επομένως, κάθε x μπορεί να γραφεί ως x = a + b για κάποια a ∈ A
και b ∈ B.
Υποθέτουμε τώρα οτι |A| + |B| < p και οτι |A + B| ≤ |A| + |B| − 2. Έστω C ένα
∏ σύνολο με
|A|+|B|−2 στοιχεία, έτσι ώστε A+B ⊂ C. Θεωρούμε το πολυώνυμο f (x, y) = c∈C (x+y −c)
βαθμού |A| + |B| − 2. Τώρα, αφού A + B ⊂ C, έχουμε f (a, b) = 0 για κάθε a ∈ A και b ∈ B.
Ο συντελεστής του όρου x|A|−1 y |B|−1 του f είναι ίσος με
( )
|A| + |B| − 2
.
|A| − 1
19
Αφού |A| + |B| − 2 < p, ο συντελεστής αυτός είναι μη μηδενικός στο σώμα Zp . Απο το
Συνδιαστικό Nullstellensatz έχουμε οτι υπάρχουν a ∈ A και b ∈ B με f (a, b) ̸= 0, άτοπο.
Θα τελειώσουμε την ενότητα με ένα απλό πρόβλημα του οποίου η εκφώνηση δεν παραπέπει
άμεσα στο Συνδιαστικό Nullstellensatz.

Παράδειγμα 8. Έστω {xi }, 1 ≤ i ≤ 6, ένα σύνολο έξι ακέραιων αριθμών, απο τους οποίους
κανένας δεν διαιρείται με το 7.

(i) Να αποδειχθεί οτι τουλάχιστον μια απο τις παραστάσεις x1 ± x2 ± x3 ± x4 ± x5 ± x6


διαιρείται με το 7, όπου το πρόσημο κάθε αριθμού στις παραστάσεις είναι ανεξάρτητο
απο τα άλλα.

(ii) Να γενικεύσετε το πιο πάνω σε κάθε πρώτο αριθμό p.

Απόδειξη. Θα αποδείξουμε οτι εαν p είναι περιττός πρώτος αριθμός και έχουμε p − 1 στοι-
∑p−1
χεία xi ∈ Zp , i = 1, 2, · · · , p−1, τότε υπάρχει ϵi ∈ {−1, +1} έτσι ώστε i=1 ϵi xi = 0. Προφανώς
η ύπαρξη τέτοιων ϵi συνεπάγεται την ύπαρξη ϵi με ϵ1 = +1.
Έστω p ένας περιττός πρώτος και y := (y1 , y2 , · · · , yp−1 ) ∈ S p−1 , όπου S := {−1, +1}. Ορί-
ζουμε το πολυώνυμο f (y) ∈ Zp [y1 , y2 , · · · , yp−1 ] όπως παρακάτω:
( p−1 )p−1

f (y) := 1 − xi yi .
i=1

∏p−1 ∑p−1
Παρατηρούμε οτι ο όρος i=1 yi είναι βαθμού p − 1 = i=1 (|S| − 1) και έχει συντελεστή


p−1

p−1
−(p − 1)! xi = xi ̸= 0.
i=1 i=1

Απο το Συνδιαστικό Nullstellensatz έχουμε τώρα οτι το f (y) ∑είναι μη μηδενικό σε κάποιο
σημείο ỹ := (ϵ1 , ϵ2 , · · · , ϵp−1 ) ∈ S . Επομένως, το άθροισμα i=1 ϵi xi ισούται με μηδέν.
p−1 p−1

20
A Α΄ & Β΄ Γυμνασίου

Μαθηματικό Βήμα

Κατασκευές τριγώνων με λογισμικό γεωμετρίας


ή με χάρακα και διαβήτη
Κωνσταντίνος Παπαγιάννης

Οι κατασκευές τριγώνων που θα παρουσιαστούν πιο κάτω βασίζονται σε κατασκευές με


χάρακα και διαβήτη. Θα παρουσιαστούν όμως και εντολές για να μπορούμε να προβούμε
στις κατασκευές αυτές με την χρήση του ελευθέρου λογισμικού GEOGEBRA. Το μέρος Α
αφορά τις εντολές του λογισμικού και το μέρος Β τις κατασκευές.

Κατασκευή ευθύγραμμου τμήματος με συγκεκριμένο μήκος.


Επιλέγουμε την εντολή «τμήμα με δοσμένο μήκος».
Πατούμε σε ένα σημείο της επιφάνεια εργασίας το οποίο θα αποτελεί το ένα άκρο του
τμήματος
Στην συνέχεια θα εμφανιστεί ένα κουτί διαλόγου το οποίο
μας ζητά να δώσουμε το μήκος του τμήματος. Θα πρέπει
εμείς να γράψουμε ένα αριθμό
Πατώντας το ΟΚ θα εμφανιστεί ένα ευθύγραμμο τμήμα ΑΒ
μήκους μονάδων.

Κατασκευή κύκλου με κέντρο Κ και ακτίνα μήκους R.


Επιλέγουμε την εντολή «κύκλος με κέντρο και ακτίνα»
Πατούμε σε ένα σημείο ( ) της επιφάνεια εργασίας το οποίο θα αποτελεί το κέντρο του
κύκλου.
Στην συνέχεια θα εμφανιστεί ένα κουτί διαλόγου το
οποίο μας ζητά να δώσουμε το μήκος της ακτίνας. Θα
πρέπει εμείς να γράψουμε ένα αριθμό
Πατώντας το ΟΚ θα εμφανιστεί ένας κύκλος με κέντρο
το και ακτίνα

Κατασκευή σημείου τομής δύο σχημάτων


Επιλέγουμε την εντολή «τομή δύο αντικειμένων»
Πατούμε διαδοχικά σε δύο σχήματα που τέμνονται και θα εμφανιστεί το σημείο τομής τους.

21
A Α΄ & Β΄ Γυμνασίου

Μαθηματικό Βήμα

Σχεδίαση τριγώνου ή πολυγώνου


Επιλέγουμε την εντολή πολύγωνο.
Στην συνέχεια κάνουμε κλικ διαδοχικά στις κορυφές του τριγώνου και τέλος κάνουμε κλικ
στην αρχική κορυφή . Θα εμφανιστεί το τρίγωνο

Κατασκευή γωνίας (ΒΑΓ) με συγκεκριμένο μέτρο.


Επιλέγουμε την εντολή «γωνία με δοσμένο μέγεθος» . Πατούμε σε ένα σημείο (σημείο της μιας
πλευράς) στην μετά στην κορυφή της γωνίας.
Στην συνέχεια θα εμφανιστεί ένα κουτί διαλόγου το οποίο μας
ζητά να δώσουμε το μέτρο της γωνίας. Θα πρέπει εμείς να
γράψουμε το μέτρο μιας γωνίας π.χ.
Πατώντας το ΟΚ θα εμφανιστεί η γωνία .

Επιλέγουμε την εντολή «ημιευθεία» και κάνουμε κλικ στα


σημεία και . Θα εμφανιστεί η ημιευθεία .

Απόκρυψη αντικειμένων
Στην περίπτωση που επιθυμούμε να μην φαίνονται κάποια από τα βοηθητικά σχήματα στην κατασκευή
μας π.χ. οι κύκλοι, τότε πατούμε δεξί κλικ στο αντικείμενο που θέλουμε να μην φαίνεται και επιλέγουμε
«Δείξε το αντικείμενο».
Προσοχή: Αν σε κάποιο σχήμα πατήσουμε διαγραφή τότε θα διαγραφεί και το συγκεκριμένο αντικείμενο
αλλά και όλα τα αντικείμενα που είναι συνδεδεμένα με αυτό.

Μέτρηση μήκους ευθύγραμμου τμήματος


Επιλέγουμε την εντολή «απόσταση ή μήκος» και πατούμε σε ένα ευθύγραμμο τμήμα. Θα
εμφανιστεί το μήκος του.

Μέτρηση μέτρου γωνιάς


Επιλέγουμε την εντολή «γωνία».
Για να μετρήσουμε την γωνία πατούμε διαδοχικά στα σημεία ή στα σημεία .

22
A Α΄ & Β΄ Γυμνασίου

Μαθηματικό Βήμα

Κατασκευές τριγώνων
Κατασκευή τριγώνου με δεδομένες τρεις πλευρές μήκους
Βήμα 1: Κατασκευάζουμε ευθύγραμμο τμήμα με μήκος
Βήμα 2: Κατασκευάζουμε κύκλο με κέντρο το και ακτίνα ίση με την
δεύτερη πλευρά
Βήμα 3: Κατασκευάζουμε κύκλο με κέντρο το και ακτίνα ίση με την τρίτη
πλευρά

Βήμα 4: Γράφουμε ως το ένα από τα σημεια τομής των δύο κύκλων

Βήμα 5: Σχεδιάζουμε το τρίγωνο


Βήμα 6: Κάνουμε απόκρυψη στους δύο κύκλους
Βήμα 7: Μετρούμε το μήκος των τριών πλευρών του τριγώνου

Προσοχή: Για να μπορέσουμε να κατασκευάσουμε ένα τρίγωνο θα πρέπει το


μήκος της μεγαλύτερης πλευράς του να είναι μικρότερο από το άθροισμα
των άλλων δύο πλευρών.
Είδη τριγώνων με βάση τις πλευρές τους
Κάθε τρίγωνο που κατασκευάζεται με τις πλευρές του άνισες.
ονομάζεται σκαληνό.
Κάθε τρίγωνο που κατασκευάζεται με ακριβώς δύο απόπλευρές τις πλευρές του ίσες
ονομάζεται Ισοσκελές

Κάθε τρίγωνο που κατασκευάζεται και με τις τρεις πλευρές του ίσες
ονομάζεται ισόπλευρο.

Σημείωση: Δύο τρίγωνα που έχουν κατασκευαστεί με τα αντιστοιχία μήκη πλευρών να είναι ίσα θα έχουν
και τις αντίστοιχες γωνίες τους ίσες.

23
A Α΄ & Β΄ Γυμνασίου

Μαθηματικό Βήμα

Κατασκευή τριγώνου με δεδομένες 2 πλευρές και την περιεχόμενη τους γωνία.


Τρίγωνο με πλευρές μήκους και την περιεχόμενη τους γωνία να έχει
μέτρο
Βήμα 1: Κατασκευάζουμε ευθύγραμμο τμήμα με μήκος
Βήμα 2: Κατασκευάζουμε κύκλο με κέντρο το και ακτίνα ίση με την δεύτερη
πλευρά .

Βήμα 3: Κατασκευάζουμε την περιεχόμενη γωνία

Βήμα 4: Κατασκευάζουμε ημιευθεία .


Βήμα 5: Σχεδιάζουμε το σημείο τομή της ημιευθείας και του κύκλου.

Βήμα 6: Σχεδιάζουμε το τρίγωνο


Βήμα 7: Αποκρύβουμε τον κύκλο, την ημιευθεία και το σημείο
Βήμα 8: Μετρούμε το μήκος των πλευρών και .

Σημείωση: Όλα τα τρίγωνα που κατασκευάζονται με τις δύο πλευρές τους


αντίστοιχα ίσες και τις περιεχόμενες προς αυτές γωνίες ίσες, έχουν και τα
υπόλοιπα στοιχεία τους αντίστοιχα ίσα.
Κάθε τρίγωνο που κατασκευάζεται με δύο δεδομένες πλευρές και την
περιεχόμενη τους γωνία να είναι ορθή, ονομάζεται ορθογώνιο.

Κάθε τρίγωνο που κατασκευάζεται με δύο δεδομένες πλευρές και την


περιεχόμενη τους γωνία να είναι αμβλεία, ονομάζεται αμβλυγώνιο.
Ένα τρίγωνο που κατασκευάζεται με δύο δεδομένες πλευρές και την περιεχόμενη προαυτές γωνία να
είναι οξεία δεν είναι σίγουρα οξυγώνιο.
Προσοχή: όταν μας δίνονται 2 δεδομένες πλευρές και η περιεχόμενη τους
γωνία τότε τα τρίγωνα είναι σίγουρα κατασκευάσιμα.

24
A Α΄ & Β΄ Γυμνασίου

Μαθηματικό Βήμα

Κατασκευή τριγώνου με δεδομένες 2 γωνίες και την προσκείμενη τους


πλευρά.
Τρίγωνο με 2 γωνίες που έχουν μέτρο ,και την προσκείμενη
προς αυτές πλευρά μήκους
Βήμα 1: Κατασκευάζουμε ευθύγραμμο τμήμα με μήκος
Βήμα2: Κατασκευάζουμε γωνία
Βήμα 3: Κατασκευάζουμε ημιευθεία

Βήμα4: Κατασκευάζουμε γωνία (ανάστροφη φορά , ρολόι)


Βήμα 5: Κατασκευάζουμε ημιευθεία .
Βήμα 6: Σχεδιάζουμε το Γ, σημείο τομής των δύο ημιευθειών.

Βήμα 7: Σχεδιάζουμε το τρίγωνο .


Βήμα 8: Απόκρυβούμε τις ημιευθειες και τα σημεί και .
Βήμα 9: Μετρούμε το μήκος της πλευράς .

Σημείωση: Όλα τα τρίγωνα που κατασκευάζονται με τις δύο γωνίες τους αντίστοιχα ίσες και τις
προσκείμενες πλευρές προς αυτές ίσες, έχουν και τα υπόλοιπα στοιχεία τους αντίστοιχα ίσα.

Κατασκευή τριγώνου με δεδομένες 2 γωνίες και την μη προσκείμενη προς αυτές πλευρά.
Π.χ.
Υπολογίζουμε την γωνία
Κατασκευάζουμε τρίγωνο με δεδομένες τις γωνίες και και την προσκείμενη πλευρά

Προσοχή: όταν μας δίνονται 2 δεδομένες γωνίες και η προσκείμενη προς αυτές πλευρά τότε για να είναι
κατασκευάσιμο το τρίγωνο πρέπει το άθροισμα των δύο δεδομένων γωνιών να είναι μικρότερο από

25
A Α΄ & Β΄ Γυμνασίου

Μαθηματικό Βήμα

Κατασκευή τρίγωνου με δεδομένες 2 πλευρές και μια μη περιεχόμενη γωνία


Π.Χ. και .
Βήμα 1: Κατασκευάζουμε ευθύγραμμο τμήμα με μήκος
Βήμα 2: Κατασκευάζουμε γωνία
Βήμα 3: Κατασκευάζουμε ημιευθεία

Βήμα 4: Κατασκευάζουμε κύκλο με κέντρο το και ακτίνα ίση με την


δεύτερη πλευρά .
Βήμα 5: Σχεδιάζουμε τα σημεία τομής και της ημιευθείας και του
κύκλου.

Βήμα 6: Σχεδιάζουμε το τρίγωνο ή το τρίγωνο .


Βήμα 7: Αποκρύβουμε τον κύκλο, την ημιευθεία και το σημείο .

Προσοχή: Με δεδομένο το μήκος δύο πλευρών και το μέτρο της μη


περιεχόμενης γωνίας το τρίγωνο δεν θα είναι σίγουρα κατασκευάσιμο.
Επίσης στην περίπτωση που είναι κατασκευάσιμο δεν προκύπτει
πάντοτε μοναδικό τρίγωνο.
Παρατηρούμε ότι από την συγκεκριμένη κατασκευή προκύπτουν δύο
και οι γωνίες ̂ και ̂ είναι παραπληρωματικές.
Για να προκύψει ένα και μοναδικό τρίγωνο θα πρέπει η γωνία να είναι ορθή.

26
A Α΄ & Β΄ Γυμνασίου

Μαθηματικό Βήμα

Κατασκευή τρίγωνου του οποίου η μια πλευρά είναι διάμετρος κύκλου


Δίνεται κύκλος με κέντρο και ακτίνα . Από τα άκρα της διαμέτρου και φέρουμε της εφαπτόμενες
και όπως φαίνεται στο πιο κάτω σχήμα.

1. Κατασκευή οξυγώνιου τριγώνου.


Δύο από τις κορυφές του τριγώνου είναι τα άκρα της διαμέτρου και η τρίτη κορυφή βρίσκεται
στην κίτρινη περιοχή, μεταξύ των εφαπτομένων και εκτός του κύκλου.
2. Κατασκευή ορθογωνίου τριγώνου.
Δύο από τις κορυφές του τριγώνου είναι τα άκρα της διαμέτρου και η τρίτη κορυφή βρίσκεται
πάνω σε μια κόκκινη γραμμή.
(α) Αν η κορυφή βρίσκεται στην περιφέρεια του κύκλου τότε η ορθή γωνία θα είναι η γωνία
(β) Αν η κορυφή Α βρίσκεται πάνω στην εφαπτόμενη τότε η ορθή γωνία θα είναι η γωνία
(γ) Αν η κορυφή βρίσκεται πάνω στην εφαπτόμενη τότε η ορθή γωνία θα είναι η γωνία
3. Κατασκευή αμβλυγώνιου τριγώνου.
Δύο από τις κορυφές του τριγώνου είναι τα άκρα της διαμέτρου και η τρίτη κορυφή βρίσκεται
στην μπλε περιοχή
(α) Αν η κορυφή βρίσκεται εντός του κύκλου ( ) τότε η αμβλεία γωνία θα είναι η
γωνία
(β) Αν η κορυφή βρίσκεται δεξιά από την εφαπτόμενη ( ) τότε η αμβλεία γωνία
θα είναι η γωνία
(γ) Αν η κορυφή βρίσκεται αριστερά από την εφαπτόμενη ( ) τότε η αμβλεία
γωνία θα είναι η γωνία

27
Γ΄ Γυμνασίου

Μαθηματικό Βήμα

ΠΑΡΑΓΟΝΤΟΠΟΙΗΣΗ-ΕΦΑΡΜΟΓΕΣ
Σάββας Τιμοθέου

Παραγοντοποιήση είναι η διαδικασία κατά την οποία μετατρέπουμε έναν αριθμό ή μία αλγεβρική
παράσταση σε γινόμενο πρώτων παραγόντων. Ας δούμε τις πιο βασικές περιπτώσεις παραγοντοποιήσης,
καθώς και τις εφαρμογές της σε κλάσματα, εξισώσεις και επίλυση προβλήματος.
Κατά την παραγοντοποίηση ενός πολυωνύμου ελέγχουμε αν το πολυώνυμο είναι:

ΕΙΔΟΣ ΠΕΡΙΠΤΩΣΕΙΣ-ΠΑΡΑΔΕΙΓΜΑΤΑ-ΣΧΟΛΙΑ
ΠΟΛΥΩΝΥΜΟΥ

ΜΟΝΩΝΥΜΟ: Δεν τίθεται θέμα παραγοντοποίησης του αφού εξ ορισμού είναι γινόμενο. Φυσικά ένα
μονώνυμο μπορεί να είναι από μόνο του ένας αριθμός, ή μία μεταβλητή ή γινόμενο
αριθμού με μεταβλητές. Για παράδειγμα:
Ένας αριθμός: ,
Μία μεταβλητή:
Γινόμενο αριθμού με μεταβλητές: ,ή

ΔΙΩΝΥΜΟ Ένα Διώνυμο έχει μεγάλες «πιθανότητες» να παραγοντοποιείται. Αυτό συμβαίνει όταν
έχει Κοινό Παράγοντα, είναι Διαφορά δύο τετραγώνων, είναι διαφορά ή άθροισμα
δύο Κύβων. Σε κάθε περίπτωση εφαρμογής μίας από τις πιο πάνω μεθόδους γίνεται
ξανά έλεγχος σε 2η φάση αν προκύπτει εκ νέου παραγοντοποίηση. Είναι όπως λέμε
συνδυασμός περιπτώσεων:
Κοινός Παράγοντας: , ή

.
Διαφορά δύο τετραγώνων: .
Η πιο πάνω επιλογή των Α και Β δεν είναι τυχαία! Τα και μπορεί να είναι
οποιεσδήποτε αλγεβρικές παραστάσεις και όχι μόνο μία απλή μεταβλητή. Για
παράδειγμα:

28
Γ΄ Γυμνασίου

Μαθηματικό Βήμα

( Για πιο απαιτητικές καταστάσεις). Τι συμβαίνει αν έχουμε περιπτώσεις της μορφής:


, με ;

√ √ √

(√ ) √ √

( )
√ √ √

(√ ) √ √

√ (√ ) √ √ √ √

Διαφορά δύο Κύβων:

( )( )
= .
Το πιο πάνω παράδειγμα αποτελεί μία απόδειξη παραγοντοποίησης του αθροίσματος 2
κύβων μέσα από τη διαφορά 2 κύβων!
Άθροισμα δύο Κύβων: .

( ).

( ) ( ) .

Συνδυασμός περιπτώσεων
( )
( Κοινός Παράγοντας-Διαφορά 2 τετραγώνων)

[ ]

( Κοινός Παράγοντας- Άθροισμα 2 κύβων)


Ερώτημα: (Για πιο απαιτητικές καταστάσεις)

Μπορεί το άθροισμα 2 τετραγώνων ( να αναλυθεί σε γινόμενο;


Ας μην βιαστούμε να δώσουμε απάντηση. Ας μελετήσουμε το τριώνυμο και στη συνέχεια
ξαναπροσπαθούμε!

29
Γ΄ Γυμνασίου

Μαθηματικό Βήμα

ΤΡΙΩΝΥΜΟ Κοινός Παράγοντας: .


.
Ανάπτυγμα τέλειου τετραγώνου:
.

.
Παρατηρώντας όταν οι 2 όροι είναι άθροισμα τετραγώνων, δηλαδή: και ο
ος
3 όρος τυχαίνει να είναι το 2-πλάσιο γινόμενο των , δηλαδή , τότε οι τρεις
όροι μαζί:
μας δίνουν το τέλειο τετράγωνο

( ) ( ) .

Τι γίνεται τελικά με το ;
Ας ξεκινήσουμε με την κλασσική «παρανόηση» μεγάλης μερίδας μαθητών

;;;
Πως μπορούμε να διορθώσουμε το πιο πάνω;

.
Τι μας θυμίζει;; Μήπως είναι διαφορά δύο τετραγώνων;;

Το είναι σίγουρα τέλειο τετράγωνο, αλλά το Ποια κατάλληλα


καθιστούν την ποσότητα τέλειο τετράγωνο;;
Αν ,και , ή και αντίστροφα ,και , φαίνεται ότι το
πρόβλημα παραγοντοποιήσης αθροίσματος 2 τετραγώνων θα έχει λύση!
Για παράδειγμα:

( ) ( ) ( ) ( )

( ) ( )( )

( ) ( ) ( )
( )
Γενικεύοντας:

(√ )
Θα επιστρέψουμε ξανά σε αυτό το σημείο όταν θα έχουμε τριώνυμο της μορφής

30
Γ΄ Γυμνασίου

Μαθηματικό Βήμα

Τριώνυμο με μορφή:

( )

Μια εναλλακτική λύση παραγοντοποιήσης τριωνύμου θα ήταν η μέθοδος


«συμπλήρωσης τέλειου τετραγώνου»
Ποσότητες της μορφής μπορούν να γραφούν ως:

Για παράδειγμα:

.
(Για πιο απαιτητικές καταστάσεις)
Επειδή τριώνυμα όπως το μπορούν πιο εύκολα να παραγοντοποιηθούν
με την προηγούμενη μέθοδο:
συνήθως αυτή τη μέθοδο ακολουθούμε!
Αν όμως, αντί το τριώνυμο: έχουμε το τριώνυμο , τότε
μπορεί να παραγοντοποιηθεί με τη γνωστή μέθοδο; Υπάρχουν 2 αριθμοί με γινόμενο
και άθροισμα ; Η απάντηση είναι ότι δεν υπάρχουν ακέραιοι αριθμοί με αυτή
την ιδιότητα ! Μήπως όμως υπάρχουν «άλλοι» αριθμοί; Ας μας βοηθήσει η «διαφορά 2
τετραγώνων». Θα προσπαθήσουμε να μετατρέψουμε το τριώνυμο σε τέλειο τετράγωνο
και αν δεν το καταφέρουμε, απλά θα προσθέσουμε ή θα αφαιρέσουμε κατάλληλα
κάποιο αριθμό.

(√ ) √ √

Τελικά οι 2 «παράξενοι αριθμοί» που ψάχναμε είναι οι: √ και √ διότι


πράγματι έχουν
Άθροισμα: √ √ και

31
Γ΄ Γυμνασίου

Μαθηματικό Βήμα

Γινόμενο: ( √ ) ( √ )
Ακόμη ένα παράδειγμα

(√ ) √ √ .

( ) ( )

√ √ √
( ) ( )

Kαι ακόμη ένα πιο απαιτητικό παράδειγμα.

[ ] [( ) ] [( ) ]

[( ) ( ) ] [( ) ( ) ( )]

ΠΟΛΥΩΝΥΜΟ Αν ένα πολυώνυμο περιέχει 4 όρους, σίγουρα θα κοιτάξουμε αν περιέχει κοινό


ΜΕ 4 Ή ΠΙΟ παράγοντα σε όλους τους όρους, όπως:
ΠΟΛΛΟΥΣ ΟΡΟΥΣ
.
Μπορεί ο κοινός παράγοντας να είναι κατά ομάδες, όπως:

32
Γ΄ Γυμνασίου

Μαθηματικό Βήμα

ΕΦΑΡΜΟΓΕΣ ΠΑΡΑΓΟΝΤΟΠΟΙΗΣΗΣ
Επίλυση εξίσωσης 2ου και ανώτερου βαθμού
Αν . (Ένας τουλάχιστον όρος είναι μηδέν). Επίσης ισχύει και για
περισσότερους όρους σε γινόμενο. Αν
Η πιο πάνω βασική ιδιότητα στους πραγματικούς αριθμούς είναι σημαντική στην επίλυση εξισώσεων
ανώτερου βαθμού.
Για Παράδειγμα

(διπλή ρίζα)

Επισημαίνουμε εδώ την «κλασσική» αντιμετώπιση από τους μαθητές


. Εδώ ο μαθητής αν «πειραματιστεί» με κάποιους ακέραιους αριθμούς με
γινόμενο , πιθανόν να πετύχει τη μία λύση ( συνήθως τη θετική). Πρέπει να τονιστεί ότι η μέθοδος αυτή
αποφεύγεται γιατί «κυνηγάμε» καταρχήν μόνο ακέραιους αριθμούς ως λύσεις και το πιο σπουδαίο, ότι
υπάρχει πληθώρα ζευγών με γινόμενο 35, όπως: ( ) √ √ κλπ. Ενώ
όταν το Β’μέλος μίας εξίσωσης είναι οι επιλογές ελαχιστοποιούνται!
Επίλυση προβλήματος!

Να βρείτε ορθογώνιο τρίγωνο με μήκη πλευρών διαδοχικούς ακέραιους


αριθμούς.
Αν τα μήκη των πλευρών είναι ακέραιοι διαδοχικοί, τότε μπορούμε να έχουμε:
και . Σε κάθε ορθογώνιο τρίγωνο ισχύει το Πυθαγόρειο Θεώρημα, άρα έχουμε

Άρα οι 3 πλευρές του τριγώνου έχουν μήκη: και .


Ας σημειώσουμε ότι η λύση στην εξίσωση απορρίπτεται αφού το παριστάνει μήκος και είναι
θετικός αριθμός!
Ορισμός και Απλοποίηση αλγεβρικών κλασμάτων
Εντοπίζουμε σε ένα κλάσμα εκείνες τις τιμές για τις οποίες έχει νόημα το κλάσμα ως πραγματικός αριθμός.

33
Γ΄ Γυμνασίου

Μαθηματικό Βήμα

Για παράδειγμα το κλάσμα

, ορίζεται όταν . Για να παραγοντοποιηθεί πρέπει να ελέγξουμε αν έχει κοινούς


παράγοντες σε αριθμητή-παρονομαστή.

Εδώ πρέπει να επισημάνουμε τους αναγκαίους περιορισμούς. Το κλάσμα: ορίζεται


όταν

, δηλαδή για . Να επισημανθεί ότι, αν και στην τελική απάντηση μετά από
την απλοποίηση δεν εμφανίζεται στον παρονομαστή ο όρος , αλλά είναι απαραίτητο να
γραφεί ο περιορισμός που ισχύει για το αρχικό μας κλάσμα και μας δίνει το «δικαίωμα» να
μπορούμε να απλοποιούμε κοινούς όρους όπως το .

Πράξεις με κλάσματα
Τα πιο πάνω μπορούν να επεκταθούν όταν έχουμε να κάνουμε τις 4 βασικές πράξεις με αλγεβρικά κλάσματα.

Για ποιες τιμές του πραγματικού αριθμού ορίζεται η παράσταση Στη συνέχεια να την απλοποιήσετε.
.

Για το κάθε κλάσμα ξεχωριστά έχουμε

Ωστόσο το κλάσμα αποτελεί διαιρέτης στην πιο πάνω περίπτωση και άρα θα πρέπει να
αποκλείσουμε εκείνες τις περιπτώσεις όπου μηδενίζεται. Δηλαδή θα πρέπει επίσης να έχουμε:

34
Γ΄ Γυμνασίου

Μαθηματικό Βήμα

. Έτσι συνολικά για την όλη πράξη της αλγεβρικής παράστασης στο έχουμε:
. Με αυτούς τους περιορισμούς είμαστε σε θέση να απλοποιήσουμε

Να τονίσουμε ιδιαίτερα ότι το Δ=1, είναι δηλαδή σταθερό, αλλά αΑυτό όμως δεν συμβαίνει για όλες τις
πραγματικές τιμές του . Ισχύει Δ=1, με .
Ακόμη και σε πιο πολύπλοκες πράξεις όταν έχουμε:

( ),

( ) ( )

,, .

Σε προβλήματα - Διερευνήσεις
Αν ένας εργάτης χρειάζεται μέρες για να τελειώσει μόνος του ένα έργο, τότε σε μία μέρα τι μέρος του
έργου θα τελειώσει;
Απλή αναλογία ( ποσά ευθέως ανάλογα). Αν συμβολίσουμε με το μέρος του έργου που θα τελειώσει σε

1μέρα, τότε:

Μπορούμε να το δούμε και με τον πιο κάτω πίνακα


Μέρες Μέρος έργου

Αν ένας 2ος εργάτης πιο «γρήγορος» χρειάζεται 5 λιγότερες για να τελειώσει μόνος του το ίδιο έργο, τότε
έχουμε ότι σε μια μέρα το μέρος του έργου που θα τελειώσει,
Μέρες Μέρος έργου
1
1

Είναι: .

Αν δουλέψουν μαζί, τότε το μέρος του έργου που θα τελειώσουν σε 1 μέρα θα είναι: .

Το πιο πάνω άθροισμα των 2 κλασμάτων μπορεί να γίνει απλό κλάσμα

Για , .

Αν γνωρίζουμε ότι δουλεύοντας μαζί οι 2 εργάτες, χρειάζονται συνολικά 6 μέρες για να τελειώσουν το έργο,

35
Γ΄ Γυμνασίου

Μαθηματικό Βήμα

να βρούμε σε πόσες μέρες τελειώνει από μόνος του ο κάθε εργάτης το έργο.

Αφού χρειάζονται μαζί 6 μέρες για ολόκληρο το έργο, τότε σε μία μέρα κάνουν το του έργου. Έτσι,

. Το απορρίπτεται διότι .Επομένως .


Ο 1ος εργάτης δηλαδή χρειάζεται 15 μέρες, ο 2ος εργάτης .

Επαλήθευση: ( Το μέρος του έργου που χρειάζονται σε μία μέρα όταν εργάζονται
μαζί).

*Ποια είναι η γραφική παράσταση της συνάρτησης ; Αφού η συνάρτηση ορίζεται


για όλες τις άλλες πραγματικές τιμές του και απλοποιείται . Είναι
εύθεία με ένα «κενό» σημείο. Για η αντίστοιχη τιμή δεν υπάρχει, οπότε η γραφική της είναι:

ΠΑΡΑΓΟΝΤΟΠΟΙΗΣΗ ΑΛΓΕΒΡΙΚΩΝ ΠΑΡΑΣΤΑΣΕΩΝ ΚΑΙ ΕΦΑΡΜΟΓΕΣ ΤΗΣ


Στην καρδιά των Μαθηματικών στεγάζονται σίγουρα πολλά! Δεν είμαστε σίγουροι αν μπορούμε να τα
απαριθμήσουμε όλα, μπορούμε όμως με βεβαιότητα να πούμε πολύ
σημαντικά: την « επίλυση προβλήματος» και την «απόδειξη
προτάσεων» .
ΕΠΙΛΥΣΗ ΕΞΙΣΩΣΕΩΝ ΑΝΩΤΕΡΟΥ ΒΑΘΜΟΥ-ΠΡΟΒΛΗΜΑΤΑ
Τρίγωνο ΑΒΓ έχει την πλευρά του BΓ να είναι κατά μεγαλύτερη
από το αντίστοιχο ύψος του ΑΔ. Αν το εμβαδόν του είναι , να
υπολογίσετε το μήκος της ΒΓ.
Nα δώσετε ένα δικό σας παράδειγμα προβλήματος που να ταιριάζει στην εξίσωση: ,αν
γνωρίζουμε ότι το αντιπροσωπεύει το μήκος ενός ορθογωνίου.
Το άθροισμα του τετραγώνου και του κύβου ενός αριθμού είναι ίσο με φορές τον ίδιο τον αριθμό. Να
βρείτε όλους τους αριθμούς με αυτή την ιδιότητα και να χαρακτηρίσετε το σύνολο στο οποίο ανήκουν.
ΟΡΙΣΜΟΣ- ΑΠΛΟΠΟΙΗΣΗ ΚΛΑΣΜΑΤΟΣ
Τι καταλαβαίνετε με τον συμβολισμό ;

36
Γ΄ Γυμνασίου

Μαθηματικό Βήμα

Πως θα ορίζατε γενικά τον αντίστροφο οποιουδήποτε πραγματικού αριθμού ;……………………………..........


Πως θα ορίζατε γενικά τον αντίστροφο οποιουδήποτε φυσικού αριθμού ;……………………………..........
Πως θα ορίζατε γενικά τον αντίστροφο οποιουδήποτε πρώτου αριθμού ;…………………………..........
Σε τι διαφέρουν οι πιο πάνω αριθμοί;……………………………………………………….
Ο Αντρέας για τη γιορτή του μοίρασε εξίσου σε όλα τα παιδιά της τάξης του 60 μικρά σοκολατάκια. Πόσα
πήρε το κάθε παιδί αν η τάξη του έχει
παιδιά;……………………………….
παιδιά; ………………………………..
παιδιά, αλλά εκείνη τη μέρα απουσίαζαν παιδιά. ………………..

Να βρείτε τις τιμές για τις οποίες ορίζεται το κλάσμα και στη συνέχεια να το απλοποιήσετε.

Να βρείτε τις τιμές για τις οποίες ορίζεται το κλάσμα .

Είμαστε σχεδόν σίγουροι ότι θα δυσκολευτείτε. Τι θα ζητούσατε από ένα Η/Υ ως βοήθεια; ( CAS –
GeoGebra)
Είναι γνωστή η γραφική παράσταση της . Ποια είναι
κατ τη γνώμη σας από τις 2 γραφικές και γιατί;
Τι ιδιαίτερο χαρακτηριστικό κατά τη γνώμη σας παρουσιάζει η γραφική
παράσταση της συνάρτησης ;

Να απλοποιήσετε το κλάσμα . Σε τι διαφέρει η συνάρτηση από την


;
ΠΡΑΞΕΙΣ ΜΕ ΚΛΑΣΜΑΤΑ-ΑΠΟΔΕΙΞΕΙΣ-ΠΡΟΒΛΗΜΑΤΑ
Να κάνετε τις πιο κάτω πράξεις

( )

37
Γ΄ Γυμνασίου

Μαθηματικό Βήμα

Αν το είναι φυσικός αριθμός, τότε:


ο επόμενος φυσικός αριθμός είναι …………………………………….
Οι αντίστροφοι αριθμοί των πιο πάνω φυσικών αριθμών είναι:………………………………
Ο μεγαλύτερος από τους 2 πιο πάνω αριθμούς είναι ………………………………………………
Η θετική διαφορά των αντίστροφων δύο διαδοχικών φυσικών αριθμών είναι
…………………………………………………………………………………………………………………………………
Να γράψετε 3 απλά κλάσματα που μπορούν να γραφούν ως διαφορά αντιστρόφων δύο διαδοχικών
φυσικών αριθμών

Να αποδείξετε ότι το άθροισμα: δεν μπορεί να υπερβεί τη μονάδα.

ΚΛΑΣΜΑΤΙΚΕΣ ΕΞΙΣΩΣΕΙΣ
Αν τα , και είναι πολυώνυμα τότε πως θα λύναμε τις πιο κάτω εξισώσεις; (Να δώσετε ένα
απλό παράδειγμα σε κάθε περίπτωση εξίσωσης, χρησιμοποιώντας πολυώνυμα πρώτου βαθμού)

; ;

Υπάρχει περίπτωση μια κλασματική εξίσωση να μην έχει λύση;


Να δώσετε παράδειγμα κλασματικής εξίσωσης που να μην έχει λύση.
Ποια εξίσωση αντιστοιχεί στο πρόβλημα:
«Ο Αντρέας για τη γιορτή του μοίρασε εξίσου σε όλα τα παιδιά της τάξης του 60 μικρά σοκολοτάκια. Την
ημέρα που θα κερνούσε απουσίαζαν 5 παιδιά. Αποφάσισε να δώσει τα σοκολατάκια σε όσους ήταν
παρόντες ( μαζί και ο εαυτός του). Διαπίστωσε ότι το κάθε παιδί, πήρε σοκολατάκι περισσότερο από ότι
θα έπαιρνε σε περίπτωση που ήταν όλοι παρόντες. Πόσα παιδιά έχει η τάξη του Αντρέα»;
Συμπληρώστε:
Η τάξη του Αντρέα έχει …………………… παιδιά
Τελικά τα σοκολατάκια θα τα μοιραστούν εξίσου………………………….. παιδιά
Τα σοκολατάκια που θα έπαιρνε το κάθε παιδί (χωρίς απόντες) είναι …………………
Τα σοκολατάκια που πήρε το κάθε παιδί τελικά είναι είναι ……………………………
Ποιος από τους δύο αριθμούς είναι μεγαλύτερος;……………………………………………..
Πόσο μεγαλύτερος είναι ο ένας αριθμός από τον άλλο;……………………………….
Πόσα είναι όλα τα παιδιά στην τάξη του Αντρέα;…………………………………………
Ασκήσεις: Να λύσετε τις πιο κάτω κλασματικές εξισώσεις-προβλήματα.

. .

38
Γ΄ Γυμνασίου

Μαθηματικό Βήμα

Μια παρέα μετά από το φαγητό σε ένα εστιατόριο ζήτησε το λογαριασμό. Ο λογαριασμός ήταν ,
αλλά δεν άφησαν το ένα ζευγάρι να πληρώσει γιατί ήταν η επέτειος τους. Να βρείτε πόσα πλήρωσε το
κάθε άτομο της παρέας, αν πλήρωσαν περισσότερα από ότι αν μοιράζονταν όλοι το λογαριασμό.

Το άθροισμα των αντιστρόφων 2 διαδοχικών φυσικών αριθμών είναι . Να βρείτε τους 2 διαδοχικούς
φυσικούς αριθμούς.

39
Α΄ Λυκείου

Μαθηματικό Βήμα

ΕΝΑ ΠΡΟΒΛΗΜΑ ΠΟΛΛΕΣ ΛΥΣΕΙΣ

Κωνσταντίνος Παπαγιάννης
Σάββας Τιμοθέου

Με αφορμή την πιο πάνω άσκηση στην Α’ Λυκείου που έχει δυσκολέψει αρκετούς μαθητές δίνουμε
κάποιες λύσεις τονίζοντας τη μεγάλη σημασία και όφελος τόσο από διδακτικής όσο και από μαθησιακής
πλευράς. Οι διαφορετικές αποδείξεις στο ίδιο πρόβλημα δείχνει πολύ καλές δεξιότητες δημιουργικότητας
και κριτικής σκέψης.

 Για το (α) ερώτημα


Αν πραγματικοί αριθμοί με , να δείξετε ότι (α)
1) ‘Eχουμε ισοδύναμα: ( )
( ) (ισχύει). Το ίσον ισχύει για δηλαδή .
2) Και άλλη λύση!!
( ) ( ) . Το ίσον ισχύει για
δηλαδή .

Και άλλη λύση!!


3) ( ) ( ) ( ) . Το ίσον ισχύει για .

Και άλλη λύση!!


4) Αν ετερόσημοι ( ή τουλάχιστον ο ένας ίσος με το 0). Τότε, και άρα, η πρόταση ισχύει!
Αν (√ √ ) √ √
√ . Το ίσον ισχύει για .

Και άλλη λύση!!


5) Αν με και άρα
Αν ,. Διαφορετικά .Άρα, ( )( )
. Το ίσον ισχύει για δηλαδή για

Και άλλη λύση!!


6) Αν √ . (Ανισότητα Α.Μ-ΓΜ). Το ίσον ισχύει για .
Και άλλη λύση!!
( )
7) {
( ) ( )

40
Α΄ Λυκείου

Μαθηματικό Βήμα

. Το ίσον ισχύει για .

Και άλλη λύση!!


8) Αν με ( σταθερό άθροισμα), το μέγιστο γινόμενο συμβαίνει όταν .
Άρα,
8) Αν , θέτοντας έχουμε
τότε .
Το ίσον ισχύει για δηλαδή για ή

ΓΕΩΜΕΤΡΙΚΕΣ ΛΥΣΕΙΣ

Το πρόβλημα μπορεί επίσης να αντιμετωπιστεί και γεωμετρικά δίνοντας έτσι περισσότερο «χρώμα» στις
αλγεβρικές αποδείξεις. Παραθέτουμε ακόμη 2 γεωμετρικές λύσεις.

 Το τρίγωνο είναι εγγεγραμμένο σε ημικύκλιο διαμέτρου .Αν θεωρήσουμε ότι ,


τότε ,ή . Aφού (Σε οποιαδήποτε θέση και αν είναι το ) και ( από την

ομοιότητα των ορθογωνίων τριγώνων ,τότε √ ( ) . Το


ίσον ισχύει όταν το ύψος του τριγώνου περάσει από το Ε έτσι ώστε να έχουμε , οπότε
και
 Στο πιο κάτω σχήμα το κάθε ένα από τα 4 ίσα ορθογώνια τρίγωνα με
κάθετες πλευρές √ και √ τοποθετούνται στο τετράγωνο πλευρά
√ ίσο με την υποτείνουσα των ορθογωνίων τριγώνων. Είναι
φανερό ότι το εμβαδόν του τετραγώνου είναι μεγαλύτερο από το
εμβαδόν των 4 τριγώνων. Αν , έχουμε (√ )
√ √

√ ( ) . Το ίσον ισχύει όταν


 Για το ερώτημα (β)
1) ( ) .
(Όταν )

2) Και άλλη λύση!!

41
Α΄ Λυκείου

Μαθηματικό Βήμα

( )
{
( ) ( )

3) Για κάθε ( ) ( )
( )
4) Για κάθε με ( )
( ) [( ) ] ( )

(Σημ. Το ισχύει για )

5) ( ) ( ) ( ) ( ) .

42
Β΄ Λυκείου

Μαθηματικό Βήμα

ΜΑΘΗΜΑΤΙΚΗ ΕΠΑΓΩΓΗ
Μια διδακτική πρόταση –Φύλλο εργασίας μαθητή
Ελένη Παπαθωμά
Μαθηματικός-Περιφερειακό Γυμνάσιο-Λύκειο Λευκάρων

Α. Επαγωγικός συλλογισμός:
Παράδειγμά 1: Να συμπληρώσετε τις παρακάτω ισότητες:

23
1 2 
2

3 4
1 2  3 
2

1  2  3  4  ........................

1  2  3  4  5  ................................
.......
1  2  3  .......  100  ...........................

1  2  3  4  .........  v  .........................

Παράδειγμα 2: Nα βρείτε ένα τύπο για το άθροισμα 1  3  5  7  ...   2  1 εργαζόμενοι επαγωγικά.

1  12
1  3  4  22
....

Παράδειγμα 3: Να βρείτε ένα γενικό τύπο για το πλήθος των διαγωνίων ενός κυρτού ν-γώνου.

   3
Γενικά: ν-γωνο Διαγώνιοι:
2

43
Β΄ Λυκείου

Μαθηματικό Βήμα

Β. Ανεπάρκεια επαγωγικού συλλογισμού:

Παράδειγμα 1: «Ο  2   41 είναι πρώτος αριθμός    »

  1  43   ώ
  2  47   ώ
  3  53   ώ
......
  40  402  40  41  40  40  1  41  40  41  41  41 40  1  412  ύ

Παράδειγμα 2: «Ο    διαιρείται με το κ, αν κ περιττος αριθμός και    »

 3   ί   3   π.χ.


23  2  6  3 6
......
 9     2  29  2  510  ά 9   ί  510

Παράδειγμα 3: «     ό 991 2  1   ί  έ  ά. »


 1 991 1  1   2
  2 991 4  1  3965   2
π.χ. .....
   12055735790331359447442538767 ( So min sky 1975)
 ό  991 2  1  ί  έ  ά .

Γ. Ανάγκη η επαγωγική διαδικασία να γίνει μαθηματική επαγωγή.


Υπάρχει λοιπόν η ανάγκη για μια αυστηρή συλλογιστική διαδικασία που με τη βοήθεια της θα
αποδεικνύουμε τις εικασίες που προέρχονται από την παρατήρηση και την επαγωγική διαδικασία.
Δηλαδή πως από την αλήθεια ειδικών περιπτώσεων θα καταλήγουμε στην αλήθεια γενικών περιπτώσεων.
Έτσι δημιουργήθηκε η μαθηματική ή τέλεια επαγωγή όπως την γνωρίζουμε σήμερα. Αν και η αρχή της
επαγωγής υπήρχε και σε αρχαία ελληνικά κείμενα (Πλάτωνας, Αριστοτέλης), η πρώτη σαφής διατύπωση
επαγωγικού συλλογισμού σε έργο γραμμένο στη δυτική γλώσσα είναι το Arithmeticorum Libri Duo (1575)
του ελληνικής καταγωγής Φραγκίσκου Μαυρόλυκου από τη Σικελία. Αργότερα ο Pascal το 1654 την
χρησιμοποίησε την επαγωγή στην απόδειξη των ιδιοτήτων του ομώνυμου τριγώνου.
Αργότερα ο Giuseppe Peano (Ιταλός Μαθηματικός και Φιλόσοφος, 1852 – 1932) σε μία προσπάθεια να
τυποποιήσει την Αριθμητική συμπεριέλαβε την αρχή της μαθηματικής επαγωγής στα πέντε αξιώματα του
για το σύνολο φυσικών αριθμών (Ν).
Αξίωμα 1: Υπάρχει ένας τουλάχιστον φυσικός αριθμός που είναι το 1, δηλ. 1 N .

44
Β΄ Λυκείου

Μαθηματικό Βήμα

Αξίωμα 2: Κάθε φυσικός αριθμός που συμβολίζεται με ν, έχει έναν επόμενο φυσικό αριθμό που
συμβολίζεται με ν+1 δηλ. Αν v  N  v  1 N .
Αξίωμα 3: Δεν υπάρχει φυσικός αριθμός που να έχει τον επόμενο τον φυσικό αριθμό 1, δηλαδή
    1  1.
Αξίωμα 4: Δεν υπάρχουν διαφορετικοί μεταξύ τους φυσικοί αριθμοί που να έχουν τον ίδιο επόμενο
δηλαδή αν ,      1    1     .

Αξίωμα 5: Αν για ένα υποσύνολο S του N ισχύουν τα εξής:

1. 1 S και 2. Αν v  S   v  1  S  ό  ί ό S  N .

(Αρχή της τέλειας ή μαθηματικής επαγωγής).


Οι όροι «τέλεια επαγωγή» ή «μαθηματική επαγωγή» καθιερώθηκαν τον 19ο αιώνα από τους De Morgan
και R. Dedekind.

Δ. Αρχή της μαθηματικής επαγωγής.


Υπάρχουν κάποιες μεταφορές και εξωτερικές αναπαραστάσεις οι οποίες ενδεχομένως βοηθούν στην
κατανόηση της σχέσης μεταξύ της μεθόδου της μαθηματικής επαγωγής και της διάταξης των φυσικών
αριθμών.
Για παράδειγμα: i. Το ανέβασμα της σκάλας, σκαλί – σκαλί
ii. Η μετάδοση ενός μηνύματος σε μια σειρά στρατιωτών
iii. Η αναχώρηση ενός τρένου, βαγόνι – βαγόνι
iv. Η είσοδος της βασίλισσας σε όλα τα κλειδωμένα δωμάτια του παλατιού,
δεδομένου πως έχει το κλειδί του πρώτου δωματίου και πως σε κάθε δωμάτιο
υπάρχει το κλειδί του επόμενου
v. Το γκρέμισμα μιας μεγάλης σειράς από ντόμινο ή βιβλία

Ας δούμε αναλυτικά το τελευταίο παράδειγμα που είναι και το πιο γνωστό:

Χαρακτηριστικά του παραδείγματος Αντίστοιχο χαρακτηριστικό της


επαγωγής
Ένα ντόμινο Ένας φυσικός αριθμός
Σειριακή τακτοποίηση των ντόμινο Διάταξη των φυσικών αριθμών
Γκρέμισμα των ντόμινο Ιδιότητα των αριθμών
Συνέπεια του σπρωξίματος Επαγωγικό βήμα
Σπρώξιμο του πρώτου ντόμινο Βασικό βήμα

45
Β΄ Λυκείου

Μαθηματικό Βήμα

Υπάρχει όμως, ο προβληματισμός μερικών ερευνητών πως η αναπαράσταση με τα ντόμινο αποτυγχάνει


να απεικονίσει την άπειρη φύση της μαθηματικής επαγωγής.
Αρχή της μαθηματικής επαγωγής

Συμβολίζουμε τον ισχυρισμό που θέλουμε να αποδείξουμε με P  v  v  N , που αναφέρεται στους


θετικούς ακέραιους. Αν:

(ι) Ο ισχυρισμός είναι αληθής για τον ακέραιο 1, δηλαδή P(1) είναι αληθής και

(ιι) Η αλήθεια του P  v  συνεπάγεται την αλήθεια του P  v  1 v  N τότε,

ο ισχυρισμός P  v  αληθεύει για όλους τους θετικούς ακεραίους ν.

Παρατηρήσεις:
1. Και τα δύο βήματα της επαγωγής είναι απαραίτητα π.χ.

(α) Ο ισχυρισμός: « ο v2  v  41 είναι πρώτος αριθμός  v  N » αληθεύει για ν=1 αλλά είναι
ψευδής  v  N .

(β) Ο ισχυρισμός: «Κάθε φυσικός αριθμός της μορφής 2ν είναι περίττος»

Αν και ο ισχυρισμός είναι προφανώς ψευδής ισχύει το 2ο βήμα της επαγωγής. Πράγματι αν ο 2ν,
v  N είναι περιττός τότε και ο 2  v  1  2v  2 είναι περιττός ως άθροισμα άρτιου και περιττού.

2. Πολλές φορές πρέπει να αποδείξουμε ότι ο ισχυρισμός P  v  αληθεύει όχι για κάθε θετικό ακέραιο ν

αλλά για κάθε v  v0 ,  v0  N 

Π.χ. (ι) 2  2v  1 v  3
v
1ο βήμα ν=3

(ιι) 3  2v  1 v  0
v
1ο βήμα ν=0

Άρα για την απόδειξη μίας πρότασης P  v  έχουμε να κάνουμε την εξής διαδικασία.

1ο Βασικό Βήμα: Αποδεικνύουμε την P  v  για ν=1.

2ο Επαγωγική υπόθεση: Υποθέτουμε ότι η P  v  είναι αληθής για κάποιο ν = κ δηλαδή ισχύει η P  k  .

3ο Επαγωγικό βήμα: Χρησιμοποιώντας την επαγωγική υπόθεση προσπαθούμε να αποδείξουμε ότι αν η


πρόταση ισχύει ν = κ τότε ισχύει και για ν = κ+1 δηλαδή, ότι ξεκινώντας από την υπόθεση ότι ισχύει η
P  k  καταλήγουμε ότι ισχύει η P  k  1 .

Ένα από τα μειονεκτήματα της μεθόδου της επαγωγής είναι ότι πρέπει κάνεις να γνωρίζει εκ των
προτέρων τον τύπο που περιγράφει την κατάσταση που μελετά γιατί τότε μόνο μπορεί να ξεκινήσει να την

46
Β΄ Λυκείου

Μαθηματικό Βήμα

αποδείξει. Στην πράξη χρειάζεται κανείς να διατυπώσει μία εικασία (ισχυρισμό) και μετά, στην περίπτωση
που η εικασία του ανταποκρίνεται στην πραγματικότητα να δώσει απόδειξη ότι είναι πράγματι αληθής.

v  v  1
Δραστηριότητα 1: Απόδειξη της πρότασης 1  2  ...  v 
2

Δραστηριότητα 2: Ερώτηση: Ποιος είναι μεγαλύτερος κατά την γνώμη σας το 2v ή το v v  N ;


2 *

Καθοδήγηση: Συμπληρώστε τις στήλες:


ν 2v v2
1 2 1
2 4 4
3 8 9
4 16 16
5 32 25
6 64 36
7 128 49
8 256 64
9 512 81
10 1024 100

Εικασία: 2  v  v5
v 2

Απόδειξη:

Δραστηριότητα 3:
Καθοδήγηση: Να σκεφτείτε ένα μικρό αριθμό (μονοψήφιο ή διψήφιο) να τον υψώσετε στον κύβο και να
προσθέσετε στο αποτέλεσμα το διπλασίο του αριθμού.
v 1 3
v  2  12
v  3  33
π.χ. για
v  4  72
v  5  135
v  6  528 .....

Παρατήρηση: Όλοι είναι πολλαπλάσια του 3.

Εικασία: Ο αριθμός v3  2v διαιρείται με το 3, v  N * .


Απόδειξη:

47
Β΄ Λυκείου

Μαθηματικό Βήμα

Εύρεση συνόλου τιμών συνάρτησης με εναλλακτικές μεθόδους


Παντελής Ζαμπυρίνης
Μαθηματικός- Οικονομολόγος-Στατιστικολόγος

Η εργασία αυτή είναι συμπληρωματική των όσων αναφέρει το σχολικό βιβλίο των Μαθηματικών
κατεύθυνσης της Β' Λυκείου, όσον αφορά το πεδίον τιμών συνάρτησης.
Ως γνωστό, πεδίον ορισμού μιας (πραγματικής) συνάρτησης με ( ), είναι το σύνολο
{ ( ) } και πεδίον (σύνολο) τιμών το σύνολο ( ) { ( )
} .
Συνήθως, αναζητούμε το σύνολο (πεδίον) τιμών πραγματικών συναρτήσεων ( ), των οποίων ο
τύπος περιέχει πηλίκα πολυωνύμων (ρητές συναρτήσεις) ή ριζικά ή λογαρίθμους ή τριγωνομετρικούς
αριθμούς της ανεξάρτητης μεταβλητής ή συνδυασμό των πιο πάνω. Συγκεκριμένα:
Πεδίον ορισμού είναι το σύνολο των πραγματικών αριθμών που παίρνει το , εκτός από :
1) τις τιμές που μηδενίζουν τον παρονομαστή (αν υπάρχει),
2) τις τιμές που κάνουν το υπόριζο αρνητικό,
3) τις τιμές που κάνουν το , όταν υπάρχει στον τύπο ,

4) τις τιμές στις οποίες δεν ορίζεται η ,

…κλπ .
Για να βρούμε το πεδίο τιμών, πρέπει να επιλύσουμε τη σχέση ( ) (τον τύπο της συνάρτησης) ως
προς και να βρούμε ποιες τιμές μπορεί να πάρει το , ώστε το να είναι στοιχείο του πεδίου
ορισμού ( ).
Όλη αυτή η διαδικασία μπορεί σε κάποιες περιπτώσεις να αποφευχθεί και το πεδίον τιμών να βρίσκεται
«με το μάτι», χρησιμοποιώντας κάποιες έξυπνες τεχνικές.
Για παράδειγμα, ξέροντας ότι το τετράγωνο κάθε πραγματικού αριθμού είναι μη αρνητικός αριθμός
( ), μπορούμε να βρούμε εύκολα το πεδίο τιμών χρησιμοποιώντας την τεχνική μετατροπής
παράστασης σε τέλειο τετράγωνο ( ) ή σε συμπλήρωση τέλειου τετραγώνου.
Επίσης πολύ χρήσιμές για τις έξυπνες τεχνικές είναι οι ιδιότητες | | ,
.
Οι έξυπνες τεχνικές βρίσκουν μεγάλη εφαρμογή στις περιπτώσεις του τύπου

( ) √ ( ).
Στην περίπτωση αυτή για να λύσουμε ως προς πρέπει θα πρέπει να υψώσουμε στο τετράγωνο και τα
δύο μέλη, αλλά αυτό μπορεί να μας οδηγήσει σε λανθασμένα συμπεράσματα αν δεν προσέξουμε την
δεδομένη συνάρτηση.

48
Β΄ Λυκείου

Μαθηματικό Βήμα

Εφαρμογές:
1. τότε
2. τότε
3. τότε
4. ( ) τότε
5. ( ) τότε
6. ( ) τότε
7. ( ) τότε
8. ( ) τότε

9. √ τότε τότε

10. √ τότε τότε

11. √ √( ) τότε

12. √ √( ) τότε

13. √ √( ) τότε

14. √ √( ) τότε

15. √ , τότε

16. √ , τότε

17. √ , τότε

18. √ , τότε

19. √ ( ) ( ) τότε

20. √ ( ) ( ) τότε

21. τότε (το , άρα αν το τότε ,


διαφορετικά ο παρονομαστής θα είναι πάντα μεγαλύτερος του 1 με αποτέλεσμα το κλάσμα να είναι
πάντα θετικό και μικρότερο του 1)

22. τότε

23. ( )
τότε

49
Β΄ Λυκείου

Μαθηματικό Βήμα

24. √ ( ] ) τότε

25. | | τότε

| |
26. { τότε { }

27. τότε ή { }
| |

28. τότε ή { }
| |

29. τότε

30. τότε

31. | |
τότε

32. Στην εύρεση της αντίστροφης συνάρτησης:


( ) )

( ) √
Έχουμε, ( ) { {

Άρα η αντίστροφη της : ( ) √ και

50
Γ΄ Λυκείου

Μαθηματικό Βήμα

ΛΙΓΗ ΕΞΑΣΚΗΣΗ ΠΡΙΝ ΤΙΣ ΠΑΓΚΥΠΡΙΕΣ ΕΞΕΤΑΣΕΙΣ

Αναστασία Ηρακλέους

 
1. Έστω μια συνάρτηση f η οποία είναι συνεχής στο 1, 2 και ισχύει f ' ( x )  2 για κάθε
x  1, 2 . Αν f (1)  1 να δείξετε ότι f ( 2)  1 .
2. α) Να διατυπώσετε τον ορισμό της πλάγιας ασύμπτωτης.

β) Με βάση τον ορισμό, να αποδείξετε ότι η γραφική παράσταση της συνάρτησης


2 x 2  5x  1
f ( x)  έχει ασύμπτωτη στο   την ευθεία y  2x  1 .
x2
3. Έστω η συνάρτηση f με f ( x)  ax  3x  2   ln x , ( a,   R ) .
2

α) Να βρείτε τα α, β ώστε η γραφική παράσταση της f να έχει σημείο καμπής το M (1,5) .

β) Για τις πιο πάνω τιμές των α, β να αποδείξετε ότι η εξίσωση f ( x)  0 έχει μόνο μια
πραγματική ρίζα.
ln( x  1)
4. Δίνεται η συνάρτηση f ( x)  .
x 1
α) Να βρείτε το σημείο της καμπύλης της f στο οποίο η εφαπτομένη είναι παράλληλη προς τον άξονα
χ΄χ.
β) Να εξετάσετε την f ως προς τη μονοτονία και τα ακρότατα.

γ) Να αποδείξετε ότι ln( x  1) e  x  1  0 , για κάθε x  1 .

5. Δίνεται η συνάρτηση f ( x )  ln x  e x , x  0 .
2

1
α) Να δείξετε ότι παρουσιάζει ολικό μέγιστο στο x  .
2e
β) Να δείξετε ότι ln x  e x , για κάθε x  0 .
2

6. Στο διπλανό σχήμα δίνεται η γραφική παράσταση

της συνάρτησης f ( x )  x  4 x  3 και η ευθεία


2

 :2 y  x  1  0 .
Να βρείτε το x0 (1,3) , ώστε η απόσταση ΑΒ

να είναι μέγιστη.

51
Γ΄ Λυκείου

Μαθηματικό Βήμα

7. Να διατυπώσετε τον ορισμό της κατακόρυφης ασύμπτωτης.


β) Να δείξετε ότι η γραφική παράσταση της συνάρτησης

f ( x )  x ln x  ( x  1) ln( x  1) , x  0 δεν έχει κατακόρυφη ασύμπτωτη.


8. Δίνεται η συνάρτηση f ( x )  x  3 
2
,   R * , η οποία έχει τοπικό ακρότατο στο x  2 .
x
α) Να βρείτε το λ.
β) Αν το  4:
- Να βρείτε τις ασύμπτωτες της γραφικής παράστασης της f.
- Να μελετήσετε την f ως προς τη μονοτονία και τα κοίλα της.
- Να χαράξετε τη γραφική παράσταση της f .

- Να βρείτε το πλήθος των ριζών της εξίσωσης f(x)  2013

9. Δίνεται η συνάρτηση f (x)  ln(ex  1) .


α) Να μελετήσετε την f ως προς τη μονοτονία και να βρείτε το σύνολο τιμών της.
β) Να αποδείξετε ότι η f είναι κυρτή (στρέφει τα κοίλα πάνω).
γ) Να βρείτε την εξίσωση της εφαπτομένης της Cf στο σημείο A(0,f (0)) .

ex  1 x
δ) Να αποδείξετε ότι ln  για κάθε x  R .
2 2
10. Έστω f δύο φορές παραγωγίσιμη συνάρτηση. Αν f ΄΄ ( x0 )  0 τότε η f έχει

σημείο καμπής στο x0 . Είναι σωστός αυτός ο ισχυρισμός; Να δικαιολογήσετε την

απάντηση σας.
11. Δίνεται η ευθεία ( ) : y  2 x  4 . Αν M ( x, y ) είναι ένα σημείο της ευθείας (ε) και φέρουμε
από το Μ τις ΜΑ και ΜΒ κάθετα στους άξονες χ΄χ και ψ΄ψ αντίστοιχα, να βρείτε τις συντεταγμένες
του Μ ώστε το ορθογώνιο ΟΑΜΒ να έχει μέγιστο εμβαδόν (Ο αρχή των αξόνων).

x3
12. Δίνεται η συνάρτηση f ( x ) 
x2
α) Να βρείτε το Π.Ο και τις τομές με τους άξονες
β) Να μελετήσετε την f ως προς τη μονοτονία και τα κοίλα της.

γ) Να βρείτε τις ασύμπτωτες της γραφικής παράστασης της f .

δ) Να χαράξετε τη γραφική παράσταση της f .

52
Γ΄ Λυκείου

Μαθηματικό Βήμα

ε) Να βρείτε το πλήθος των ριζών της εξίσωσης x   ( x  2) για τις διάφορες


3

τιμές του R .


13. Να υπολογίσετε τα ολοκληρώματα

3
 (6 x  x   2) dx ιι)   x   x dx
4 2
ι)
x
x2
ιιι)  x( x  5) ιv)  ( x   3 x  e
2 4
dx ) dx
14. Να υπολογίσετε τα ολοκληρώματα

6x  9 2x
α)  dx β)  2 dx
x 2  3x  4 x  4x  5

5  3 2 x
γ)  3 6 x   4 x dx δ)  dx
 2 x

 2 x 1 dx
ε)  3 2 x  e dx στ) 
6x  x2

x2
15. Να υπολογίσετε το ολοκλήρωμα  dx χρησιμοποιώντας την αντικατάσταση x  3 ,
9 x 2


0   ή με οποιονδήποτε άλλο τρόπο.
2
16. Να υπολογίσετε τα ολοκληρώματα

x3  2 x 2  3x
i)  2
x  2x  1
dx ii)  x  ln 1   x  dx

x 1
 συν x  ημ x dx
4 3
iii)  dx iv)
2 x3  x
dx x
17. Να υπολογίσετε το ολοκλήρωμα  2   x χρησιμοποιώντας την αντικατάσταση   t ή με
2
οποιονδήποτε άλλο τρόπο.
2
18. Να υπολογίσετε το ολοκλήρωμα  dx με το μετασχηματισμό u  x  3 ή και
x2 x3
διαφορετικά.

53
Γ΄ Λυκείου

Μαθηματικό Βήμα

19. Να βρείτε την εξίσωση της καμπύλης C που περνά από το σημείο   0,0  και ο συντελεστής
x
διεύθυνσης της εφαπτομένης σε κάθε σημείο  x, y  της C είναι ίσος με .
x 1
2

xv
20. Δίνεται το ολοκλήρωμα I v   2 dx με v  N * .
x 2

x v 1
α) Να δείξετε ότι I v   2 I v2
v 1
β) Να υπολογίσετε το ολοκλήρωμα I 6 .

 e ( f ( x)  f '( x)) dx  e f ( x)  c .
x x
21.α) Να αποδείξετε ότι:

 x  x  1
e
x
β) Να υπολογίσετε το ολοκλήρωμα: dx
 2 x
22. Να υπολογίσετε τα ολοκληρώματα

3 1 1
x
 (x  2)dx 0 x 2  1 dx xe
2 x
α) β) γ) dx
2 0

23. Αν η συνάρτηση f έχει συνεχή δεύτερη παράγωγο με f(π)  6 και

 (f(x)  f (x))ημxdx  11
''
να δείξετε ότι : f(0)  5
0

24. α)Χρησιμοποιώντας την αντικατάσταση x  α  u ή διαφορετικά, να αποδείξετε


α α

 x (α  x) dx   x (α  x) dx
κ λ λ κ
ότι: όπου  ,   * .
0 0

 x(1  x)
40
β) Να υπολογίσετε το ολοκλήρωμα dx .
0

25. Δίνονται οι συναρτήσεις f ( x)  x και g(x)  4  x . Έστω χωρίο Ω το χωρίο που περικλείεται
από τις γραφικές παραστάσεις των συναρτήσεων f , g και του άξονα των χ.

α) Να υπολογίσετε το εμβαδόν του χωρίου Ω

β) Τον όγκο του στερεού που παράγεται αν το χωρίο κάνει μια πλήρη στροφή γύρω από τον άξονα των
χ .

54
Γ΄ Λυκείου

Μαθηματικό Βήμα

26. Δίνονται τα σημεία M(α,α3 ) και Ν(α,0) , α  0 .

α) Να αποδείξετε ότι η γραφική παράσταση της συνάρτησης f(x)  x 3 χωρίζει την

επιφάνεια του τριγώνου ΟΜΝ σε δύο ισοδύναμα μέρη, όπου Ο η αρχή των

αξόνων.

β) Aν α  1, να βρείτε τον όγκο του χωρίου που περικλείεται από την Cf και την

ευθεία ΟΜ αν στραφεί μια πλήρη στροφή γύρω από τον άξονα των y.

27. Δίνεται η συνάρτηση y 2  x και σημείο της Α(4,2). Έστω Ω το χωρίο που περικλείεται από την Cf , την
εφαπτομένη της Cf στο Α και τον άξονα των x. Να υπολογίσετε τον όγκο του στερεού που παράγεται
από την πλήρη περιστροφή του χωρίου Ω γύρω από τον άξονα των x.


4

28. Αν I v    xdx ,
0

1
α) να δείξετε ότι    , 2
 1  2

4
  xdx .
4
β) Να υπολογίστε το ολοκλήρωμα
0
29. α) Να δώσετε τον ορισμό του κύκλου.

β) Με βάση τον ορισμό του κύκλου να δείξετε ότι η εξίσωση του κύκλου με κέντρο

το Κ(1,-3) και ακτίνα R=4 είναι C : x 2  y 2  2x  6y  6  0 .

γ) Δίνεται κύκλος C : x 2  y 2  4y  3  0 . Να βρείτε το κέντρο, την ακτίνα και

τις παραμετρικές εξισώσεις του


30. Να βρείτε την εξίσωση του κύκλου σε καθεμιά από τις παρακάτω περιπτώσεις:

α) έχει κέντρο το K(1, 2) και εφάπτεται εξωτερικά του κύκλου x  22  y  32  16

β) έχει κέντρο το σημείο K(3,1) και εφάπτεται της ευθείας  : y  x  6

γ) έχει κέντρο στον άξονα x΄x και διέρχεται από τα σημεία O(0,0) και A(2, 2)

55
Γ΄ Λυκείου

Μαθηματικό Βήμα

31. Δίνεται κύκλος με εξίσωση x 2  y2  25 . Να βρείτε:

α) την εξίσωση της εφαπτομένης κύκλου στο σημείου του Α(3,-4)

β) την εξίσωση της εφαπτομένης του κύκλου που είναι παράλληλη με την ευθεία ε : 4x  3y  7  0

32. Δίνονται οι κύκλοι C1 : x 2  y 2  4x  1  0 , και C 2 : x 2  y 2  6x  2y  1  0 και η ευθεία

 : y  2x  

α) Να βρείτε τη σχετική θέση των κύκλων C1 και C2

β) Να βρεθεί η του κ ώστε η ευθεία ε να είναι εφαπτομένη του κύκλου C1

33. Δίνεται ο κύκλος x  y  4 x  2 y  4  0 και το σημείο M(1,1) .


2 2

α) Να βρείτε το κέντρο και την ακτίνα του κύκλου.

β) Να δείξετε ότι το Μ είναι εσωτερικό σημείο του κύκλου.

γ) Να βρείτε την εξίσωση και το μήκος της χορδής, η οποία έχει μέσο το Μ.

34. Δίνεται ο κύκλος  x  3   y  2   9 και το σημείο A(6,8) .


2 2

α) Να βρείτε τη θέση του Α ως προς τον κύκλο.

β) Να βρείτε τις εξισώσεις των εφαπτόμενων που άγονται από το σημείο A(6,8) .

35. Δίνεται η εξίσωση ( Cλ ) : x 2  y2  2λx  4(λ  1)y  3λ  34  0 .

α) Να βρείτε τις τιμές του   R για τις οποίες η εξίσωση αυτή παριστάνει κύκλο.

β) Να βρείτε το γ.τ στον οποίο ανήκουν τα κέντρα αυτών των κύκλων

36. Δίνεται η λέξη « ΕΛΛΕΙΨΗ»


α) Να βρείτε πόσοι είναι όλοι οι αναγραμματισμοί της πιο πάνω λέξης.
β) Να βρείτε πόσοι από αυτούς ξεκινούν με φωνήεν.
γ) Να βρείτε πόσοι από αυτούς δεν έχουν τα σύμφωνα μαζί.
δ) Πόσες λέξεις μπορείτε να σχηματίσετε με 2 φωνήεντα και 2 σύμφωνα της πιο πάνω λέξης;

56
Γ΄ Λυκείου

Μαθηματικό Βήμα

37. α) Να βρείτε τα μήκη των αξόνων, τις εστίες, την εκκεντρότητα της και τις παραμετρικές εξισώσεις της
x2 y2
έλλειψης  1
36 144
β) Δίνεται η ισοσκελής υπερβολή xy  9 . Να βρείτε τις κορυφές, τις εστίες και τις παραμετρικές
εξισώσεις της.

1 1
38. Έστω τα ενεχόμενα Α και Β του ίδιου δειγματικού χώρου με P( Α )  , P( Β)  και
3 4
5
P( Α  Β) 
12
α) Να υπολογίσετε τις πιθανότητες P( Α ') , P(Α  Β) , P(Α  Β ')
β) Να υπολογίσετε την πιθανότητα να πραγματοποιηθεί ακριβώς ένα από τα Α,Β
γ) Είναι τα Α και Β ασυμβίβαστα ενδεχόμενα ;
39. Το 60% των μαθητών μιας πόλης έχουν κινητό τηλέφωνο. Το 40% έχουν
ηλεκτρονικό υπολογιστή (Η.Υ.) και το 25% και τα δύο. Αν επιλέξουμε τυχαία
ένα μαθητή της πόλης αυτής, να βρείτε τις πιθανότητες ο μαθητής αυτός:
(i) να έχει ένα μόνο από τα δύο
(ii) να έχει το πολύ ένα από τα δύο.

40. Θεωρούμε τον κύκλο x  y  a και το σημείο A(2α, 2α) .


2 2 2

Από τυχαίο σημείο Μ εκτός του κύκλου φέρνουμε εφαπτομένη στον κύκλο . Αν το μήκος του εφαπτόμενου
τμήματος είναι ίσο με την απόσταση ΜΑ, να δείξετε ότι η εξίσωση της γραμμής στην οποία κινείται το Μ
είναι 4 x  4 y  9a

41. Το σύνολο Ω  α1, α2 , α3 είναι ο δειγματικός χώρος ενός πειράματος τύχης και

2 3
Α  α1, α2  , Β  α2 , α3 δύο ενδεχόμενα του. Αν P( Α )  και P ( B)  να βρείτε τις
5 4
πιθανότητες P(α1) , P(α 2 ) , P(α3) .

42. Να βρεθεί η εξίσωση της παραβολής με κορυφή το (0, 0) στις παρακάτω

περιπτώσεις:

α) είναι συμμετρική ως προς τον άξονα Οy και διέρχεται από το σημείο (4,-1)

β) έχει άξονα της τον xx΄ και διευθετούσα την ευθεία δ : x  5

57
Γ΄ Λυκείου

Μαθηματικό Βήμα

43. Δίνονται οι παραβολές C1 : y 2  2x , C2 : y 2  4x και η μεταβλητή ευθεία ε : y  κ με κ  R* .

Έστω Α και Β τα σημεία τομής της ευθείας  με τις C1 και C2 , αντίστοιχα. Να αποδείξετε ότι αν το κ

μεταβάλλεται τότε ο γ.τ. του μέσου του ευθύγραμμου τμήματος ΑΒ βρίσκεται σε παραβολή.

44. Δίνεται η παραβολή y 2  12x και τα σημεία της P(3p2 , 6p) και T(3t 2 , 6t) .

α) Να δείξετε ότι η εξίσωση της εφαπτομένης στο P είναι x  py  3p2  0

β) Οι εφαπτόμενες στα σημεία P και T τέμνονται στο Σ. Αν το Σ βρίσκεται


ˆ  900 .
στην ευθεία x  12  0 , O η αρχή των αξόνων, να δείξετε ότι POT

γ) Να βρεθεί η εξίσωση του σχήματος στο οποίο ανήκει ο γ.τ. του Σ αν p2  t 2  2 .

45. Δίνεται η παραβολή y 2  4x . Να βρείτε:

α) τη θέση του σημείου Α(2,3) ως προς την παραβολή.

β) Τις εξισώσεις των εφαπτόμενων της παραβολής που περνούν από το σημείο Α.

46. Κατά την απογραφή του 1891 στην Μεγάλη Βρετανία, βρέθηκε ότι οι μελαχρινοί πατέρες με
μελαχρινούς γιούς ανέρχονται στο 5% του πληθυσμού, οι μελαχρινοί πατέρες με ξανθούς γιούς στο
7,9% του πληθυσμού, οι ξανθοί πατέρες με μελαχρινούς γιούς στο 8,9% του πληθυσμού και οι ξανθοί
πατέρες με ξανθούς γιούς στο 78,2%. Βρείτε την πιθανότητα να αποκτήσει ξανθό γιό ένας μελαχρινός
άνδρας.

 3   3
47. Δύο σημεία A 3 p,  B 3t ,  βρίσκονται σε ένα από τους κλάδους της υπερβολής xy  9 .
 p  t 
Φέρνουμε τις εφαπτόμενες της υπερβολής στα σημεία Α και Β οι οποίες τέμνονται στο σημείο Δ. Αν η
 3
εφαπτομένη της υπερβολής σε άλλο σημείο E  3κ,  είναι παράλληλη προς την ΑΒ να δείξετε ότι :
 κ
α) τα τρία σημεία Ε, Ο, Δ είναι συνευθειακά. (Το Ο η αρχή των αξόνων).
β) οι εφαπτόμενες στα σημεία Α και Β ποτέ δεν είναι κάθετες.

2 2
48. Δίνεται η ισοσκελής υπερβολή xy  4 και τα σημεία P ( 2 p, ) , T (2t , ) . Οι εφαπτόμενες στα Ρ
p t
1
και Τ τέμνονται στο Μ . Να βρείτε τον γ.τ. του Μ αν ισχύει    .
2

58
Γ΄ Λυκείου

Μαθηματικό Βήμα

x2 y2
49. Να βρείτε την εξίσωση της ευθείας ε, η οποία τέμνει την έλλειψη   1 σε δύο σημεία Κ, Λ
16 4
έτσι, ώστε το τμήμα ΚΛ να έχει ως μέσο το σημείο Μ(2,1).

50. Να βρείτε το γ.τ των μέσων των χορδών της παραβολής y 2  16x οι οποίες είναι παράλληλες προς

την ευθεία ε : y  2x  3 .

51. Δίνεται κύκλος C : x 2  y 2  R2 και σημείο T (Rσυνθ, Rημθ).

α) Να δείξετε ότι η εξίσωση της εφαπτομένης του κύκλου στο P είναι η συνθ  x  ημθ  y  R .

β) Η εφαπτομένη του κύκλου C στο σημείο T , τέμνει τον άξονα των Ox στο Α και τον Oy στο Β. Να
βρείτε την εξίσωση της καμπύλης στην οποία βρίσκεται ο γ.τ. της τέταρτης κορυφής Γ του ορθογωνίου
ΟΒΓΑ

59
Παγκύπριες Εξετάσεις

Μαθηματικό Βήμα

ΠΑΓΚΥΠΡΙΕΣ ΕΞΕΤΑΣΕΙΣ 2013

ΜΑΘΗΜΑΤΙΚΑ

Ημερομηνία: Πέμπτη, 30/5/2013

ΠΡΟΤΕΙΝΟΜΕΝΕΣ ΛΥΣΕΙΣ ΑΠΟ ΤΗΝ ΚΥΠΡΙΑΚΗ ΜΑΘΗΜΑΤΙΚΗ ΕΤΑΙΡΕΙΑ

ΜΕΡΟΣ A΄

Να λύσετε και τις 10 ασκήσεις του Μέρους Α΄.

1. Να βρείτε το αόριστο ολοκλήρωμα ( ) .

Λύση

( )

2. Να υπολογίσετε το όριο .

Λύση

( ) , μη επιτρεπτή πράξη.

Εφαρμόζουμε κανόνα De L’ Hospital: .

3. Να βρείτε τα τοπικά ακρότατα και τα σημεία καμπής της συνάρτησης , αν


υπάρχουν.

Λύση

, .
( ). ( ).

Για . Άρα η συνάρτηση παρουσιάζει τοπικό μέγιστο ( )

60
Παγκύπριες Εξετάσεις

Μαθηματικό Βήμα

Για . Άρα η συνάρτηση παρουσιάζει τοπικό ελάχιστο ( )

Άρα ( )

4. Δίνεται η έλλειψη , με . Αν η εστιακή απόσταση μονάδες και η

εκκεντρότητα της είναι , να βρείτε τις τιμές του και του και να γράψετε την εξίσωση της
έλλειψης.

Λύση

Εστιακή απόσταση
Εκκεντρότητα
Επίσης, .
Η εξίσωση της έλλειψης είναι:

5. Δίνεται ο πίνακας ( ) Να βρείτε τις πραγματικές τιμές των και , για τις οποίες ισχύει
( ), όπου και ( ) είναι ο μοναδιαίος και ο μηδενικός πίνακας αντίστοιχα.

Λύση

Είναι,

( ) ( )( ) ( ) ( ) ( )

( ) ( ) ( ) ( )

( ) ( ),

61
Παγκύπριες Εξετάσεις

Μαθηματικό Βήμα

απ΄ όπου { {

6. Δίνονται τα ψηφία . Να βρείτε:

(α) Πόσους τετραψήφιους αριθμούς μπορούμε να σχηματίσουμε με τα ψηφία αυτά αν δεν


επιτρέπεται επανάληψη ψηφίων.

(β) Πόσοι από τους πιο πάνω αριθμούς είναι άρτιοι.

Λύση

(α) Χρησιμοποιώντας την αρχή της απαρίθμησης, έχουμε:

Χ Ε Δ Μ
5

Άρα, το πλήθος των τετραψήφιων που μπορούμε να κατασκευάσουμε με τα πιο πάνω ψηφία, αν δεν
επιτρέπεται η επανάληψη ψηφίων, είναι:

αριθμοί

(β) Αριθμοί που λήγουν σε : Αριθμοί που λήγουν σε ή :

Χ Ε Δ Μ Χ Ε Δ Μ

αριθμοί αριθμοί

Αθροίζοντας τα πιο πάνω, οι άρτιοι αριθμοί που μπορούμε να σχηματίσουμε είναι :

αριθμοί

7. Δίνεται η υπερβολή και το σημείο της ( ).

(α) Να δείξετε ότι η εξίσωση της κάθετης της υπερβολής στο σημείο A είναι .

62
Παγκύπριες Εξετάσεις

Μαθηματικό Βήμα

(β) Η κάθετη της υπερβολής στο σημείο τέμνει ξανά την υπερβολή στο σημείο Β. Να βρείτε την
εξίσωση του κύκλου με διάμετρο την .

Λύση

(α) Παραγωγίζοντας, έχουμε :

, |

Άρα, η εξίσωση της κάθετης της υπερβολής στο σημείο της Α είναι η :

( )

(β) Σημείο :

} ( )

( ) √
{ {

( )

Ο ζητούμενος κύκλος έχει διάμετρο την ΑΒ. Θεωρούμε το σημείο Μ το μέσο της.

( ) ( )

Η ακτίνα του ζητούμενου κύκλου είναι η :


√( ) ( ) √ √ μονάδες

Επομένως, η εξίσωση του κύκλου είναι η :

63
Παγκύπριες Εξετάσεις

Μαθηματικό Βήμα

( ) ( )

8. Έστω και δύο ενδεχόμενα του ίδιου δειγματικού χώρου .

(α) Να γράψετε πότε τα ενδεχόμενα και είναι ασυμβίβαστα.

(β) Να αποδείξετε ότι ( ) ( ) ( ) ( ).

Λύση

(α) Δύο ενδεχόμενα Α και Β του ιδίου δειγματικού χώρου Ω είναι ασυμβίβαστα όταν η
πραγματοποίηση του ενός συνεπάγεται την μη πραγματοποίηση του άλλου και η μη
πραγματοποίηση του ενός δεν συνεπάγεται κατ’ ανάγκην την πραγματοποίηση του άλλου.

Ισοδύναμα, δύο ενδεχόμενα Α και Β του ιδίου δειγματικού χώρου Ω είναι ασυμβίβαστα αν
.

(β) Τα ενδεχόμενα και είναι ασυμβίβαστα. Ισχύει :

( ) ( ) [( ) ( )] ( )

( ) ( ) ( ) ( ) ( ) ( )

9. Να υπολογίσετε:

(α) Το εμβαδόν του χωρίου που περικλείεται από τις καμπύλες , , με ,


και τον άξονα .

(β) Τον όγκο του στερεού που παράγεται από την πλήρη περιστροφή του πιο πάνω χωρίου γύρω από
τον άξονα .

64
Παγκύπριες Εξετάσεις

Μαθηματικό Βήμα

Λύση

(α)


Σημείο Α : . Άρα ( )

( ) [ ] ( ) ( )

√ √
(√ ) τ. μ.

(β) ( ) [ ] κ. μ.

10. Συνάρτηση , με συνεχή δεύτερη παράγωγο στο , παρουσιάζει τοπικό ακρότατο στο
και η καμπύλη της περνά από το σημείο ( ). Αν ισχύει ( ( ) ( )) , να
υπολογίσετε το ( ).

Λύση

Η συνάρτηση είναι παραγωγίσιμη στο και παρουσιάζει τοπικό ακρότατο για . Συνεπώς,
( ) . Επιπλέον, η γραφική παράσταση της περνά από το σημείο ( ). Επομένως, ( ) .

( ( ) ( )) ( ) ( ( ))

∫ ( ) [ ( )] ∫ ( )

∫ ( ) ( ) ( ) [ ( )] ( ) ( )

65
Παγκύπριες Εξετάσεις

Μαθηματικό Βήμα

( ) ( )

ΜΕΡΟΣ B’ Να λύσετε και τις 5 ασκήσεις του Μέρους Β'.

1. Δίνεται η συνάρτηση .

Αφού βρείτε το πεδίο ορισμού, τα σημεία τομής με τους άξονες των συντεταγμένων, τα τοπικά
ακρότατα και τις ασύμπτωτες της συνάρτησης, να την παραστήσετε γραφικά.

Λύση

( )( )

Πεδίο ορισμού: { }

} τα σημεία τομής είναι ( ) και ( )

( ) ( )( )
( ) ( )

1 4

Άρα για ( )

για ( )

οριζόντια ασύμπτωτη.

66
Παγκύπριες Εξετάσεις

Μαθηματικό Βήμα

} κατακόρυφη ασύμπτωτη.

} κατακόρυφη ασύμπτωτη.

2. Δίνεται η καμπύλη και σημείο της ( ), .

(α) Να βρείτε την εξίσωση της εφαπτομένης της καμπύλης στο σημείο .

(β) Αν η εφαπτομένη στο σημείο τέμνει τους θετικούς ημιάξονες και σε σημεία και
αντίστοιχα, να δείξετε ότι το εμβαδόν ( ) του τριγώνου (Ο η αρχή των αξόνων) είναι
( ) ( ) .

(γ) Να βρείτε την τιμή του έτσι ώστε το εμβαδόν ( ) του τριγώνου να είναι μέγιστο.

Λύση

(α) |

Εξίσωση της εφαπτομένης είναι:

( ) (1)

67
Παγκύπριες Εξετάσεις

Μαθηματικό Βήμα

Επειδή το σημείο ( ) ανήκει στην γραφική παράσταση της συνάρτησης τότε,

(2)

Από τις (1) και (2) έχουμε ότι, ( ) είναι η εξίσωση εφαπτομένης.

(β)

Τα σημεία τομής της εφαπτομένης με τους άξονες είναι:

Αν ( ) ( )

Αν ( ) ( ( ))

( ) ( ) ( )( )
( ) ( )

(γ) ( ) [ ( ) ( ) ] ( )( )

( ) (απορρίπτεται)

1
( ) 0
( )

Άρα για το εμβαδόν του τριγώνου είναι μέγιστο.

3. Έξι παντρεμένα ζευγάρια βρίσκονται σε μια αίθουσα. Επιλέγουμε τυχαία τέσσερα άτομα από αυτά.
Να βρείτε:

(α) Την πιθανότητα να επιλεγούν παντρεμένα ζευγάρια μόνο.

(β) Την πιθανότητα να μην επιλεγεί κανένα παντρεμένο ζευγάρι.

68
Παγκύπριες Εξετάσεις

Μαθηματικό Βήμα

(γ) Την πιθανότητα να επιλεγεί ένα μόνο παντρεμένο ζευγάρι.

Λύση

Δυνατές περιπτώσεις ( ) ( ) .

( )
(α) Ευνοϊκές περιπτώσεις : ( ) ( ) . Άρα ( ) ( )
.

(β) Πρώτος τρόπος

Πρέπει να επιλέξουμε 4 άτομα μη παντρεμένα μεταξύ τους. Αυτό επιτυγχάνεται, αν

επιλέξουμε 4 από τα 6 ζευγάρια και στη συνέχεια από κάθε ζευγάρι ένα από τα δυο

άτομα. Άρα οι ευνοϊκές περιπτώσεις είναι: ( ) ( )( )( )( )( ) .

( )
Επομένως, ( ) ( )
.

Δεύτερος τρόπος

Το πρώτο άτομο επιλέγεται κατά 12 τρόπους, το δεύτερο κατά 10 τρόπους ( εξαιρούνται το άτομο που
επιλέγηκε και ο/η σύζυγός του), το τρίτο άτομο κατά 8 τρόπους

(εξαιρούνται τα δυο άτομα που επιλέγηκαν και οι σύζυγοί τους) και όμοια το τέταρτο άτομο κατά 6
τρόπους. Όμως δεν μας ενδιαφέρει η σειρά, συνεπώς οι ευνοϊκές περιπτώσεις είναι:

( )

( )
Επομένως, ( ) ( )
.

(γ) Πρώτος τρόπος

Πρέπει να επιλέξουμε 1 ζευγάρι από τα 6. Στη συνέχεια από τα υπόλοιπα 5 ζευγάρια να επιλέξουμε 2
ζευγάρια και κατόπιν 1 άτομο από κάθε ζευγάρι.

Άρα οι ευνοϊκές περιπτώσεις είναι: ( ) ( )( )( )( ) .

( )
Επομένως, ( ) ( )
.

69
Παγκύπριες Εξετάσεις

Μαθηματικό Βήμα

Δεύτερος τρόπος

( ) ( ) ( ) ( ) ( )

( ).

4. Δίνεται η παραβολή με εστία και τυχαίο σημείο της ( ), . Φέρουμε ευθεία


κάθετη στην στο σημείο , η οποία τέμνει τη διευθετούσα της παραβολής στο σημείο .

(α) Να δείξετε ότι η είναι η εφαπτομένη της παραβολής στο σημείο .

(β) Να βρείτε την εξίσωση της καμπύλης στην οποία ανήκει ο γεωμετρικός τόπος της κορυφής του
ορθογωνίου παραλληλογράμμου , καθώς το κινείται πάνω στην παραβολή.

Λύση

(α) ,

Εξίσωση της ΒΕ: ( )

Αν ( )

70
Παγκύπριες Εξετάσεις

Μαθηματικό Βήμα

Άρα

Επειδή και κοινό σημείο έχουμε ότι η ευθεία ταυτίζεται με την εφαπτομένη στο
σημείο .

(β) Το σημείο είναι το μέσο της και .

Άρα ( )

[ ( ) ]
} ( ) } ( ) ( )

Αν τότε ( ), ( ) και η κορυφή ( ). Οι συντεταγμένες του σημείου


επαληθεύουν την εξίσωση του γεωμετρικού τόπου.

5. Δίνονται δύο συνεχείς συναρτήσεις και , τέτοιες ώστε

( ) ( ) ( ) .

(α) Με τη βοήθεια της αντικατάστασης ή με οποιοδήποτε άλλο τρόπο, να δείξετε ότι


( ) ( ) .

(β) Να βρείτε την τιμή του ολοκληρώματος

71
Παγκύπριες Εξετάσεις

Μαθηματικό Βήμα

Λύση

(α) 1ος τρόπος:

και για , ενώ για .

Άρα ( ) ( ) ( )

( ) ( )

( ) ( )

( ( ) ( ))

( ) .

2ος τρόπος:

Αφού η συνεχής στο , είναι ολοκληρώσιμη στο [ ]. Έστω μια παράγουσα της .

( ) [ ( )] ( ) ( ) και

∫ ( ) ∫ ( ( ) ( )) [ ( )] [ ( )]

( ) ( ) ( ) ( ) ( ) ( ).

(β) Θεωρώ τη συνάρτηση ( ) , [ ]. Τότε,

( )
( ) .

Για κάθε [ ],

( ) ( ) ( ) ( ) .

Λόγω του ερωτήματος (α), έχουμε:

( ) =[ ] .

72
Παγκύπριες Εξετάσεις

Μαθηματικό Βήμα

ΠΑΓΚΥΠΡΙΕΣ ΕΞΕΤΑΣΕΙΣ 2013

ΜΑΘΗΜΑΤΙΚΑ ΚΟΙΝΟΥ ΚΟΡΜΟΥ

Ημερομηνία: Παρασκευή, 31 Μαΐου 2013

ΠΡΟΤΕΙΝΟΜΕΝΕΣ ΛΥΣΕΙΣ ΑΠΟ ΤΗΝ ΚΥΠΡΙΑΚΗ ΜΑΘΗΜΑΤΙΚΗ ΕΤΑΙΡΕΙΑ

ΜΕΡΟΣ Α΄:

Να λύσετε και τις 10 ασκήσεις του Μέρους Α΄.

1. Δίνεται κύλινδρος με ακτίνα βάσης . Αν το ύψος του κυλίνδρου είναι τριπλάσιο της ακτίνας της
βάσης του, να υπολογίσετε τον όγκο του κυλίνδρου.

Λύση
Ακτίνα:

Ύψος:

Όγκος: .

2. Δίνεται η λέξη «A N A K Α Μ Ψ Η». Να βρείτε:


α) το πλήθος των αναγραμματισμών της πιο πάνω λέξης.
β) πόσοι από αναγραμματισμούς αυτούς έχουν όλα τα σύμφωνα τους συνεχόμενα.

Λύση
α)
Τα γράμματα της λέξης είναι:
Άρα το πλήθος των αναγραμματισμών είναι

β) Θεωρούμε τα 4 σύμφωνα Ν, Κ, Μ, Ψ ως ένα «γράμμα», οπότε έχουμε 5 «γράμματα», από τα οποία


τα 3 είναι ίδια (Α, Α, Α).

Το πλήθος των αναγραμματισμών των 5 «γραμμάτων» είναι .

Επίσης, το πλήθος των αναγραμματισμών των 4 συμφώνων είναι .

73
Παγκύπριες Εξετάσεις

Μαθηματικό Βήμα

Σύμφωνα με την Βασική Αρχή της Απαρίθμησης, το πλήθος των των αναγραμματισμών με όλα τα
σύμφωνά τους συνεχόμενα είναι .

3. Ορθό πρίσμα έχει ύψος 6 cm και βάση τετράγωνο πλευράς 5 cm. Να υπολογίσετε το εμβαδόν της
ολικής επιφάνειας του πρίσματος.

Λύση
Πρώτος τρόπος
Ακμή βάσης:
Ύψος πρίσματος:
Περίμετρος βάσης:
Εμβαδόν παράπλευρης επιφάνειας:
Εμβαδόν βάσης:
Εμβαδόν ολικής επιφάνειας: .

Δεύτερος τρόπος
Το ορθό πρίσμα του προβλήματος είναι ορθογώνιο παραλληλεπίπεδο με διαστάσεις
. Άρα το εμβαδόν ολικής επιφάνειας είναι
( ) ( ) .

4. Στο πιο κάτω ραβδόγραμμα συχνοτήτων παρουσιάζονται τα επιστημονικά πεδία σπουδών και το
πλήθος των μαθητών που επέλεξαν το κάθε επιστημονικό πεδίο, σαν πρώτη επιλογή τους, τη σχολική
χρονιά 2012 – 2013, ενός τμήματος της Γ΄ Λυκείου.

α) Να υπολογίσετε το σύνολο των μαθητών του τμήματος.


β) Να βρείτε το ποσοστό % των μαθητών που επέλεξαν το πεδίο σπουδών Δ.
Λύση
α) Το σύνολο των μαθητών του τμήματος είναι:

β) Το ποσοστό των μαθητών που επέλεξαν το πεδίο Δ είναι

74
Παγκύπριες Εξετάσεις

Μαθηματικό Βήμα

5. Έμπορος αγόρασε εμπορεύματα αξίας με έκπτωση πάνω στην αξία των


εμπορευμάτων.
α) Να βρείτε πόσα ευρώ κόστισαν τα εμπορεύματα.
β) (Πώλησε) τα εμπορεύματα με κέρδος πάνω στο κόστος αγοράς των εμπορευμάτων. Να βρείτε
πόσα ευρώ κέρδισε.
Λύση
Πρώτος τρόπος
α)
Αξία Έκπτωση Κόστος
100 30 70
80000 x

ευρώ

β)
Κόστος Κέρδος
100 20
56000 y

ευρώ .

Δεύτερος τρόπος
α) Κόστος: ευρώ
β) Κέρδος: ευρώ.

6. Ο μέσος όρος της ηλικίας 10 παιδιών και 8 γονιών είναι 25 χρόνια. Αν ο μέσος όρος της ηλικίας των 8
γονιών είναι 40 χρόνια, να βρείτε:
α) το μέσο όρο που έχουν οι ηλικίες των 10 παιδιών σήμερα, και
β) το μέσο όρο που θα έχουν οι ηλικίες των γονιών μετά από 5 χρόνια.

Λύση
α) Πλήθος Μέση ηλικία
Μαθητές 10 ̅
Γονείς 8 ̅
Σύνολο 18 ̅
Ισχύει,
̅ ̅ ̅ ̅ ̅ χρόνια

β) Μετά από 5 χρόνια ο νέος μέσος όρος των ηλικιών των 8 γονιών θα είναι

75
Παγκύπριες Εξετάσεις

Μαθηματικό Βήμα

̅ ̅ χρόνια.

7. Ένα κεφάλαιο τοκίζεται με απλό τόκο για δύο χρόνια ως εξής: τα του κεφαλαίου προς 5% και το
υπόλοιπο μέρος του κεφαλαίου προς 4%. Να βρείτε το κεφάλαιο αν ο συνολικός τόκος που θα
αποδώσει σε δυο χρόνια είναι € 620 .

Λύση
Αρχικό Κεφάλαιο : Κ
Κεφάλαιο , αντίστοιχος τόκος σε 2 χρόνια προς 5%:
Κεφάλαιο , αντίστοιχος τόκος σε 2 χρόνια προς 4%:
Συνολικός τόκος:
ευρώ.

8. Για τα ενδεχόμενα και του ιδίου δειγματικού χώρου ισχύουν:


( ) , ( ) , ( ) .
Να βρείτε τις πιθανότητες ( ) ( ), ( ) και ( ) .

Λύση
 ( ) ( )
 ( ) ( ) ( ) ( ) ( )
 ( ) ( ) ( ) ( )
 ( ) .

9. Το εμβαδόν της ολικής επιφάνειας κανονικής τετραγωνικής πυραμίδας είναι και το


παράπλευρο ύψος της είναι ίσο με τα της ακμής της βάσης της.
α) Να αποδείξετε ότι το μήκος της πλευράς της βάσης της πυραμίδας είναι .
β) Να υπολογίσετε τον όγκο της πυραμίδας.

Λύση
Πλευρά βάσης: Κ

Παράπλευρο ύψος:
Ύψος πυραμίδας:

Δ Γ
α)

Μ
Λ
.
A Β

76
Παγκύπριες Εξετάσεις

Μαθηματικό Βήμα

β)
Στο ορθογώνιο τρίγωνο είναι , και .

Εφαρμόζουμε το Πυθαγόρειο Θεώρημα:

( )
Ο όγκος της πυραμίδας είναι

10. Σε ένα δοχείο υπάρχουν 6 κόκκινες και 2 πράσινες μπάλες. Σε ένα άλλο δοχείο υπάρχουν 6
κόκκινες και μερικές άσπρες μπάλες.
α) Επιλέγω μια μπάλα από το δοχείο . Να βρείτε τη πιθανότητα του ενδεχομένου
Π: «η μπάλα είναι πράσινη».
β) Αν η πιθανότητα να επιλέξω μια άσπρη μπάλα από το δοχείο είναι διπλάσια της πιθανότητας να
επιλέξω μια πράσινη μπάλα από το δοχείο , να βρείτε πόσες είναι οι άσπρες μπάλες του
δοχείου .

Λύση
α) Για το πρώτο δοχείο : ( )
β) Για το δεύτερο δοχείο :
Θεωρούμε το ενδεχόμενο Α: «Επιλογή άσπρης μπάλας από το ».
Αν οι άσπρες μπάλες στο δοχείο είναι , τότε
( ) ( )

ΜΕΡΟΣ Β΄
Να λύσετε και τις 5 ασκήσεις του Μέρους Β'.
1. Ο πιο κάτω πίνακας παρουσιάζει τον αριθμό των ορθογραφικών λαθών που έκαναν οι μαθητές ενός
Λυκείου σε μια έκθεση ιδεών.
Αριθμός λαθών ( ) 0 1 2 3 4 5 6

Αριθμός (μαθητών) ( ) 9 9 20 17 15 10 10
Να βρείτε:
α) την επικρατούσα τιμή ( )
β) την διάμεσο της κατανομής ( )
γ) τη μέση τιμή ( ̅ ) και
δ) την τυπική απόκλιση ( ) των λαθών.

77
Παγκύπριες Εξετάσεις

Μαθηματικό Βήμα

Λύση
α) Η επικρατούσα τιμή είναι η τιμή με τη μεγαλύτερη συχνότητα:
β) Τα επόμενα ερωτήματα θα απαντηθούν με τη βοήθεια του ακόλουθου πίνακα
̅ ( ̅) ( ̅)
0 9 9 0 –3 9 81
1 9 18 9 –2 4 36
2 20 38 40 –1 1 20
3 17 55 51 0 0 0
4 15 70 60 1 1 15
5 10 80 50 2 4 40
6 10 90 60 3 9 90
( ̅)

Αφού έχουμε παρατηρήσεις, τότε η διάμεσος της κατανομής είναι ο μέσος όρων των
παρατηρήσεων που βρίσκονται στη και στη θέση.
η η
Από τον πίνακα βλέπουμε ότι η 45 κι η 46 παρατήρηση είναι ίσες με 3, άρα η διάμεσος είναι
.

γ) Η μέση τιμή είναι ̅ ∑
∑ ( ̅)
δ) Η τυπική απόκλιση είναι √ ∑

2. Μια εταιρεία κάλεσε σε συνέντευξη 8 άνδρες και 10 γυναίκες για την πλήρωση έξι κενών θέσεων εργασίας.
Να βρείτε με πόσους τρόπους μπορεί να γίνει η πλήρωση των θέσεων, αν:
α) δεν υπάρχει κανένας περιορισμός.
β) θα προσληφθούν 3 άνδρες και 3 γυναίκες.
γ) θα προσληφθούν τουλάχιστον 4 άνδρες.
δ) θα προσληφθούν άτομα του ιδίου φύλου.

Λύση
α) Δεν υπάρχει περιορισμός, άρα πρέπει να επιλέξουμε 6 άτομα από τα 18 άτομα που έχει η εταιρία
στη διάθεση της.

Υπάρχουν ( ) διαφορετικοί τρόποι να γίνει η πλήρωση θέσεων

β) Πρώτα θα επιλέξουμε τους 3 άνδρες από τους 8. Αυτό μπορεί να γίνει με

( ) διαφορετικούς τρόπους

Έπειτα θα επιλέξουμε τις 3 γυναίκες από τις 10. Αυτό μπορεί να γίνει με

78
Παγκύπριες Εξετάσεις

Μαθηματικό Βήμα

( ) διαφορετικούς τρόπους

Με τη βοήθεια της Βασικής Αρχής της Απαρίθμησης, οι θέσεις εργασίας μπορούν να πληρωθούν με
διαφορετικούς τρόπους.

γ) Έχουμε τρεις περιπτώσεις:

4 άνδρες και 2 γυναίκες 5 άνδρες και 1 γυναίκα 6 άνδρες

( )( ) ( )( ) ( )

Επομένως η πρόσληψη τουλάχιστον 4 ανδρών γίνεται με


διαφορετικούς τρόπους.

δ) Σ’ αυτή την περίπτωση, έχουμε δύο περιπτώσεις:

Μόνο με άνδρες Μόνο με γυναίκες

( ) ( )

Επομένως η πρόσληψη ατόμων του ιδίου φύλου γίνεται με διαφορετικούς


τρόπους.

3. Ένα αυτοκίνητο αναχώρησε από την πόλη Α στις 6 το πρωί και κινείται προς την πόλη Β με σταθερή
ταχύτητα . Διέτρεξε τα της απόστασης μεταξύ των δυο πόλεων σε 10 ώρες. Στην συνέχεια
αύξησε την ταχύτητα του κατά της αρχικής του ταχύτητας και κινήθηκε με αυτή την ταχύτητα
μέχρι το τέλος της διαδρομής. Να βρείτε τι ώρα έφτασε στην πόλη Β.

Λύση
Τις πρώτες 10 ώρες το αυτοκίνητο διάνυσε
Αυτή η απόσταση είναι τα της συνολικής απόστασης των δύο πόλεων. Επομένως η απόσταση των
δύο πόλεων είναι

Η απόσταση που απομένει είναι


Για να βρούμε το χρόνο που χρειάζεται μέχρι να φτάσει στον προορισμό του, χρησιμοποιούμε τον
τύπο

Άρα συνολικά θα χρειαστεί για να φτάσει στην πόλη Β. Αφού ξεκίνησε στις 6:00 το πρωί,
σημαίνει ότι θα φτάσει στην πόλη Β στις 8:00 το βράδυ.

79
Παγκύπριες Εξετάσεις

Μαθηματικό Βήμα

4. Τέσσερις τουρίστες φτάνουν σε μια πόλη που διαθέτει 5 ξενοδοχεία. Αν ο κάθε τουρίστας θα επιλέξει
τυχαία το ξενοδοχείο στο οποίο θα διαμείνει, να βρείτε:
α) με πόσους διαφορετικούς τρόπους μπορεί να γίνει αυτό.
β) την πιθανότητα όλοι οι τουρίστες να μείνουν στο ίδιο ξενοδοχείο.
γ) την πιθανότητα οι τουρίστες να μείνουν σε διαφορετικά ξενοδοχεία.

Λύση
α) Πρώτος τρόπος
Οι τουρίστες μπορούν να επιλέξουν ένα από τα 5 ξενοδοχεία που έχουν στη διάθεση τους χωρίς
κανένα περιορισμό. Αυτό μπορεί να γίνει με τρόπους.
Δεύτερος τρόπος
Μπορεί να γίνει επίσης με τη Βασική Αρχή της Απαρίθμησης, με τον εξής τρόπο:
Φάσεις Τ1 Τ2 Τ3 Τ4

Τρόποι 5 5 5 5

Επομένως μπορεί να γίνει με τρόπους.

β) Θεωρούμε το ενδεχόμενο: Α: “οι τουρίστες να μείνουν στο ίδιο ξενοδοχείο”.

Έχουμε ( ) , αφού έχουμε να επιλέξουμε ουσιαστικά ένα από τα 5 ξενοδοχεία και

( ) , από το προηγούμενο ερώτημα. Η ζητούμενη πιθανότητα είναι

( )
( )
( )

γ) Θεωρούμε το ενδεχόμενο:

Β: “οι τουρίστες να μείνουν σε διαφορετικά ξενοδοχεία”

Πρώτος τρόπος

Η επιλογή των διαφορετικών ξενοδοχείων γίνεται με Διατάξεις, αφού είναι σημαντική και η σειρά
επιλογής τους. Επομένως έχουμε ( ) ( )
. Η ζητούμενη πιθανότητα είναι

( )
( )
( )

Δεύτερος τρόπος

Η επιλογή των διαφορετικών ξενοδοχείων μπορεί να γίνει και με τη Βασική Αρχή της Απαρίθμησης
.

80
Παγκύπριες Εξετάσεις

Μαθηματικό Βήμα

Φάσεις Τ1 Τ2 Τ3 Τ4

Τρόποι 5 4 3 2

Άρα έχουμε ( ) . Η ζητούμενη πιθανότητα είναι

( )
( )
( )

5. Στο διπλανό σχήμα φαίνεται ένα ανοικτό δοχείο


μεταφοράς γάλακτος που αποτελείται από δυο
κυλινδρικά τμήματα και ένα κόλουρο κώνο. Το ύψος του
δοχείου είναι 54 cm και οι διάμετροι των βάσεων του
κόλουρου κώνου είναι 30 cm και 24 cm. Τα ευθύγραμμα
τμήματα ΑΒ και ΓΔ έχουν μήκη 9 cm και 41 cm αντίστοιχα.

Να υπολογίσετε:
α) το εμβαδόν της ολικής εξωτερικής επιφάνειας και
β) τον όγκο του δοχείου.

Λύση
Ακτίνες βάσεων κόλουρου κώνου:

Ύψος κόλουρου κώνου:

Η γενέτειρα του κόλουρου κώνου υπολογίζεται με το


Α
Πυθαγόρειο Θεώρημα
Β

( ) ( ) ( )

( ) ( ) Ε
Γ

Ορίζουμε: και .

α) Το εμβαδόν ολικής εξωτερικής επιφάνειας του δοχείου


είναι
Δ

( )

81
Παγκύπριες Εξετάσεις

Μαθηματικό Βήμα

( )

β) Ο όγκος του στερεού είναι

( )

( )

82
Μαθηματικές Ιστορίες

Μαθηματικό Βήμα

Ο Αρχιμήδης, η Γραμμική Άλγεβρα και τα Βόδια


Κ. Κουππάρης

Ο Αρχιμήδης (287-212π.Χ) θεωρείται ίσως ο μεγαλύτερος μαθηματικός της αρχαιότητας και ένας από τους
σημαντικότερους όλων των εποχών. Στην Μαθηματική Ανάλυση επινόησε μεθόδους αντίστοιχες του
ολοκληρωτικού λογισμού των Λάιμπνιτς και Νεύτωνα. Πρόσφορε στην θεωρητική και εφαρμοσμένη
μηχανική, αλλά και στην αστρονομία. Επίσης, άφησε διάφορα άλλα σημαντικά μαθηματικά κείμενα,
συμπεριλαμβανομένης της περίφημης «Εφόδου».
Γύρω στο 1770 ανακαλύφθηκε από τον Γερμανό φιλόσοφο και ελληνολάτρη G.E. Lessing (1729-1781) ένα
χειρόγραφο στα Ελληνικά το οποίο υπό την μορφή ποιήματος αποτελούμενο από 22 δίστιχα ήταν η
εκφώνηση ενός προβλήματος που πολλοί θεωρούν ότι είχε θέσει ο Αρχιμήδης στο Ερατοσθένη (276π.Χ-
194π.Χ) , ένα άλλο επίσης πολύ μεγάλο Έλληνα μαθηματικό της αρχαιότητας.
Το πρόβλημα ήταν δύσκολο να λυθεί και από τότε ένα δύσκολο πρόβλημα λέγεται Αρχιμήδειο ή βόειο
πρόβλημα, επειδή το ποίημα αναφέρεται στον υπολογισμό του αριθμού των ταύρων και αγελάδων που
υπήρχαν στο κοπάδι του Θεού Ήλιου.
Θα λύσουμε πιο κάτω το πρόβλημα που προκύπτει από τα 15 πρώτα δίστιχα των οποίων μια πρόχειρη
μετάφραση είναι η εξής:
(1) Μέτρησε φίλε μου με τέλεια ακρίβεια τα γελάδια του Θεού Ήλιου. Λογάριασε τα με μεγάλη
φροντίδα, αν ισχυρίζεσαι την οποιαδήποτε σοφία.
(2) Πόσα γελάδια ταξινομημένα σε 4 αγέλες, υπήρχαν κάποτε και έβοσκαν εκεί στα λιβάδια του
νησιού της Σικελίας;
(3) Κάθε μία από αυτές τις 4 αγέλες με διαφορετικό χρώμα:
Η πρώτη αγέλη ήταν λευκή σαν γάλα, ενώ η δεύτερη γυάλιζε με ένα βαθύ εβένινο μαύρο.
(4) Καφετιά ήταν η τρίτη ομάδα, η τέταρτη ήταν με βούλες. Εκτός μιας υποδιαίρεσης οι ταύροι στις
αντίστοιχες αποχρώσεις ξεπερνούσαν κατά πολύ σε αριθμό τις αγελάδες.
(5) Τώρα, αυτές ήταν οι αναλογίες μεταξύ των ταύρων: Οι λευκοί θα εξισώνονταν σε αριθμό με τους
καφετί αν προσθέταμε σε αυτούς το 1/3,
(6) συν το 1/2 όλων μαζί των εβένινων ταύρων. Παραπέρα η ομάδα των μαύρων εξισωνόταν με το 1/4
των πιτσιλωτών,
(7) συν το 1/5 τους, παίρνοντας και όλους μαζί τους καφετί. Τέλος πρέπει να λάβεις υπόψη σου φίλε,
ότι όλοι οι πιτσιλωτοί,
(8) εξισώνονταν με το 1/6, συν το 1/7 της αγέλης των λευκών ταύρων, αν τους προσθέσουμε και
ολόκληρη την αγέλη των καφέχρωμων.
(9) Όμως, αρκετά διαφορετικές αναλογίες ισχύουν για το σώμα των θηλυκών: οι αγελάδες με
λευκόχρωμο τρίχωμα εξισώνονταν σε αριθμό με το 1/3,
(10) συν το 1/4 της μαύρης απόχρωσης γελαδιών αρσενικών και θηλυκών. Παραπέρα οι γελάδες με
χρώμα μαύρο αθροίζονταν σε αριθμό ίσο με το 1/4,
(11) συν το 1/5 όλων των πιτσιλόχρωμων, μετρώντας σ’ αυτόν τον λογαριασμό μαζί, κάθε πιτσιλωτή
αγελάδα και καθένα πιτσιλωτό ταύρο.

83
Μαθηματικές Ιστορίες

Μαθηματικό Βήμα

(12) Όμοια οι πιτσιλωτές αγελάδες συμπεριλάμβαναν το 1/5 συν το 1/6 του αθροίσματος όλων των
καφετί γελαδιών που έβοσκαν.
(13) Τέλος, οι καφέχρωμες αγελάδες συναποτελούσαν το 1/6 συν το 1/7 της λευκόχρωμης αγέλης,
αρσενικών και θηλυκών μαζί.
(14) Αν φίλε μου μπορείς να μου πεις ακριβώς ποιος ήταν ο αριθμός όλων όσων είχαν τότε μαζευτεί
εκεί, καθώς και τον ακριβή αριθμό,
(15) χρώμα προς χρώμα των καλοθρεμμένων αρσενικών και θηλυκών τότε δικαίως θα σε αποκαλούν
ικανότατο στο να κρατάς λογαριασμούς.

Εικόνες από τα βόδια του θεού Ήλιου και την σφαγή κάποιων από αυτά από τον Οδυσσέα και τους
συντρόφους του για θυσίες στους Θεούς και φαγητό. (πηγή: http://users.sch.gr, http://alexis-
chryssanthie.blogspot.com)

Λύση:
Συμβολίζουμε:
Λ/λ: λευκή αγέλη ταύρων/αγελάδων
Μ/μ: μαύρη αγέλη ταύρων/αγελάδων
Κ/κ: καφετιά αγέλη ταύρων/αγελάδων
Π/π: πιτσιλωτή αγέλη ταύρων/αγελάδων

Εξισώσεις Ταύρων:
Οι λευκοί θα εξισώνονταν σε αριθμό με τους καφετί αν προσθέταμε σε αυτούς το 1/3, συν το 1/2 όλων
μαζί των εβένινων ταύρων.
Λ= (1/2 + 1/3)Μ + Κ = 5/6. Μ + Κ => 6Λ – 6Κ = 5Μ (1)
Η ομάδα των μαύρων εξισωνόταν με το 1/4 των πιτσιλωτών,
συν το 1/5 τους, παίρνοντας και όλους μαζί τους καφετί

84
Μαθηματικές Ιστορίες

Μαθηματικό Βήμα

Μ = (1/4 + 1/5)Π + Κ = 9/20.Π + Κ => 20Μ – 20Κ = 9Π (2)


Όλοι οι πιτσιλωτοί, εξισώνονταν με το 1/6, συν το 1/7 της αγέλης των λευκών ταύρων, αν τους
προσθέσουμε και ολόκληρη την αγέλη των καφέχρωμων.
Π = (1/6 + 1/7)Λ + Κ = 13/42.Λ + Κ=> 42Π – 42Κ = 13Λ => Π= (13Λ + 42Κ)/42 (3)

Εξισώσεις Αγελάδων:
Οι αγελάδες με λευκόχρωμο τρίχωμα εξισώνονταν σε αριθμό με το 1/3 συν το 1/4 της μαύρης απόχρωσης
γελαδιών αρσενικών και θηλυκών.
λ = (1/3 + 1/4)(Μ + μ) => λ = (7/12) .(Μ + μ) (4)
Οι γελάδες με χρώμα μαύρο αθροίζονταν σε αριθμό ίσο με το 1/4 συν το 1/5 όλων των πιτσιλόχρωμων,
μετρώντας σ’ αυτόν τον λογαριασμό μαζί, κάθε πιτσιλωτή αγελάδα και καθένα πιτσιλωτό ταύρο.
μ = (1/4 + 1/5)(π + Π) => μ = (9/20). (Π + π) (5)
Οι πιτσιλωτές αγελάδες συμπεριλάμβαναν το 1/5 συν το 1/6 του αθροίσματος όλων των καφετί γελαδιών
που έβοσκαν.
π = (1/5 + 1/6)(Κ + κ) => π = (11/30). (Κ + κ) (6)
οι καφέχρωμες αγελάδες συναποτελούσαν το 1/6 συν το 1/7 της λευκόχρωμης αγέλης, αρσενικών και
θηλυκών μαζί.
κ = (1/6 + 1/7)(Λ + λ) => κ = (13/42). (Λ + λ) (7)
Καταλήγουμε λοιπόν σε ένα ομογενές σύστημα 7 εξισώσεων με 8 αγνώστους που πρέπει να έχει θετικές
ακέραιες λύσεις.
Από την (1) και (3) => 20Μ – 20Κ = 9/42 .(13Λ + 42Κ) =>
840Μ – 840Κ = 117Λ +378Κ => 840Μ = 117Λ + 1218Κ (8)
Η (1) επί 168 => 840Μ = 1008Λ -1008Κ (9)
Από (8) και (9) => 117Λ + 1218Κ = 1008Λ -1008Κ => Λ = (2226/891).Κ (10)
Αντικαθιστώντας την (10) στην (8) βρίσκουμε Μ = 178/99. Κ =>
Μ = (1602/891). Κ (11)
Αντικαθιστώντας την (10) στην (3) βρίσκουμε Π = (1580/891). Κ (12)
Έστω φυσικός αριθμός τ= Κ/891 => Κ =891τ (13)
(10 ) => Λ = 2226τ (14)
(11) => Μ = 1602τ (15)
(12) => Π = 1580τ (16)
Αντικαθιστώντας την (15) στην (4) => λ = (7/12).(1602τ + μ) => 12λ – 7μ = 11214τ (17)

85
Μαθηματικές Ιστορίες

Μαθηματικό Βήμα

Αντικαθιστώντας την (16) στην (5) => μ = (9/20).(1580τ + π) => 20μ - 9π = 14220τ (18)
Αντικαθιστώντας την (13) στην (6) => π = (11/30).(891τ + κ) => 30π – 11κ = 9801τ (19)
Αντικαθιστώντας την (14) στην (7) => κ = (13/42).(2226τ + λ) => 42κ – 13λ = 28938τ (20)
Θα λύσουμε τώρα το σύστημα των 4 εξισώσεων που έχει προκύψει από τις (17), (18), (19) και (20) με
άγνωστους τα λ,μ,π,κ.
Από τις (17) και (18) απαλείφουμε το μ πολλαπλασιάζοντας την (17) με το 20 και την (18) με το 7 και
προσθέτοντας τις δύο εξισώσεις που προκύπτουν κατά μέλη έχουμε την εξίσωση
240λ – 63π = 323820τ (21)
Από τις (19) και (20) απαλείφουμε το κ πολλαπλασιάζοντας την (19) με το 42 και την (20) με το 11 και
προσθέτοντας τις δύο εξισώσεις που προκύπτουν κατά μέλη έχουμε την εξίσωση
1260π – 143λ = 729960τ (22)
Στην συνέχεια λύνουμε το σύστημα δύο εξισώσεων με δύο αγνώστων που προκύπτει από τις (21) και (22)
και βρίσκουμε
π = (3515820/4657).τ (23) και λ = (7206360/4657).τ (24)
Αντικαθιστώντας το πιο πάνω αποτέλεσμα του π στην (19) βρίσκουμε
κ = (5439213/4657 ).τ (25)
Ενώ αντικαθιστώντας το πιο πάνω αποτέλεσμα του λ στην (17) βρίσκουμε
μ =(4893246/4657).τ (26)
Το 4657 είναι πρώτος. Θέτουμε τον φυσικό αριθμό ρ = τ/4657 => τ = 4657.ρ
στις (13), (14), (15) , (16), (23), (24), (25) και (26)
Προκύπτουν λοιπόν
Κ = 4149387ρ κ = 5439213ρ
Λ = 10366482ρ λ = 7206360ρ
Μ = 7460514ρ μ = 4893246ρ
Π = 7358060ρ π = 3515820ρ

Επομένως το πρόβλημα που προκύπτει από τα 15 πρώτα δίστιχα του βόειου προβλήματος του έθεσε ο
Αρχιμήδης, λύθηκε και έχει άπειρες λύσεις που προκύπτουν για τις διάφορες τιμές του φυσικού
αριθμού ρ.
Για την ελάχιστη δυνατή λύση για ρ=1 έχουμε:
Λευκή αγέλη ταύρων: 10366482 ζώα
Μαύρη αγέλη ταύρων: 7460514 ζώα

86
Μαθηματικές Ιστορίες

Μαθηματικό Βήμα

Καφετιά αγέλη ταύρων: 4149387 ζώα


Πιτσιλωτή αγέλη ταύρων: 7358060 ζώα

Λευκή αγέλη αγελάδων: 7206360 ζώα


Μαύρη αγέλη αγελάδων: 4893246 ζώα
Καφετιά αγέλη αγελάδων : 5439213 ζώα
Πιτσιλωτή αγέλη αγελάδων: 3515820 ζώα

Ελάχιστη λύση των 15 πρώτων διστίχων: 41 059 248 βόδια συνολικά.


Αξίζει να σημειωθεί ότι στο δεύτερο μέρος του προβλήματος που προκύπτει από τα δίστιχα 16 μέχρι 22
του ποιήματος απαιτείται το Π + Κ να είναι τρίγωνος αριθμός και το Λ + Μ να είναι τετράγωνος αριθμός
και με βάση αυτά πρέπει να υπολογιστεί η κατάλληλη τιμή του ρ. (Ορισμένοι δίνουν διαφορετική
ερμηνεία στην τελευταία συνθήκη επειδή οι ταύροι έχουν διαφορετικό μήκος από πλάτος, θα σχημάτιζαν
τετράγωνο όταν το πλήθος τους ήταν γινόμενο α×β).
Οι απαντήσεις του προβλήματος που προκύπτουν από όλα τα 22 δίστιχα είναι ασύλληπτα τεράστιοι
αριθμοί-χαώδεις. Η κάθε μια από τις απαντήσεις αυτές είναι αριθμός με περισσότερα από 206500 ψηφία
και για το λόγο αυτό είναι αδύνατο να τις προσδιορίσει κάποιος χωρίς την χρήση ηλεκτρονικού
υπολογιστή, κάτι που έγινε το 1965. Για να γραφτούν οι τιμές όλων των αγνώστων του προβλήματος
απαιτείται ένας τόμος 660 σελίδων. Το χειρόγραφο πάντως έδινε τις απαντήσεις για ρ = 80 (το οποίο δεν
ικανοποιεί τις δύο τελευταίες συνθήκες), χωρίς όμως να παρουσιάζει την λύση.
Ο Αρχιμήδης άσκησε μέσω των έργων του μεγάλη επιρροή όχι μόνο στους συγχρόνους του αλλά και στους
περισσότερους μεταγενέστερους μαθηματικούς σε ολόκληρο τον κόσμο. Απασχόλησε σε μεγάλο βαθμό
την ευρωπαϊκή επιστημονική σκέψη, καθώς και τους Άραβες επιστήμονες.
Ο αναγνωρισμένα πολύ σπουδαίος μαθηματικός G. Leibnitz (Λάιμπνιτς) γράφει για τον Αρχιμήδη κάτι που
δείχνει όλο το μεγαλείο και την διαχρονική αξία του αρχαίου Έλληνα Μαθηματικού:
«Όποιος διαβάζει τα έργα του Αρχιμήδη θαυμάζει λιγότερο τα επιτεύγματα των νεοτέρων».

Βιβλιογραφία:
 Λάμπρου Μ. (1992), Το Βοεικό πρόβλημα του Αρχιμήδη,
 Αναπολιτάνος Δ. – Καρασμάνης Β., Κείμενα Ιστορίας και Φιλοσοφίας των Αρχαίων Ελληνικών
Μαθηματικών, Εκδόσεις Τροχαλία, Αθήνα
 Στεφανάκη Σ. (2008), «Διδακτική προσέγγιση του εμβαδού της έλλειψης με αναφορά στο έργο Σφαιροειδή
και Κωνοειδή του Αρχιμήδη», Αθήνα

87
Λύση Προβλήματος

Μαθηματικό Βήμα

Μεθοδολογία για Λύση Προβλήματος


Ανδρέας Σκοτεινός

Ο Πρόεδρος μιας μεγάλης χώρας αποφασίζει να καταστρέψει τα χημικά όπλα μιας άλλης μικρότερης
επειδή τα χρησιμοποιούσε κατά τρόπο που καταπατεί τις διεθνείς συνθήκες. Καλεί λοιπόν τον
Επιτελάρχη του στρατού του και του δίνει οδηγίες για μια Στρατιωτική Επιχείρηση με στόχο την
καταστροφή των χημικών όπλων.
Ο Επιτελάρχης οργανώνει τους σχεδιασμούς του όπως φαίνεται πιο κάτω. Βοηθήστε τον αναλύοντας την
κάθε μια από τις δραστηριότητες που σκέφτεται ώστε να διαμορφώσει τον τρόπο που θα δράσει για να
λύσει το πρόβλημα που του έχει θέσει ο Πρόεδρός του:

Τι νομίζετε ότι θα μπορούσε να περιλαμβάνει η


κάθε μια από αυτές τις δραστηριότητες;
Συγκέντρωση Πληροφοριών

Καταρτισμός Σχεδίου Δράσης

Υλοποίηση Σχεδίου Δράσης

Έλεγχος για το τι επιτεύχθηκε

88
Λύση Προβλήματος

Μαθηματικό Βήμα

Κατά εντελώς ανάλογο τρόπο μπορούμε να διακρίνουμε τα ακόλουθα βασικά στάδια στην επίλυση ενός
μαθηματικού προβλήματος:

Στάδιο Δραστηριότητα - Αρχή


Στάδιο 1 Κατανόηση του προβλήματος – Συγκέντρωση Πληροφοριών
Στάδιο 2 Διαμόρφωση Σχεδίου Δράσης
Στάδιο 3 Υλοποίηση Σχεδίου Δράσης
Στάδιο 4 Έλεγχος αποτελεσμάτων/ επανεξέταση/ αναθεώρηση/ ανατροφοδότηση/
επέκταση
Μερικές καθοδηγητικές ερωτήσεις/ οδηγίες για να μπορέσουμε να λύσουμε ένα μαθηματικό πρόβλημα
είναι, σύμφωνα με τον πρωτοπόρο στον τομέα Μαθηματικό, George Polya, και οι ακόλουθες (χωρίς
βέβαια να σημαίνει ότι είναι και οι μόνες):
Στάδια ΑΡΧΗ Πιθανές ενέργειες/ ερωτηματικά που πρέπει να απαντηθούν
Στάδιο 1 Κατανόηση του Τι ζητείται; Μπορούμε να επαναδιατυπώσουμε το πρόβλημα με
Προβλήματος δικά μας λόγια; Κατανοούμε όλες τις φράσεις / έννοιες που
αναφέρονται στο πρόβλημα; Ποια τα δεδομένα και ποια τα
ζητούμενα; Έχουμε αρκετά δεδομένα για να προχωρήσουμε σε
λύση; Μπορούμε να βρούμε κάτι βοηθητικό για να
κατανοήσουμε το πρόβλημα (π.χ. ένα σχήμα); Μήπως
χρειαζόμαστε να ρωτήσουμε περισσότερες πληροφορίες για να
προχωρήσουμε; Μήπως θα μπορούσαμε να θεωρήσουμε απλές
περιπτώσεις που θα μας βοηθούσαν να κατανοήσουμε μερικές
ειδικές περιπτώσεις του προβλήματος;
Στάδιο 2 Διαμόρφωση ενός
σχεδίου
Στάδιο 3 Υλοποίηση του
σχεδίου
Στάδιο 4 Επανεξέταση/ Μαντέψετε και ελέγξετε. Ετοιμάστε ένα οργανωμένο πλάνο/
αναθεώρηση/ απαρίθμηση/ κατάσταση/ κατάλογο. Αποκλείστε διάφορες
επέκταση δυνατότητες. Θεωρείστε/ μελετήστε ειδικές περιπτώσεις.
Διαμορφώστε ένα απλούστερο πρόβλημα. Χρησιμοποιείστε
ευθύ συλλογισμό. Δοκιμάστε την μέθοδο της εις άτοπο
απαγωγής. Διαμορφώστε κάποιες εξισώσεις. Διερευνήστε για
κάποιο/ κάποια μοτίβο/α. Ετοιμάστε κάποιο διάγραμμα/ σχήμα.
Ακολουθείστε την αντίθετη πορεία (ξεκινώντας από το ζητούμενο
προς τα αρχικά δεδομένα για να μπορέσουμε να βρούμε κάποια
πορεία. Χρησιμοποιήστε κάποιους τύπους. Δοκιμάστε
δημιουργικές και πρωτότυπες προσεγγίσεις. Πειραματιστείτε.
Διαμορφώστε μοντέλα. Αλλάξετε τις απόψεις σας.

89
Λύση Προβλήματος

Μαθηματικό Βήμα

Χρησιμοποιείστε παραγωγικό συλλογισμό. Βάλτε το κεφάλι σας


να δουλέψει.
Προσπαθήστε με υπομονή να υλοποιήσετε το σχέδιο που
διαμορφώσατε. Αν δεν μπορείτε να το προχωρήσετε σκεφτείτε τι
δεν πάει καλά από το αρχικό σας σχέδιο και κάμετε κάποιες
αναθεωρήσεις πηγαίνοντας ξανά στο Στάδιο 2. Αν δεν υλοποιείτε
σκεφτείτε κάτι άλλο πηγαίνοντας ξανά στα Στάδιο 1 και Στάδιο 2.

Παράδειγμα
Πρόβλημα
Οι σελίδες ενός βιβλίου είναι αριθμημένες από το 1 μέχρι το 384. Να βρείτε πόσες φορές παρουσιάζεται
το ψηφίο 8 σε αυτή την αρίθμηση
Υποδείξεις για τη λύση
Κατανόηση του Τι ζητείται; Ποια τα δεδομένα και ποια τα ζητούμενα; Μήπως θα μπορούσατε να
Προβλήματος λύσετε το πρόβλημα για μια πιο απλή περίπτωση, π.χ για ένα βιβλίο που οι
σελίδες του είναι αριθμημένες από το 1 μέχρι το 92;
Διαμόρφωση ενός Πόσες φορές το 8 παρουσιάζεται ως ψηφίο των μονάδων του αριθμού;
σχεδίου Πόσες φορές το 8 παρουσιάζεται ως ψηφίο των δεκάδων του αριθμού;
Μήπως το 8 παρουσιάζεται ως ψηφίο των εκατοντάδων του αριθμού;
Υλοποίηση του Υλοποιείστε το πιο πάνω σχέδιο
σχεδίου
Επανεξέταση/ Επανερχόμαστε στο αρχικό πρόβλημα για να επαληθεύσουμε αυτά που βρήκαμε;
αναθεώρηση/ Μήπως υπάρχουν και άλλοι τρόποι επίλυσης;
επέκταση Μήπως θα μπορούσατε τώρα να αντιμετωπίσετε το ίδιο πρόβλημα για τον
αριθμό των σελίδων μιας εγκυκλοπαίδειας που είναι 94167;

Προβλήματα για Λύση

Πρόβλημα 1
Το γινόμενο του αριθμού 180 με το θετικό ακέραιο Ν είναι τέλειος κύβος. Ποια είναι η μικρότερη τιμή του
Ν;

Πρόβλημα 2
Δίνονται τα ψηφία 5, 3, 9, 7, 2 και 4. Να σχηματίσετε με αυτά δύο τριψήφιους αριθμούς, χρησιμοποιώντας
κάθε ψηφίο μια φορά έτσι ώστε το άθροισμα και το γινόμενο των τριψηφίων να είναι το μεγαλύτερο
δυνατό.

Πρόβλημα 3
Να λυθεί με δύο στρατηγικές τουλάχιστον το ακόλουθο πρόβλημα:

90
Λύση Προβλήματος

Μαθηματικό Βήμα

Ένας αστυνομικός σταθμός έχει 25 οχήματα εκ των οποίων μερικά είναι αυτοκίνητα (τετράκυκλα) και τα
υπόλοιπα μοτοσυκλέτες (δίκυκλες). Ο συνολικός αριθμός των τροχών τους είναι 70, Πόσα αυτοκίνητα και
πόσες μοτοσυκλέτες έχει ο σταθμός;

Πρόβλημα 4
Ένας ποδηλάτης θέλει να διανύσει μια απόσταση 10 km ανάμεσα στις πόλεις Α και Γ με μέση ταχύτητα 40
km/h. Στο μέσο όμως της διαδρομής υπάρχει το χωριό Β και για να φτάσει κανείς εκεί πρέπει να ανέβει
έναν αρκετά ανηφορικό δρόμο. Όταν ο ποδηλάτης έφτασε στο χωριό Β διαπίστωσε ότι η μέση ταχύτητά
του ήταν μόνο 21 km/h. Με πόση ταχύτητα πρέπει να κατέβει την πλαγιά ταξιδεύοντας από το χωριό Β
προς την πόλη Γ ώστε η συνολική μέση ταχύτητα του να είναι 40 km/h, όπως αρχικά το σχεδίαζε.

Πρόβλημα 5
Σε μια πόλη υπάρχουν 4 ποδοσφαιρικές ομάδες, οι Άπιαστοι, οι Βλαμμένοι, οι Γέροι και οι Δράκοι. Σε μια
πρόσφατη διοργάνωση έπαιξαν μεταξύ τους από ένα παιγνίδι ανά δύο και συγκέντρωσαν την ακόλουθη
βαθμολογία:

ΟΜΑΔΑ ΒΑΘΜΟΙ
Δράκοι 6
Άπιαστοι 4
Βλαμμένοι 4
Γέροι 2
Δοθέντος ότι για νίκη μια ομάδα συγκεντρώνει 3 βαθμούς, για ισοπαλία 1 βαθμό και για ήττα 0 βαθμούς
να βρεθούν τα ακόλουθα:
(1) Πόσες συναντήσεις είχαν αποτέλεσμα ισοπαλία;
(2) Ποιο ήταν το αποτέλεσμα της συνάντησης μεταξύ των Δράκων κα των Γέρων;
(3) Αν ξέρετε ότι οι Βλαμμένοι νίκησαν τους Δράκους, μπορείτε να βρείτε ποιο ήταν το αποτέλεσμα
της κάθε συνάντησης;

Πρόβλημα 6
Ο Δημήτρης και ο Μιχάλης θα πάνε από το σπίτι τους στην πόλη Α που βρίσκεται σε απόσταση 12 km.
Έχουν μόνο ένα ποδήλατο και αποφάσισαν να το χρησιμοποιούν έτσι ώστε ο ένας να ποδηλατεί για μια
απόσταση και ο άλλος να περπατά. Στη συνέχεια αυτός που ποδηλατούσε θα αφήνει το ποδήλατο στην
άκρη του δρόμου για να το πάρει ο άλλος όταν φτάσει εκεί και θα συνεχίσει με τα πόδια. Ο αρχικός
πεζοπόρος θα πάρει στη συνέχεα το ποδήλατο και θα διανύσει ποδηλατώντας την ίδια απόσταση όπως
και ο αρχικός ποδηλάτης. Θα επαναλαμβάνουν δε αυτή τη διαδικασία μέχρι που να φτάσουν στην πόλη Α.
Ξεκινούν και οι δύο την ίδια στιγμή. Δίδεται ότι ο Δημήτρης ποδηλατεί με ταχύτητα 15 km/h και περπατά
με ταχύτητα 6 km/h. Επίσης ξέρουμε ότι ο Μιχάλης ποδηλατεί με ταχύτητα 20 km/h και περπατά με
ταχύτητα 4 km/h.
Πόσο χρόνο θα τους πάρει για να πάνε στην πόλη αν χρησιμοποιήσουν την πιο καλή στρατηγική (αυτή
δηλαδή με την οποία θα επιτύχουμε το μικρότερο χρόνο διακίνησης προς την πόλη και για τους δύο) ;
………………………………

91
Λύση Προβλήματος

Μαθηματικό Βήμα

Πρόβλημα 7
Ένας περίεργος μαθητής λέει στο καθηγητή του των Μαθηματικών:
Άκουσα Κύριε ότι έχετε τρεις κόρες. Πόσον χρονών είναι;
Και ο καθηγητής του απαντά:
Το γινόμενο των ηλικιών τους είναι 72 και το άθροισμα των ηλικιών τους είναι ο αριθμός του δωματίου
στο οποίο βρισκόμαστε τώρα.
Ο μαθητής κάνει λίγες πράξεις και σε λίγο λέει στον καθηγητή του:
Κύριε δεν μας δώσατε αρκετά δεδομένα για μια μόνο απάντηση
Και ο καθηγητής απαντά:
Ω μα βέβαια ξέχασα να σας πω ότι η μεγαλύτερη κόρη μου έχει πράσινα μάτια.
Και ο μαθητής στη συνέχεια απαντά:
Μάλιστα τώρα είναι καθαρό οι ηλικίες τους είναι …..
Ποιες είναι οι ηλικίες που ζητούσε ο καθηγητής;

Πρόβλημα 8
Μια ομάδα φανατικών οπαδών μιας θρησκευτικής αίρεσης, αξιοποιώντας τις πληροφορίες των γραπτών
τους κειμένων και τις δυνατότητες των ηλεκτρονικών υπολογιστών κατέληξε στο συμπέρασμα ότι το τέλος
του κόσμου θα έλθει όταν η πρώτη μέρα κάποιου από τους επόμενους αιώνες συμπέσει να είναι Κυριακή.
Με αυτό το σκεπτικό ποια χρονιά θα είναι το τέλος του κόσμου;

Πρόβλημα 9
Δίνεται οξυγώνιο τρίγωνο ΑΒΓ. Να εγγραφεί σε αυτό τετράγωνο ΚΛΘΗ έτσι ώστε οι κορυφές Κ και Λ να
βρίσκονται πάνω στην ΑΒ, η κορυφή Η πάνω στην ΑΓ και η κορυφή Θ πάνω στην ΓΒ.

ΥΠΟΔΕΙΞΕΙΣ ΓΙΑ ΤΗΝ ΕΠΙΛΥΣΗ


Πιο κάτω γίνονται κάποιες υποδείξεις για την επίλυση των πιο πάνω προβλημάτων. Πρέπει να σημειωθεί
ότι δεν είναι αναγκαστικά και οι μόνες προσεγγίσεις που μπορείτε να ακολουθήσετε για την επίλυση των
προβλημάτων:

Πρόβλημα 1
Το γινόμενο του αριθμού 180 με το θετικό ακέραιο Ν είναι τέλειος κύβος. Ποια είναι η μικρότερη τιμή του
Ν;
Υποδείξεις
Κατανόηση του Τι ζητείται; Ποια τα δεδομένα και ποια τα ζητούμενα; Ξέρουμε τι σημαίνει
Προβλήματος τέλειος κύβος;
Διαμόρφωση ενός Η παραγοντοποίηση του 180 σε γινόμενο πρώτων αριθμών θα μπορούσε να
σχεδίου βοηθήσει;
Υλοποίηση του σχεδίου
Επανεξέταση/ Επανερχόμαστε στο αρχικό πρόβλημα για να επαληθεύσουμε αυτά που
αναθεώρηση/ βρήκαμε; Μήπως υπάρχουν και άλλοι τρόποι επίλυσης;
επέκταση

92
Λύση Προβλήματος

Μαθηματικό Βήμα

Πρόβλημα 2
Δίνονται τα ψηφία 5, 3, 9, 7, 2 και 4. Να σχηματίσετε με αυτά δύο τριψήφιους αριθμούς, χρησιμοποιώντας
κάθε ψηφίο μια φορά έτσι ώστε το άθροισμα και το γινόμενο των τριψηφίων να είναι το μεγαλύτερο
δυνατό.
Υποδείξεις
Κατανόηση του Τι ζητείται; Μπορούμε να επαναδιατυπώσουμε το πρόβλημα με δικά μας λόγια;
Προβλήματος Κατανοούμε όλες τις φράσεις / έννοιες που αναφέρονται στο πρόβλημα; Ποια τα
δεδομένα και ποια τα ζητούμενα; Μήπως θα μπορούσαμε να διατυπώσουμε ένα
απλούστερο πρόβλημα π.χ. με 4 ψηφία (διαφορετικά μεταξύ τους) 3,4,5,7
Διαμόρφωση ενός Δοκιμάζω να βρω αριθμούς με το άθροισμα μεγαλύτερο. Ποια η διαδικασία που θα
σχεδίου ακολουθήσω;
Για το γινόμενο τώρα ελέγξετε (με παραδείγματα) για ένα ορθογώνιο με σταθερή
περίμετρο πότε γίνεται να έχει μέγιστο εμβαδό;
Μήπως μπορείτε να αξιοποιήσετε αυτή την πληροφορία;
Διαμορφώστε ένα τελικό σχέδιο για το δοθέν πρόβλημα.
Υλοποίηση του Προσπαθήστε με υπομονή να υλοποιήσετε το σχέδιο που διαμορφώσατε. Αν δεν
σχεδίου μπορείτε να το προχωρήσετε σκεφτείτε τι δεν πάει καλά από το αρχικό σας σχέδιο
και κάμετε κάποιες αναθεωρήσεις πηγαίνοντας ξανά στο Στάδιο 2. Αν δεν
υλοποιείτε σκεφτείτε κάτι άλλο πηγαίνοντας ξανά στα Στάδιο 1 και Στάδιο 2.
Επανεξέταση/ Έλεγχος για τη λογικότητα των απαντήσεων; Επανερχόμαστε στο αρχικό
αναθεώρηση/ πρόβλημα για να επαληθεύσουμε αυτά που βρήκαμε; Μήπως υπάρχουν και άλλες
επέκταση απαντήσεις; Μήπως υπάρχουν και άλλοι τρόποι επίλυσης; Μήπως χρειάζεται να
διερευνήσουμε περισσότερο τις απαντήσεις μας; Μπορούμε να επεκτείνουμε/
γενικεύσουμε;

Πρόβλημα 3
Να λυθεί με δύο στρατηγικές τουλάχιστον το ακόλουθο πρόβλημα:
Ένας αστυνομικός σταθμός έχει 25 οχήματα εκ των οποίων μερικά είναι αυτοκίνητα (τετράκυκλα) και τα
υπόλοιπα μοτοσυκλέτες (δίκυκλες). Ο συνολικός αριθμός των τροχών τους είναι 70, Πόσα αυτοκίνητα και
πόσες μοτοσυκλέτες έχει ο σταθμός;

Υποδείξεις
Κατανόηση του Τι ζητείται; Μπορούμε να επαναδιατυπώσουμε το πρόβλημα με δικά μας λόγια;
Προβλήματος Κατανοούμε όλες τις φράσεις / έννοιες που αναφέρονται στο πρόβλημα; Ποια τα
δεδομένα και ποια τα ζητούμενα; Έχουμε αρκετά δεδομένα για να
προχωρήσουμε σε λύση;
Διαμόρφωση ενός Σχέδιο 1: Αν ήταν όλα τα οχήματα μοτοσυκλέτες πόσους τροχούς θα είχαν; Που
σχεδίου οφείλονται οι επιπλέον τροχοί;
Σχέδιο 2: Μπορώ να χρησιμοποιήσω βοηθητικούς αγνώστους και να
διαμορφώσω εξισώσεις;

93
Λύση Προβλήματος

Μαθηματικό Βήμα

Υλοποίηση του
σχεδίου
Επανεξέταση/ Έλεγχος για τη λογικότητα των απαντήσεων; Επανερχόμαστε στο αρχικό
αναθεώρηση/ πρόβλημα για να επαληθεύσουμε αυτά που βρήκαμε; Μήπως υπάρχουν και
επέκταση άλλες απαντήσεις;

Πρόβλημα 4

Ένας ποδηλάτης θέλει να διανύσει μια απόσταση 10 km ανάμεσα στις πόλεις Α και Γ με μέση ταχύτητα 40
km/h. Στο μέσο όμως της διαδρομής υπάρχει το χωριό Β και για να φτάσει κανείς εκεί πρέπει να ανέβει
έναν αρκετά ανηφορικό δρόμο. Όταν ο ποδηλάτης έφτασε στο χωριό Β διαπίστωσε ότι η μέση ταχύτητά
του ήταν μόνο 21 km/h. Με πόση ταχύτητα πρέπει να κατέβει την πλαγιά ταξιδεύοντας από το χωριό Β
προς την πόλη Γ ώστε η συνολική μέση ταχύτητα του να είναι 40 km/h, όπως αρχικά το σχεδίαζε.

Υποδείξεις
Κατανόηση του Τι ζητείται; Ποια τα δεδομένα και ποια τα ζητούμενα; Έχουμε αρκετά δεδομένα
Προβλήματος για να προχωρήσουμε σε λύση; Ξέρουμε την έννοια της μέσης ταχύτητας και
πως μπορούμε να τη βρούμε;

Διαμόρφωση ενός Ετοιμάστε κάποιο διάγραμμα/ σχήμα. Μπορούμε να υπολογίσουμε κάποιους


σχεδίου χρόνους και ποιους; Έχουμε κάποιους τύπους για τον υπολογισμό της
ζητούμενης ταχύτητας;
Υλοποίηση του σχεδίου Προσπαθήστε να υλοποιήσετε το σχέδιο σας

Επανεξέταση/ Έλεγχος για τη λογικότητα των απαντήσεων;


αναθεώρηση/
επέκταση

Λύση
Ο χρόνος που χρειάστηκε για να κινηθεί από Α σε Β (απόσταση 5 km) είναι 5/21 h.
Ο χρόνος γα να διανύσει την απόσταση ΑΓ με μέση ταχύτητα 40 km/h είναι 10/40=1/4 h.
Κατά συνέπεια την απόσταση ΒΓ πρέπει να τη διανύσει σε 1/4 - 5/21 = 1/84 h. Δηλαδή πρέπει να
αναπτύξει μέση ταχύτητα ⁄ km/h

Διερεύνηση
Προφανώς η λύση αυτή είναι αμφισβητούμενη γιατί δεν είναι δυνατό ένας ποδηλάτης να αναπτύξει
τέτοια ταχύτητα μέσα στα συμβατικά πλαίσια.

Λύστε το πρόβλημα αν η ταχύτητα ανάβασης ήταν 20 km/h αντί 21 km/h

94
Λύση Προβλήματος

Μαθηματικό Βήμα

Πρόβλημα 5
Σε μια πόλη υπάρχουν 4 ποδοσφαιρικές ομάδες, οι Άπιαστοι, οι Βλαμμένοι, οι Γέροι και οι Δράκοι. Σε μια
πρόσφατη διοργάνωση έπαιξαν μεταξύ τους από ένα παιγνίδι ανά δύο και συγκέντρωσαν την ακόλουθη
βαθμολογία:

ΟΜΑΔΑ ΒΑΘΜΟΙ
Δράκοι 6
Άπιαστοι 4
Βλαμμένοι 4
Γέροι 2
Δοθέντος ότι για νίκη μια ομάδα συγκεντρώνει 3 βαθμούς, για ισοπαλία 1 βαθμό και για ήττα 0 βαθμούς
να βρεθούν τα ακόλουθα:
(4) Πόσες συναντήσεις είχαν αποτέλεσμα ισοπαλία;
(5) Ποιο ήταν το αποτέλεσμα της συνάντησης μεταξύ των Δράκων κα των Γέρων;
(6) Αν ξέρετε ότι οι Βλαμμένοι νίκησαν τους Δράκους, μπορείτε να βρείτε ποιο ήταν το αποτέλεσμα
της κάθε συνάντησης;

Υποδείξεις
Το 6 πως μπορεί να προκύψει ως άθροισμα τριών προσθετέων από τους αριθμούς 0,1 και 3; Τι
συμπέρασμα προκύπτει;
Μπορείτε να οργανώσετε τις πληροφορίες σε πίνακα;

Πρόβλημα 6
Ο Δημήτρης και ο Μιχάλης θα πάνε από το σπίτι τους στην πόλη Α που βρίσκεται σε απόσταση 12 km.
Έχουν μόνο ένα ποδήλατο και αποφάσισαν να το χρησιμοποιούν έτσι ώστε ο ένας να ποδηλατεί για μια
απόσταση και ο άλλος να περπατά. Στη συνέχεια αυτός που ποδηλατούσε θα αφήνει το ποδήλατο στην
άκρη του δρόμου για να το πάρει ο άλλος όταν φτάσει εκεί και θα συνεχίσει με τα πόδια. Ο αρχικός
πεζοπόρος θα πάρει στη συνέχεα το ποδήλατο και θα διανύσει ποδηλατώντας την ίδια απόσταση όπως
και ο αρχικός ποδηλάτης. Θα επαναλαμβάνουν δε αυτή τη διαδικασία μέχρι που να φτάσουν στην πόλη Α.
Ξεκινούν και οι δύο την ίδια στιγμή. Δίδεται ότι ο Δημήτρης ποδηλατεί με ταχύτητα 15 km/h και περπατά
με ταχύτητα 6 km/h. Επίσης ξέρουμε ότι ο Μιχάλης ποδηλατεί με ταχύτητα 20 km/h και περπατά με
ταχύτητα 4 km/h.
Πόσο χρόνο θα τους πάρει για να πάνε στην πόλη αν χρησιμοποιήσουν την πιο καλή στρατηγική (αυτή
δηλαδή με την οποία θα επιτύχουμε το μικρότερο χρόνο διακίνησης προς την πόλη) ;

Υποδείξεις
Ας δοκιμάσουμε ως στρατηγική την οργάνωση της διαδικασίας να φτάσουν στην πόλη Α την ίδια στιγμή.
Μπορείτε να βρείτε το χρόνο που χρειάζονται για να επιτύχουν το στόχο τους με αυτή τη στρατηγική;
Σε ποια έκταση νομίζετε ότι έχει σημασία να χωρίσουν την απόσταση σε πολλά κομμάτια και να
επαναλαμβάνουν τη διαδικασία που αναφέρεται στο πρόβλημα ή να περιοριστούν σε ένα μόνο
διαχωρισμό;

95
Λύση Προβλήματος

Μαθηματικό Βήμα

Μήπως τα πράγματα αλλάζουν ανάλογα με το ποιος θα ξεκινήσει πρώτος;


Μπορείτε να αποδείξετε ότι ο χρόνος που επιτυγχάνεται με αυτή τη στρατηγική είναι ο πιο λίγος;

Πρόβλημα 7
Ένας περίεργος μαθητής λέει στο καθηγητή του των Μαθηματικών:
Άκουσα Κύριε ότι έχετε τρεις κόρες. Πόσον χρονών είναι;
Και ο καθηγητής του απαντά:
Το γινόμενο των ηλικιών τους είναι 72 και το άθροισμα των ηλικιών τους είναι ο αριθμός του δωματίου
στο οποίο βρισκόμαστε τώρα.
Ο μαθητής κάνει λίγες πράξεις και σε λίγο λέει στον καθηγητή του:
Κύριε δεν μας δώσατε αρκετά δεδομένα για μια μόνο απάντηση
Και ο καθηγητής απαντά:
Ω μα βέβαια ξέχασα να σας πω ότι η μεγαλύτερη κόρη μου έχει πράσινα μάτια.
Και ο μαθητής στη συνέχεια απαντά:
Μάλιστα τώρα είναι καθαρό οι ηλικίες τους είναι …..
Ποιες είναι οι ηλικίες που ζητούσε ο καθηγητής;

Υποδείξεις
Κατανόηση του Τι ζητείται; Ποια τα δεδομένα και ποια τα ζητούμενα; Έχουμε αρκετά δεδομένα
Προβλήματος για να προχωρήσουμε σε λύση. Τι είδους αριθμοί πρέπει να είναι οι ηλικίες;
Διαμόρφωση ενός Λαμβάνοντας υπόψη ότι το γινόμενο των ηλικιών είναι 72 ποια είναι η μορφή
σχεδίου των ηλικιών;
Μπορούμε να θεωρήσουμε διάφορες περιπτώσεις και να διαμορφώσουμε ένα
πίνακα;
Υλοποίηση του σχεδίου Προσπαθήστε να υλοποιήσετε το σχέδιο σας
Η πρόσθετη πληροφορία σε τι μας οδηγεί για να αποκλείσουμε;
Επανεξέταση/ Έλεγχος για τη λογικότητα των απαντήσεων;
αναθεώρηση/
επέκταση

Πρόβλημα 8
Μια ομάδα φανατικών οπαδών μιας θρησκευτικής αίρεσης, αξιοποιώντας τις πληροφορίες των γραπτών
τους κειμένων και τις δυνατότητες των ηλεκτρονικών υπολογιστών κατέληξε στο συμπέρασμα ότι το τέλος
του κόσμου θα έλθει όταν η πρώτη μέρα κάποιου από τους επόμενους αιώνες συμπέσει να είναι Κυριακή.
Με αυτό το σκεπτικό ποια χρονιά θα είναι το τέλος του κόσμου;

96
Λύση Προβλήματος

Μαθηματικό Βήμα

Υποδείξεις
Κατανόηση του Τι ζητείται; Κατανοούμε όλες τις φράσεις / έννοιες που αναφέρονται στο
Προβλήματος πρόβλημα; Ποια τα δεδομένα και ποια τα ζητούμενα;
Ξέρετε πως καθορίζεται η αρχή ενός αιώνα; Για το παρόν πρόβλημα δεχόμαστε
ότι η αρχή ενός αιώνα καθορίζεται ως η χρονιά που τα τελευταία δύο ψηφία
της είναι 00.
Ξέρετε πως καθορίζεται ένα δίσεκτο έτος με βάση το Γρηγοριανό Ημερολόγιο;
Ξέρετε ότι η 1η Ιανουαρίου 2000 ήταν Σάββατο
Διαμόρφωση ενός Σημαντικό στοιχείο που πρέπει να ληφθεί υπόψη είναι ποια χρόνια είναι
σχεδίου δίσεκτα και ποια όχι ώστε να μπορέσουμε να εκτιμούμε την 1 μέρα του
χρόνου ξεκινώντας από την 1η Ιανουαρίου 2000
Υλοποίηση του σχεδίου
Επανεξέταση/ Έλεγχος για τη λογικότητα των απαντήσεων Μήπως υπάρχουν και άλλοι
αναθεώρηση/ τρόποι επίλυσης;
επέκταση

Πρόβλημα 9
Δίνεται οξυγώνιο τρίγωνο ΑΒΓ. Να εγγραφεί σε αυτό τετράγωνο ΚΛΘΗ έτσι ώστε οι κορυφές Κ και Λ να
βρίσκονται πάνω στην ΑΒ, η κορυφή Η πάνω στην ΑΓ και η κορυφή Θ πάνω στην ΓΒ.

Υποδείξεις
Κατανόηση του Τι ζητείται; Μπορούμε να βρούμε κάτι βοηθητικό για να κατανοήσουμε το
Προβλήματος πρόβλημα (π.χ. ένα σχήμα); Μήπως θα μπορούσαμε να θεωρήσουμε απλές
περιπτώσεις που θα μας βοηθούσαν να κατανοήσουμε μερικές ειδικές
περιπτώσεις του προβλήματος;
Μπορείτε να λύσετε το πρόβλημα αν αντί οξυγώνιο τρίγωνο έχουμε ορθογώνιο
τρίγωνο με ορθή γωνία στο Α

Για την περίπτωση του αρχικού προβλήματος

97
Λύση Προβλήματος

Μαθηματικό Βήμα

Κατανόηση του ΑΝΑΛΥΣΗ: Έστω ότι έχουμε κατασκευάσει το ζητούμενο, δηλαδή έχουμε το
Προβλήματος

Μήπως θα μπορούσαμε να δούμε κάποια βοηθητική κατασκευή που να μας


οδηγεί στο βοηθητικό πρόβλημα; Αν φέρουμε από το Γ παράλληλη προς την ΑΒ
και από το Α κάθετη στην ΑΒ τι γίνεται;

Μήπως μπορούμε να κατασκευάσουμε το τετράγωνο το εγγεγραμμένο στο


τρίγωνο ΑΒΓ΄, που είναι τώρα ορθογώνιο;
Ποια σχέση έχουν τα σχήματα ΑΛ΄Μ΄Ν΄ και ΚΛΜΝ;
Διαμόρφωση ενός ΣΥΝΘΕΣΗ: προωθούμε ένα σχέδιο που περιλαμβάνει την κατασκευή της ΓΓ΄ ,
σχεδίου της ΑΓ΄ , του τετραγώνου ΑΛ΄Μ΄Ν΄ και του σχήματος ΚΛΜΝ . Στη συνέχεια
προχωρούμε στην Απόδειξη ότι το σχήμα που προκύπτει είναι τετράγωνο, έχει
τις ιδιότητες που απαιτεί το πρόβλημα και είναι μοναδικό
Υλοποίηση του σχεδίου ΚΑΤΑΣΚΕΥΗ: Κατασκευάζουμε τη ΓΓ΄ παράλληλη προς ΑΒ και ΑΓ΄ κάθετη στην
ΑΒ. Κατασκευάζουμε το τετράγωνο ΑΛ΄Μ΄Ν΄ . Προεκτείνουμε τις αναγκαίες
ευθείες για την κατασκευή του σχήματος ΚΛΜΝ .
ΑΠΟΔΕΙΞΗ: Αποδεικνύουμε ότι Ν΄Μ΄=ΝΜ και στη συνέχεια ότι το σχήμα ΚΛΜΝ
τετράγωνο που ικανοποιεί τις ιδιότητες του προβλήματος.
Επανεξέταση/ ΔΙΕΡΕΥΝΗΣΗ: Εξετάζουμε αν είναι δυνατές οι κατασκευές που προηγήθηκαν ή
αναθεώρηση/ υπάρχουν περιορισμοί.
επέκταση Ελέγχουμε αν η απόδειξη μας ισχύει στις ακόλουθες περιπτώσεις:
(α) Η γωνία ΓΑΒ μεγαλύτερη από 45ο και οξεία
(β) Η γωνία ΓΑΒ μικρότερη από 45ο
(γ) Η γωνία ΓΑΒ ίση με 45ο
Εξετάζουμε αν η λύση που βρήκαμε είναι μοναδική ή υπάρχουν και άλλες.

98
Διασκεδαστικά Μαθηματικά

Μαθηματικό Βήμα

Διασκεδάζοντας με τα Μαθηματικά
«Σκέφτομαι. Κατά συνέπεια Υπάρχω»
Ανδρέας Σκοτεινός
I. Μαθηματικά Προβλήματα
1. Βρέστε το Λάθος
Στην πιο κάτω αποδεικτική διαδικασία να προσδιορίσετε το στάδιο και το λόγο για τον οποίο υπάρχει
Λάθος

Απόδειξη ότι 2=1

Ισχυρισμός Δικαιολογία/ Τεκμηρίωση Αναφορά στις ιδιότητες

Υπόθεση/ Δεδομένο

Πολλαπλασιάζουμε και τα δύο Πολλαπλασιασμός κατά μέλη


μέρη επί a δύο ισοτήτων

Αφαιρούμε και από τα δύο μέρη Αφαίρεση κατά μέλη δύο


το b2 ισοτήτων

( )( ) ( ) Παραγοντοποιούμε και τα δύο Διαφορά τετραγώνων και


μέρη κοινός παράγοντας

( ) Διαιρούμε και τα δύο μέρη με το Ιδιότητα της διαγραφής


a-b

( ) Αντικαθιστούμε το b με το a

Προσθέτουμε

Διαιρούμε και τα δύο μέρη με το Ιδιότητα της διαγραφής


a

2. Πως ένας μαθηματικός μπορεί να γίνει ΜΑΓΟΣ


Ένας Μαθηματικός θέλοντας να εντυπωσιάσει μια ομάδα παιδιών τους είπε ότι θα μπορούσε να
μαντέψει τις σκέψεις τους χωρίς δυσκολία αρκεί να ακολουθούσαν μια σειρά από οδηγίες του.
Ξεκίνησε λοιπόν ως εξής:

99
Διασκεδαστικά Μαθηματικά

Μαθηματικό Βήμα

i. Σκεφτείτε ένα τριψήφιο ακέραιο αριθμό και ξαναγράψετε τον ίδιο με επανάληψη ώστε να
σχηματιστεί ένας εξαψήφιος αριθμός. (π.χ. αν σκεφτήκατε το 258 θα σχηματιστεί ο
εξαψήφιος 258258)
ii. Διαιρέστε τον αριθμό αυτό (τον εξαψήφιο) με το 7. Τότε ισχυρίζομαι ότι ο αριθμός που
βρήκατε είναι ακέραιος (δηλαδή το υπόλοιπο της διαίρεσης είναι το 0). Τι λέτε;
iii. Διαιρέστε τον αριθμό που βρήκατε (μετά τη διαίρεση με το 7) με το 11. Τότε ισχυρίζομαι
ότι ο αριθμός που βρήκατε είναι ακέραιος (δηλαδή το υπόλοιπο της διαίρεσης είναι το 0).
Τι λέτε;
iv. Διαιρέστε τον αριθμό που βρήκατε (μετά τη διαίρεση με το 7 και με το 11) με το 13. Τότε
ισχυρίζομαι ότι ο αριθμός που βρήκατε είναι ακέραιος (δηλαδή το υπόλοιπο της διαίρεσης
είναι το 0). Τι λέτε;
v. Ισχυρίζομαι επίσης ότι ο νέος αριθμός που βρήκατε μετά τις τρεις διαιρέσεις είναι ο
αρχικός τριψήφιος που σκεφτήκατε. Τι λέτε;
Μπορείτε να δώσετε μια μαθηματική εξήγηση για το πώς ο Μαθηματικός κατάφερνε να μαντεύει
και να δίνει σωστά τις πιο πάνω απαντήσεις;

II. Γρίφοι
1. Πως είναι δυνατόν να προσθέσετε οκτώ 8-ρια και να βρείτε τον αριθμό 1000;
2. Μια ομάδα ατόμων από τους οποίους οι δύο είναι πατεράδες και οι δύο είναι γιοι έφαγαν
ακριβώς τρία αυγά για πρωινό και ο καθένας από αυτούς έφαγε ακριβώς ένα αυγό. Μπορείτε να
εξηγήσετε αυτό το γεγονός;
3. Δίνονται όλοι οι ακέραιοι από το 1 μέχρι το 1000 (συμπεριλαμβανομένων). Να βρείτε ποιο ψηφίο
είναι που παρουσιάζεται πιο συχνά και ποιο ψηφίο λιγότερο συχνά.
4. Ένας λογικολόγος (θεράπων της Λογικής) θα λέγεται ακριβής αν οτιδήποτε μπορεί να αποδείξει
είναι αληθές ενώ δεν αποδεικνύει τίποτα εσφαλμένο. Μια μέρα ένας ακριβής λογικολόγος
επισκέφθηκε το νησί των Ιπποτών και των Ιπποκόμων, στο οποίο κάθε κάτοικος είναι ιππότης ή
ιπποκόμος (αλλά όχι και τα δύο) και οι δηλώσεις των ιπποτών είναι πάντοτε αληθείς, ενώ των
ιπποκόμων είναι πάντοτε ψευδείς. Ο λογικολόγος συνάντησε ένα ντόπιο ο οποίος έκανε μια
δήλωση από την οποία προκύπτει ότι ο άνθρωπος αυτός πρέπει να ήταν ιππότης, όμως ο
λογικολόγος δεν μπορούσε με κανένα τρόπο να το αποδείξει. Ποια δήλωση επιτυγχάνει αυτό το
αποτέλεσμα;
(Σημείωση: Ο γρίφος αυτός σχετίζεται στενά με το Θεώρημα της Μη Πληρότητας του Gödel.)

III. Μαθηματικά Παιγνίδια


1. Στο πιο κάτω σχήμα (κατασκευασμένο από σπίρτα) να αφαιρέσετε ένα σπίρτο ώστε να
απομείνουν τρία τετράγωνα

100
Διασκεδαστικά Μαθηματικά

Μαθηματικό Βήμα

2. Στο πιο κάτω σχήμα (κατασκευασμένο από σπίρτα) να μετακινήσετε 4 σπίρτα ώστε να
απομείνουν τρία ισόπλευρα τρίγωνα

3. Να λύσετε τα ακόλουθα Sudoku


Συμπληρώστε τις θέσεις στα μικρά τετραγωνάκια με τα ψηφία 1, 2, … 9 έτσι ώστε σε κάθε
στήλη και σε κάθε γραμμή του μεγάλου (9x9) τετραγώνου και σε κάθε θέση των (3x3)
σημειωμένων τετραγώνων τα 9 ψηφία να παρουσιάζονται ακριβώς μια φορά.
Σημειώστε ότι ενδείκνυται να κάμετε λογικούς συλλογισμούς για τη συμπλήρωση αυτή και
να αποφύγετε την τυχαία τοποθέτηση.
(α) ΕΥΚΟΛΟ

101
Διασκεδαστικά Μαθηματικά

Μαθηματικό Βήμα

(β) ΜΕΤΡΙΟ

(γ) ΔΥΣΚΟΛΟ

102
Διασκεδαστικά Μαθηματικά

Μαθηματικό Βήμα

(δ) ΠΟΛΥ ΔΥΣΚΟΛΟ

IV. Μαθηματικά Ανέκδοτα


1. Μια μητέρα έχει ήδη 4 παιδιά και είναι έγκυος και περιμένει το πέμπτο παιδί της. Μια μέρα
λοιπόν η μεγαλύτερη κόρη, Μαρία, που είναι πολύ μελετηρή, λέει στον πατέρα της:
Μαρία: Ξέρεις μπαμπά τι έχω βρει;
Μπαμπάς: ‘Όχι.
Μαρία: Το νέο μας μωρό θα είναι Κινεζάκι.
Μπαμπάς: Τι έκανε λέει;
Μαρία: Μάλιστα, θα είναι Κινεζάκι. Έχω διαβάσει ένα άρθρο που λέει ότι στατιστικά το κάθε
πέμπτο παιδί που γεννιέται στον κόσμο είναι Κινέζος.

2. Μερικοί μηχανικοί προσπαθούν να μετρήσουν τον ιστό μιας σημαίας. Έχουν μόνο μια μετρική
κορδέλα και είναι απογοητευμένοι προσπαθώντας να κρατήσουν την κορδέλα κατά μήκος του
ιστού.
Εκείνη την ώρα περνούσε και ένας μαθηματικός από εκεί και τους ρώτησε τι κάνουν. Εκείνοι του
εξήγησαν ότι θέλουν να μετρήσουν τον ιστό. « Ω μα αυτό είναι εύκολο» τους λέει και τραβά τον
ιστό από το έδαφος, τον ξαπλώνει οριζόντια και τον μετρά με την κορδέλα.
Μόλις έφυγε, οι μηχανικοί αρχίζουν να σχολιάζουν: ‘Τι τυπική συμπεριφορά για ένα μαθηματικό!
Όταν εμείς θέλουμε το ύψος εκείνος μας δίνει το μήκος!»

103
Διασκεδαστικά Μαθηματικά

Μαθηματικό Βήμα

3. Η Κυπριακή Μαθηματική Εταιρεία αποφάσισε να οργανώσει λαχείο και τα έσοδα από αυτό να τα
διαθέσει για να καλύψει τα έξοδα του Υπουργείου Παιδείας για το πρόγευμα των άπορων
παιδιών. Ως χρηματικό έπαθλο ανακοίνωσε ότι ο λαχνός που κερδίζει θα δικαιούται ένα άπειρο
ποσό σε ευρώ που θα το πάρει σε άπειρο χρόνο..
Όπως καταλαβαίνετε με ένα τόσο δελεαστικό ποσό όλοι οι λαχνοί, που ήταν εκατοντάδες χιλιάδες
πωλήθηκαν σε ελάχιστο χρόνο. Όταν έγινε η κλήρωση αυτός που είχε τον τυχερό λαχνό
παρουσιάστηκε στον Πρόεδρο της ΚΥΜΕ και ζήτησε τα κέρδη του. Οπότε ο Πρόεδρος του είπε ότι
θα πάρει τα λεφτά του με τον εξής τρόπο: «Την πρώτη μέρα θα πάρεις ένα ευρώ, τη δεύτερη
μέρα ½ του ευρώ, την τρίτη μέρα 1/3 του ευρώ, την τέταρτη μέρα ¼ του ευρώ και ούτω καθεξής».

4. Σε μια τάξη ο δάσκαλος προσπαθεί να διδάξει το κλάσματα και αρχίζει να ρωτά τους μαθητές:
Δάσκαλος: Έχουμε ένα κομμάτι κρέας και το μοιράζουμε, με ένα μαχαίρι, ακριβώς στα δύο. Τι θα
πάρουμε;
Πρώτος μαθητής: 2/2 Κύριε
Δάσκαλος: Ξαναμοιράζουμε το κάθε ένα από αυτά τα κομμάτια και πάλι στα δύο. Τι θα πάρουμε;
Δεύτερος μαθητής: 4/4 Κύριε
Δάσκαλος: Ξαναμοιράζουμε το κάθε ένα από αυτά τα κομμάτια και πάλι στα δύο. Τι θα πάρουμε;
Τρίτος μαθητής: 8/8 Κύριε
Και συνεχίζει έτσι ο δάσκαλος μέχρι που φτάνει στον Μπόμπο
Δάσκαλος: Ξαναμοιράζουμε το κάθε ένα από αυτά τα κομμάτια και πάλι στα δύο. Τι θα πάρουμε;
Μπόμπος: Κιμά Κύριε

5. Στο μάθημα για τα όρια ο καθηγητής εξήγησε συστηματικά γιατί έχουμε

Στη συνέχεια για να ελέγξει τι έμαθαν οι μαθητές έδωσε ένα παρόμοιο παράδειγμα ζητώντας από
τους μαθητές να του δώσουν την τιμή του

Ένας λοιπόν μαθητής έδωσε την ακόλουθη απάντηση:

104
Διαγωνισμός «Kangourou»

Μαθηματικό Βήμα

ΠΡΩΤΟΤΥΠΕΣ ΑΣΚΗΣΕΙΣ ΑΠΟ ΤΑ ΘΕΜΑΤΑ KANGOUROU ΜΕ ΤΙΣ ΛΥΣΕΙΣ

Ανδρέας Σαββίδης

1. Ο αριθμός 2004 είναι διαιρετός δια του 12 και το άθροισμα των ψηφίων του είναι ίσο με 6. Πόσοι,
από όλους τους τετραψήφιους αριθμούς έχουν αυτή την ιδιότητα;

A: 10 B: 12 Γ: 13 Δ: 15 Ε: 18

Λύση
Ζητούμε όλους τους τετραψήφιους αριθμούς που έχουν άθροισμα ψηφίων 6 και είναι πολλαπλάσιοι του
4, αφού είναι πολλαπλάσιοι του 12, άρα τα δύο τελευταία τους ψηφία πρέπει να διαιρούνται με το 4.
Αριθμοί Πλήθος αριθμών
Με ένα ψηφίο το 6 6000 1
Με δύο ψηφία τα 1,5 1500, 5100 2
Με δύο ψηφία τα 2,4 2400, 4200, 2004, 2040, 4020 5
Με δύο ψηφία τα 3,3 3300 1
Με τρία ψηφία τα 1,1,4 1140, 1104 2
Με τρία ψηφία τα 1,2,3 1320, 3120, 3012, 1032 4
Με τρία ψηφία τα 2,2,2 2220 1
Με τέσσερα ψηφία 1,1,2,2 1212, 2112 2
Με τέσσερα ψηφία 1,1,1,3 - 0
Σύνολο 18

Επιλογή: Ε

2. Στα μικρά τετράγωνα του μεγάλου τετραγώνου τοποθετούνται οι διαδοχικοί φυσικοί αριθμοί όπως
φαίνονται στο σχήμα. Ποιος από τους πιο κάτω αριθμούς δεν μπορεί να τοποθετηθεί στη θέση του x;

10
4 9
3 5 8
1 2 6 7

Α: 128 Β: 256 Γ: 81 Δ: 121 Ε: 400

105
Διαγωνισμός «Kangourou»

Μαθηματικό Βήμα

Λύση
Το μεγάλο τετράγωνο χωρίζεται σε μικρά τετράγωνα. Το πλήθος αυτών των μικρών τετραγώνων πρέπει να
είναι τετράγωνο κάποιου αριθμού, π.χ. 52, 82, 122, 152 κ.ο.κ. Άρα ο αριθμός x, που είναι ο τελευταίος
αριθμός στα μικρά τετράγωνα, θα είναι τετράγωνος αριθμός. Ο μη τετράγωνος αριθμός στις επιλογές
είναι το 128.
Επιλογή: Α

3. Τα μέσα των ευθύγραμμων τμημάτων που ανά δύο συνδέουν τις κορυφές ενός κύβου είναι:

Α: 8 Β: 12 Γ: 16 Δ: 19 Ε: 28

Λύση
Ο κύβος έχει 8 κορυφές, A, B, C, ..., H. Αν ενώσουμε, ανά δύο τις 8 κορυφές σχηματίζονται 28 ευθύγραμμα
τμήματα, αφού ( ) . Τα 28 ευθύγραμμα τμήματα χωρίζονται σε:
12 ακμές του κύβου AB, BC, CD, ..., FG, GH, HE που έχουν 12 μέσα,
12 διαγωνίους των εδρών του κύβου AC, BD, BG, CF, ..., EG, HF που έχουν 6 μέσα και 4 διαγωνίους του
κύβου AG, BH, CE, DF που έχουν ένα μέσο.
Συνολικά έχουμε 12 + 6 + 1 = 19 μέσα.
Επιλογή: Δ

4. Για κάθε διψήφιο αριθμό αφαιρούμε το ψηφίο των μονάδων από το ψηφίο των δεκάδων. Αν
προσθέσουμε όλα τα αποτελέσματα που θα βρούμε, το άθροισμα θα είναι:

Α: 90 Β: 100 Γ: 55 Δ: 45 Ε: 30

Λύση
Οι διψήφιοι αριθμοί είναι 10, 11, 12, ..., 98, 99. Το πρόβλημα μας ζητά να προσθέσουμε όλους τους
αριθμούς που εμφανίζονται στον πιο κάτω πίνακα
1-0 1-1 1-2 1-3 ... 1-8 1-9
2-0 2-1 2-2 2-3 ... 2-8 2-9
3-0 3-1 3-2 3-3 ... 3-8 3-9
. . . . . .
. . . . . .
. . . . . .
8-0 8-1 8-2 8-3 ... 8-8 8-9
9-0 9-1 9-2 9-3 ... 9-8 9-9

Παρατηρούμε ότι

106
Διαγωνισμός «Kangourou»

Μαθηματικό Βήμα

α) οι διαφορές των αριθμών που ανήκουν στη διαγώνιο, που αρχίζει από το 1-1 και καταλήγει στο 9-9,
είναι όλες 0.
β) οι συμμετρικοί αριθμοί ως προς αυτή τη διαγώνιο είναι αντίθετοι άρα έχουν άθροισμα 0. Π.χ. 1-2 και 2-
1, 1-8 και 8-1, 3-5 και 5-3, 8-9 και 9-8. Άρα όλοι οι αριθμοί από τη 2η στήλη μέχρι και την τελευταία έχουν
άθροισμα 0. Οπότε παραμένει η 1η στήλη που είναι:
.
Επιλογή: Δ

5. Ένας πεζοπόρος κάνει βόλτα διάρκειας 2 ωρών που αποτελείται από: αρχικά ένα επίπεδο δρόμο και
ακολούθως ένα ανηφορικό δρόμο και μετά επιστροφή πίσω (πρώτα πηγαίνει στον κατηφορικό δρόμο
και μετά στον επίπεδο δρόμο). Η ταχύτητά του είναι 4km/h στο επίπεδο μέρος, 3km/h στο ανηφορικό
μέρος και 6km/h στο κατηφορικό μέρος του δρόμου. Πόση απόσταση ήταν η βόλτα του πεζοπόρου;

Α: Δεν μπορούμε να ξέρουμε Β: 6km Γ: 7,5km Δ: 8km Ε: 10km

Λύση
Ζητείται η απόσταση 2(ΑΒ + ΒΓ) = 2 (S1 + S2). 6km/h Γ
Ο χρόνος που χρειάζεται για να καλύψει την
3km/h
απόσταση ΑΒ και την ΒΑ είναι ο ίδιος, έστω t1. A 4km/h B
Για την απόσταση ΒΓ χρειάζεται χρόνο t2 και για
την απόσταση ΓΒ χρειάζεται χρόνο t3. Αφού έχουμε:

Ώστε: 2(s1+s2) = 8km.


Επιλογή: Δ

6. Γράφονται σε ένα πίνακα οι φυσικοί αριθμοί από το 1 μέχρι το 10. Οι μαθητές της τάξης παίζουν το
εξής παιγνίδι. Ένας μαθητής σβήνει 2 αριθμούς και στη θέση τους γράφει στον πίνακα το άθροισμα
τους μειωμένο κατά 1, μετά άλλος μαθητής σβήνει 2 αριθμούς και στη θέση τους γράφει το άθροισμά
τους μειωμένο κατά 1, κ.ο.κ. Το παιγνίδι συνεχίζεται μέχρι να μείνει ένας αριθμός στον πίνακα. Ο
αριθμός που μένει είναι:

Α: μικρότερος του 11 Β: 11 Γ: 46 Δ: μεγαλύτερος του 46 Ε: άλλη απάντηση

Λύση
Ο 1ος μαθητής σβήνει 2 αριθμούς και γράφει 1 άρα μένουν 8 αριθμοί. Ο 2ος μαθητής ομοίως άρα μένουν
7 αριθμοί κ.λ.π. 'Ωστε για να μείνει ένας αριθμός στον πίνακα το παιγνίδι παίζεται 9 φορές. Σε αυτές τις 9
φορές, αφού κάθε φορά αφαιρούμε μία μονάδα, αφαιρούμε από το άθροισμα των αριθμών 1 έως 10 το 9.
Ήτοι

Επιλογή: Γ

107
Ετήσιες Δραστηριότητες ΚΥΜΕ

Μαθηματικό Βήμα

ΔΡΑΣΤΗΡΙΟΤΗΤΕΣ ΤΗΣ ΚΥΠΡΙΑΚΗΣ ΜΑΘΗΜΑΤΙΚΗΣ ΕΤΑΙΡΕΙΑΣ


ΚΑΤΑ ΤΟ 2013

Η Κυπριακή Μαθηματική Εταιρεία κατά το 2013 έχει πραγματοποιήσει τις παρακάτω δραστηριότητες
 Την Πέμπτη 31 Ιανουαρίου 2013 έγινε με μεγάλη επιτυχία ο διαγωνισμός
«Μαθηματική Σκυταλοδρομία για τα Γυμνάσια 2013».
Στον διαγωνισμό, που είναι ομαδικός και έγινε στην Πανεπιστημιούπολη του Πανεπιστημίου Κύπρου,
πήραν μέρος 68 Γυμνάσια από όλη την Κύπρο με 9μελείς ομάδες το καθένα.

108
Ετήσιες Δραστηριότητες ΚΥΜΕ

Μαθηματικό Βήμα

 Το 15ο Παγκύπριο συνέδριο Μαθηματικής Παιδείας και Επιστήμης, όπως και το 9ο Παγκύπριο
Μαθητικό Συνέδριο για τα Μαθηματικά, έγιναν το τριήμερο 8 – 10 Μαρτίου 2013. Τα συνέδρια
οργανώθηκαν στον Αγρό, στο ξενοδοχείο Ροδον.

Για πρώτη φορά διοργανώθηκε στο πλαίσιο του Συνεδρίου, ο Διαγωνισμός MATHfactor, με πολύ
μεγάλη επιτυχία

109
Ετήσιες Δραστηριότητες ΚΥΜΕ

Μαθηματικό Βήμα

 Το 5ο Ευρωπαϊκό Μαθητικό Συνέδριο για τα Μαθηματικά (Euromath 2013) πραγματοποιήθηκε στο


Γκέτεμποργκ Σουηδίας από 10 έως 14 Απριλίου 2013.

Στο συνέδριο έλαβαν μέρος μαθητές 12 – 18 ετών από πολλές Ευρωπαϊκές και άλλες χώρες, με
παρουσίαση εισηγήσεων ή παρακολουθώντας τις εργασίες του Συνεδρίου.
Στο Συνέδριο παρουσιάστηκαν 135 εισηγήσεις μαθητών και οργανώθηκαν 9 εργαστήρια (workshops), που
όλα αφορούσαν στα Μαθηματικά.

Επίσης διοργανώθηκε για δεύτερη φορά στα πλαίσια του συνεδρίου ο διαγωνισμός MATHfactor, που είχε
πολλές συμμετοχές και σημείωσε μεγάλη επιτυχία.

110
Ετήσιες Δραστηριότητες ΚΥΜΕ

Μαθηματικό Βήμα

 Την Κυριακή 21 Απριλίου 2013 πραγματοποιήθηκε η 14η Κυπριακή Μαθηματική Ολυμπιάδα στην
οποία πήραν μέρος 3500 περίπου μαθητές Γ΄ Δημοτικού μέχρι Γ΄ Λυκείου από όλη την Κύπρο. Ο
διαγωνισμός διεξήχθη ταυτόχρονα σε 5 πόλεις της Κύπρου.

 Από 28 Ιουνίου έως 3 Ιουλίου διοργανώθηκε στον Αγρό η 30η Βαλκανική Μαθηματική Ολυμπιάδα
(ΒΜΟ 2013) με τη συμμετοχή όλων των χωρών – μελών της ΒΜΟ, εκτός της Τουρκίας. Επίσης πήραν
μέρος και 6 χώρες μη μέλη. Συγκεκριμένα, το Αζερμπαιζάν, το Ηνωμένο Βασίλειο, η Ιταλία, το
Καζακστάν, το Τατζικιστάν και το Τουρκμενιστάν.

Ο Διαγωνισμός ανατέθηκε εκτάκτως στην Κύπρο και, παρόλη τη χρονική στενότητα, η διοργάνωση
ήταν εξαιρετικά επιτυχής.

111
Ετήσιες Δραστηριότητες ΚΥΜΕ

Μαθηματικό Βήμα

 Το 23ο Καλοκαιρινό Μαθηματικό Σχολείο λειτούργησε στον Αγρό από 1 έως 16 Ιουλίου
2013, σε τρείς διαφορετικές περιόδους , για μαθητές Α΄ Γυμνασίου μέχρι Α΄ Λυκείου.

 Η Κύπρος έλαβε μέρος σε 5 Διεθνείς Διαγωνισμούς, που διοργανώθηκαν μέσα στο 2013.
Συγκεκριμένα:
 Στη 17η Βαλκανική Μαθηματική Ολυμπιάδα Νέων (JBMO 2013),
Τουρκία 21 – 26 Ιουνίου

 Στην 30η Βαλκανική Μαθηματική Ολυμπιάδα (ΒΜΟ 2013),


Κύπρος 28 Ιουνίου – 3 Ιουλίου
 Στο Διεθνή Διαγωνισμό Μαθηματικών Νέων < 14 χρόνων (IMC 2013),
Βουλγαρία 30 Ιουνίου – 5 Ιουλίου
 Στο 1ο Μεσογειακό Μαθηματικό Πρωτάθλημα Νεολαίας ( MYMC 2013),
Ιταλία 17 – 19 Ιουλίου
 Στην 54η Διεθνή Μαθηματική Ολυμπιάδα (ΙΜΟ 2013),
Κολομβία 18 – 28 Ιουλίου

 Το δίμηνο Οκτωβρίου – Νοεμβρίου, η ΚΥΜΕ οργάνωσε και πρόσφερε δωρεάν σε όλες τις Επαρχίες
βοήθεια για μαθητές Α΄ Γυμνασίου και Α΄ Λυκείου στο πλαίσιο κοινωνικής προσφοράς, με την

112
Ετήσιες Δραστηριότητες ΚΥΜΕ

Μαθηματικό Βήμα

ευκαιρία των 30 χρόνων από την ίδρυσή της. Το πρόγραμμα θα επαναληφθεί το δίμηνο Μαρτίου –
Απριλίου 2014

 Πραγματοποιήθηκαν το Νοέμβριο και το Δεκέμβριο αντίστοιχα οι Επαρχιακοί Διαγωνισμοί και ο


Παγκύπριος Διαγωνισμός για όλα τα επίπεδα.

113

You might also like